主題:公共管理的基礎理論 - web.thu.edu.twweb.thu.edu.tw/lu.bk/www/news/99-1.pdf ·...

115
主題:公共管理的基礎理論 焦點一:試說明Weber對傳統官僚制度的反省。 【思考方向】 Weber 建構了理性-法律的科層官僚制之後,Weber 本人亦對科 層官僚制之未來發展抱著戒慎恐懼與「放心不下」的心情。一方 面,理性-法律的科層官僚制具有治理的技術優越性與不可或缺 性;另一方面,理性-法律的科層官僚制在政治上會逐漸失去控 制,在社會上會形成特殊的階級,和對人性的貶抑。 (一)Weber 曾云:「充分發展了的官僚機構相較於其他組織, 正如生產模式之機械與非機械的比較。精確、迅速、果決、 檔案的知識、持續、謹慎、一致、嚴格的服從、摩擦、和 物資人力成本的降低—這些都在官僚的管理中被提高到最 適當的地位。」科層官僚制發展得愈完全,它便愈去人性 化,愈能成功地在公務行政中排除愛、恨和所有純屬個人 的、非理性的、和未經計算過的情緒性因素。相對的,理 性官僚制的形式化、精確性、可計算性等技術上的優勢地 位則就被凸顯出來了。 (二)依據Weber 的觀點,理性官僚體制之講求對事不對人、形 式主義普遍標準的精神,這都有助於民主政治中平等原則 的實現。 (三)從統治者的觀點來看,他們必須依賴龐大的官僚系統來強 化其統治;從廣大民眾的觀點來看,人民期待官僚制度能 夠加強為民服務,並解決社會上層出不窮的問題。這使得 我們實在無法想像,一旦官僚體制停止運轉,社會將呈現 什麼樣的亂象。對Weber 而言,馬克思(K. Marx)所主張 的「無產階級專政」,只是一個夢想,實施社會主義「將 會導致一個比在資本主義社會中更為官僚制度化的國 家」;而且,事實上,「社會主義比資本主義可能需要更 高程度的形式官僚化」。 (四)Weber 認為科層官僚制會利用其專業知識和公務機密,而 掠奪各種利益和權力,並擺脫外界的監督。Weber 曾云:

Upload: others

Post on 09-Sep-2019

26 views

Category:

Documents


0 download

TRANSCRIPT

Page 1: 主題:公共管理的基礎理論 - web.thu.edu.twweb.thu.edu.tw/lu.bk/www/news/99-1.pdf · 主題:公共管理的基礎理論 焦點一:試說明Weber對傳統官僚制度的反省。

主題:公共管理的基礎理論

焦點一:試說明Weber對傳統官僚制度的反省。【思考方向】

Weber 建構了理性-法律的科層官僚制之後,Weber 本人亦對科

層官僚制之未來發展抱著戒慎恐懼與「放心不下」的心情。一方

面,理性-法律的科層官僚制具有治理的技術優越性與不可或缺

性;另一方面,理性-法律的科層官僚制在政治上會逐漸失去控

制,在社會上會形成特殊的階級,和對人性的貶抑。

(一)Weber 曾云:「充分發展了的官僚機構相較於其他組織,

正如生產模式之機械與非機械的比較。精確、迅速、果決、

檔案的知識、持續、謹慎、一致、嚴格的服從、摩擦、和

物資人力成本的降低—這些都在官僚的管理中被提高到最

適當的地位。」科層官僚制發展得愈完全,它便愈去人性

化,愈能成功地在公務行政中排除愛、恨和所有純屬個人

的、非理性的、和未經計算過的情緒性因素。相對的,理

性官僚制的形式化、精確性、可計算性等技術上的優勢地

位則就被凸顯出來了。

(二)依據Weber 的觀點,理性官僚體制之講求對事不對人、形

式主義普遍標準的精神,這都有助於民主政治中平等原則

的實現。

(三)從統治者的觀點來看,他們必須依賴龐大的官僚系統來強

化其統治;從廣大民眾的觀點來看,人民期待官僚制度能

夠加強為民服務,並解決社會上層出不窮的問題。這使得

我們實在無法想像,一旦官僚體制停止運轉,社會將呈現

什麼樣的亂象。對Weber 而言,馬克思(K. Marx)所主張

的「無產階級專政」,只是一個夢想,實施社會主義「將

會導致一個比在資本主義社會中更為官僚制度化的國

家」;而且,事實上,「社會主義比資本主義可能需要更

高程度的形式官僚化」。

(四)Weber 認為科層官僚制會利用其專業知識和公務機密,而

掠奪各種利益和權力,並擺脫外界的監督。Weber 曾云:

Page 2: 主題:公共管理的基礎理論 - web.thu.edu.twweb.thu.edu.tw/lu.bk/www/news/99-1.pdf · 主題:公共管理的基礎理論 焦點一:試說明Weber對傳統官僚制度的反省。

「正常態狀況下,充分發展的官僚體制總是超越其原有的

權力地位,而政務首長會發現在面對受有訓練的官僚專家

面前,他只是個外行主義者或一知半解者(dilettante)。」

(五)在Weber 看來,民主政治主張人民應有參政權利和政府施

政應儘量以民意為依歸,而理性官僚體制卻以專家自居,

固守其「專業知識」,而與民主原則不符。甚至,在嚴密

的層級體制內,終日與公文為伍,終年案牘勞形,自成為

一封閉體系,儼然像個特殊階級。再加上科層官僚制中設

定許多所謂「機密性」的公務,不願將公務處理的過程向

外界公開,往往使其具有不可替換性,而得以鞏固基本的

地位。

(六)隨著「官僚化」的發展,個人自願或非自願地被納入理性

-法律的科層官僚體制之支配下,每個人都變成這個不停

運轉機器上的一個小小齒輪,並照著指定路線行動,人的

自主意願會將被秩序和紀律所取代,不自主地淪為這部大

機器內的一個小螺絲釘,而處處顯得無力而渺小,只能循

規蹈矩地「運轉」。這種「去人性化」的特性,當官僚制

度愈是充分發展時,愈是嚴重。有鑑於此,Weber 認為人

類的未來將被束縛在鐵的牢籠(the iron cage)之中。

焦點二:試說明第一次明諾布魯克會議的內涵。

【思考方向】

根據Marini 的分析,新公共行政運動的主要特徵為(Marini, 1971:

348-352):

1、趨向相關的(relevant)公共行政在探索「知識何用?」(knowledge

for what?)的提問下,Kronenberg 為此下了結論,道:「知識應

在促使我們使用科學,並善用我們的技術與才能來改善人類的條

件。」而Marini 歸納指出今後公共行政應致力於研究下列數個問

題(Marini,1971: 348):

(1)研究動盪不安時代的相關問題,如分權、組織退化

(organizational devolution)、及參與觀念等與我們日常生活相關

的問題。

Page 3: 主題:公共管理的基礎理論 - web.thu.edu.twweb.thu.edu.tw/lu.bk/www/news/99-1.pdf · 主題:公共管理的基礎理論 焦點一:試說明Weber對傳統官僚制度的反省。

(2)研究行政學術的相關問題,例如比較都市行政、行政區域比

較、和組織單元異同的比較等。

(3)研究行政實務者的相關問題;如設計規劃預算制度(PPBS)

如何分權化和具有參與管理精神,以成為變遷的機制等。

2、採用後邏輯實證論(postpositivism)

新公共行政學者認為以往實證論所主張的「價值中立」(value-free)

是種危險的誤導,它忽略了行政理論建構所必須注意的政治與道

德本質。社會公正(social equity)而非效率才是學術研究直接面

對的主要價值。尤其是,社會科學的新近發展,如人本心理學

(humanistic psychology)、存在主義(existentialism)、現象學

(phenomenology)、批判理論(critical theory)及其他學說的發展,

皆可為未來公共行政研究、認識論與教育哲學提供基礎(Marini,

1971: 350)。惟在價值引進的同時,是否應放棄實證性的研究?

關於此者,誠如Frederickson 所言,新公共行政既非反實證論,亦

非反科學主義,而係主張應用其科學與分析的技術,以助益於分

析、實驗、和評估各種的政策和執行方式。因此, Frederickson 將

之形容為「第二代的行為主義」( the second-generation

behavioralism)。換言之,第二代行為主義與第一代行為主義不同

的是,它較少「一般」而較多「公共」;較少「描述」而較具「規

約」;較少「制度取向」而較具「顧客影響取向」;較少「中立」

而較具「規範」;而且希望作到「科學」一點也不能少(Frederickson,

1971: 315)。如同後來萬斯來(Gary Wamsley)所稱它是一種的「新

的社會科學」(new social science)。

3、適應環境的動盪不安

新公共行政學者咸認為,面對社會與日俱增的複雜性與互賴性,

未來環境的

動盪不安是可預期的挑戰。是故,行政理論與實務方面應修正過

去傳統的作法,

不是不重視環境,就是將環境視為單獨孤立的實體。其中,「面

對面境遇的行政」(confrontation administration)、參與式管理、

顧客導向組織、行政人員價值判斷、以及對動盪環境的容忍,是

不容忽視的行政課題,亦是可行的適應過程(Marini,1971:

Page 4: 主題:公共管理的基礎理論 - web.thu.edu.twweb.thu.edu.tw/lu.bk/www/news/99-1.pdf · 主題:公共管理的基礎理論 焦點一:試說明Weber對傳統官僚制度的反省。

350-351)。

4、發展新的組織型式

基於行政的相關性、面對面的境遇、和動盪不安的環境考量,傳

統的科層官僚制越來越無法迎合時代的需求,於是科哈特(Larry

Kirkhart)提出了協和式模式(consociated model)的組織型態以為

取代,並認為此種組織型態的結構特徵如下:(1)專案團隊是基

本的工作單元;(2)多元的權威結構;(3)所有組織均植基於

時間的迫切性(time imperatives):解決特定問題需在特定的時間

限制下為之;(4)以不同的次級(專案)計畫來處理相同的基本

問題;(5)社會關係是以高度的獨立自主和相互倚賴為特徵;(6)

顧客的需求得以在組織中表達;(7)組織講究短暫雇用,而非終

身職;(8)以電腦來保存文獻紀錄;(9)專業角色除注重技術

技能外,更要避免形成額外的社會階層(Kirkhart, 1971:160)。此

外,懷特(O. F. White, Jr.)亦認為面對社會變遷,學者亟應企圖

發展一套符合行政人員的調適技能,而主張以「愛的政治」(the

politics of love),或「面對面境遇」(confrontation)來適應變遷,

而且強調政治制度或行政人員須做到下列三點:(1)完全真實的

溝通是直接而自願的;(2)所有團體皆處於平等的地位,權威係

依功能性分配;(3)互動是以人員的專業能力和擬達成的目的為

基礎所形成的明顯的原則架構(an explicit framework of principles)

而持續地進行(White, Jr., 1971:59-83)。

5、建立受益者導向的組織

根據Frederickson 的看法,新公共行政的基本要旨,即在傳統公共

行政所要回答的二個問題:(1)如何在可資運用的資源下提供更

多或更好的服務(效率)?和(2)如何以減少支出來維持目前的

服務水平(經濟)?加上一個重要的課題,那就是「此一服務是

否在增進社會公正」?甚至其言:「公共行政如不能加以改變以

彌補少數族群的剝奪,終將會壓抑這些少數族群。」(Frederickson,

1971:311)。誠如歷史哲學家金恩(H. Zinn)所言:「窮人的哭訴

並不一定總是正確的,但要是聽不到他們的哭訴,你就永遠不知

道公平是什麼。」因此建立具社會正義的受益者導向之行政,是

刻不容緩的工作;也因此,行政人員應比過去更應強調受益者忠

Page 5: 主題:公共管理的基礎理論 - web.thu.edu.twweb.thu.edu.tw/lu.bk/www/news/99-1.pdf · 主題:公共管理的基礎理論 焦點一:試說明Weber對傳統官僚制度的反省。

誠和計畫忠誠(program loyalty)(Marini, 1971: 352)。

焦點三:公共管理的研究途徑

【思考方向】

根據波茲曼( Bang Bozeman )所著《公共管理:迄今最佳文輯》

(Public Management : The State of The Art) ( 1993)一書,他認為公

共管理的研究起始於 70 年代末,80 年代初,由兩個學派所組成:

(一)公共政策途徑(P 途徑)下的公共管理:1. 對傳統式的公共行政及政策執行研究予以拒絕。因為政策學派

中的公共管理學者大多是來自政治學或公共行政學等領域外

的學術背景,故均強調以自我原有的學科專長來研究公共管

理,並將公共行政或政策執行的研究成果予以排除。

2. 偏向於前瞻性的、規範性的理論研究。政策執行的研究焦點主

要在克服執行時所遭遇之問題,而公共管理有效地結合政策分

析,將重點放在方案結果的促成,並將政治性主管人員視為政

府績效關鍵人物。

3. 著眼於高階層管理者所制定之策略的研究。其認為高階層官員

的主要工作即在研擬策略,以利屬員朝其所定之目標提供最佳

的服務。

4. 透過個案研究來發展所需的知識。即從實務運作及管理者的親

身經歷中,萃取出最佳的管理原則。

5. 在政策學派影響下的公共管理研究,乃視公共管理與政策分析

為一體。但受企業管理學派影響的公共管理,則與政策分析幾

無交集。

(二)企業管理途徑(B 途徑)下的公共管理:一般而言,企業管理學派(B 途徑)取向下的公共管理與傳統

的公共行政較為接近,且倡導的學者亦多是受過企業管理或

一般管理課程的訓練。B 途徑下的公共管理研究,傾向以師

法企業的方法來提昇公部門的服務品質,晚近實務界所崛起

的「新公共管理主義」( New Public Managerialism)可為代表。

1. 偏好運用企業管理的原則。

2. 對公私部門間的差異不作嚴格的區分,並以經驗性的理論

Page 6: 主題:公共管理的基礎理論 - web.thu.edu.twweb.thu.edu.tw/lu.bk/www/news/99-1.pdf · 主題:公共管理的基礎理論 焦點一:試說明Weber對傳統官僚制度的反省。

發展作為公私組織間差異的解釋基礎。

3. 除了對策略管理及組織間的管理逐漸予以重視外,並強調

組織設計、人力資源、預算等方面的過程取向的研究,並

非如同政策途徑僅強調政策及政治的研究。

4. 以量化的實驗設計作為主要的研究方法,個案研究僅是教

學上的一項補充教材而已。

(三)比較 P 途徑的公共管理與 B 途徑的公共管理之差異:

1. 公共政策途徑的公共管理學者,一心一意想撇清與傳統公

共行政的關係;企業管理途徑的公共管理學者與公共行政

學者,特別是認同「公共行政即管理學」的學者,有較密

切的關係與相似的理念。

2. 公共政策途徑的公共管理學者,認為公私部門管理有別;企業管理途徑的公共管理學者不認為公私部門的管理有

什麼太大的差異,或者說他們沒有公私部門有所差異之先

入為主的觀念,而主張以較嚴謹的態度發展實證理論來描

述與解釋公私部門之異同,在此前提下,他們偏好、認同

並接受企業管理概念與工具,並將之引介運用於公部門的

管理之上。

3. 公共政策途徑的公共管理學者幾乎完全仰賴個案研究為

獲取知識均方法;企業管理途徑的公共管理學者兼採個案

研究與量化研究做為教學研究的方法。

4. 公共政策途徑的公共管理學者擅長於務實個案的編纂與

羅列;企業管理途徑的公共管理學者比較重視學術的研究

與理論的建構,擅長於從不同的學術領域中汲取有用的方

法、技術與理論。

5. 公共政策途徑的公共管理學者以高階管理者,特別是政治

性任命人員,為主要教學研究對象;企業管理途徑的公共

管理學者以具備文官資格的公共管理者為對象。

焦點四:公共選擇理論之意涵

【思考方向】

(一)以經濟學為途徑從事非市場決策的公共選擇理論,便基於

Page 7: 主題:公共管理的基礎理論 - web.thu.edu.twweb.thu.edu.tw/lu.bk/www/news/99-1.pdf · 主題:公共管理的基礎理論 焦點一:試說明Weber對傳統官僚制度的反省。

個人自由和效率的理由,極力倡導選擇的極大化

(maximization of choice)之觀念,鼓吹在公共事務的決策和

運作方面,應讓民眾個人擁有最大的選擇權。

(二)公共選擇理論乃是基於經濟學的基本假定:認為人是一個

理性、自私、追求效用極大化的行動者,而此亦為新公共

管理論述的基礎。

(三)該學派有五個主要特徵:1. 調效率與理性的型模。

2. 注重公共財(即政府最終的角色扮演,因為政府對公共財負有

最大的管理責任)。

3. 方法論上的個人主義(Methodological individualism) :該學派假

定經濟人的本質乃是理性的人,而理性人以擴大其效用為主要

目的。

4. 強調法律與秩序:公共選擇理論在規範上偏向於個人自由,而

個人自由的保障有賴於限制強制性的集體行動 (limitingcoercive collective action),使集體行動的成本降低,而個人自

由卻能有效率的達成。簡單的說,即是強調分權和不同政府層

級小規模的服務系統。

5. 以市場規則作為決策規則:凡法規的制定或市場不足以應付外

部性的問題時,均靠政府的運作。

(四)公共選擇學派對官僚的批評與建議主張:2. 公共選擇學派的學者相信,官僚的行動主要在增加自己的權

力、聲望、以及安全,從而運用層級節制的結構所得到的收穫

並非在於完成組織目標,而是在成就個人目的。

3. 因此,行政學者歐斯洞(Vicent Ostrom)根據前述公共選擇理論

對於自利理性人的基本假定,強調運用大眾參與、多元的決策

中心、分割的(fragmented)和重疊的管轄權等機制,箝制政府

的權力,俾以建構他所謂的民主行政之典範(The Democratic

Administrative Paradigm).4. 是以,歐斯洞亦從此一角度指陳官僚體制缺乏效率,他認為官

僚體制在效率方面不如市場機制,原因在於:(1) 官僚體制回應各種分歧的需求時顯得雜亂無章、囫圇吞

Page 8: 主題:公共管理的基礎理論 - web.thu.edu.twweb.thu.edu.tw/lu.bk/www/news/99-1.pdf · 主題:公共管理的基礎理論 焦點一:試說明Weber對傳統官僚制度的反省。

棗。

(2) 社會成本不斷升高。

(3) 無法拿捏回應需求的適當比例。

(4) 坐視公共財日益耗損,因為官僚無能採取行動防杜逐漸增

加的對立性現象。

(5) 官僚體制所採取公共行動與其宣稱所欲達成公共目的完全

無關,甚至背道而馳的傾向日益嚴重。

(6) 結果.,補救措施不但未能減輕問題,反而使問題更為惡

化。

5. 到了 1980 年,此種以市場理論為基礎的公共選擇理論,在精

簡政府的規模和管制範圍方面,獲得了實踐。此使得 1980 年

代以主流行政論述的內涵,從大有為政府走向小而美政府的觀

念,其所採取公共管理作為,主軸環繞在於如何透過組織精簡

的方式,減少政府支出以舒緩財政上的壓力。

(五)理論與實務的落差:

直得特別注意的是,歐斯洞以公共選擇理論為基礎所提倡的民主

行政典範,其所蘊含的基本理念乃是 1980 年代美、英執政者的實

際作為所未能及者。歐斯洞根據公共選擇理論的自利人性假定,

試圖透過權力分割的手段,以及擴大民眾參與公共決策的途徑,

避免政府部門及其人員權力專擅,提倡小而美均政府乃是為了確

保民主。但盱衡雷根總統和佘契爾首相的行政改革作為,卻僅是

基於現實的考量,為解決政府財政窘境,不得不進行政府組織的

精簡,此一小而美政府的發展趨勢並非為了追求民主行政。

焦點四:公共管理的興起背景?

【答題方向】

公共管理的出現,源於二次大戰後、後工業時代來臨之際,當時

理論界與實務界均普遍感受到建構在韋伯式系絡下的公共行政已

經出現「合法性危機」,在偏狹的理性主義下,明顯發生知識無法

指引行動,理論缺乏解釋力及無法忠實體現人民需求等現象。

(一)理論層面;公共管理的出現,是一種隨對公共行政舊典範進行挑戰的「後官

Page 9: 主題:公共管理的基礎理論 - web.thu.edu.twweb.thu.edu.tw/lu.bk/www/news/99-1.pdf · 主題:公共管理的基礎理論 焦點一:試說明Weber對傳統官僚制度的反省。

僚典範」而來;企圖跳脫傳統官僚典範陷入於層級節制的嚴密控

制、狹隘的效率觀、空泛的行政執行程序(以法辦事)和抽象的公

共利益等問題,而依循後官僚典範強調預見性、策略性、結果趨

向、主管領導、市場取向、消費者導向及企業性等方向發展。

(二)實務層面:1980 年代以來的新公共管理(New Public Management, NPM)運動

浪潮風行全球,並被冠以不同的稱號,如管理主義,以市場為基

礎的公共行政,企業型政府,甚至具有政治理念色彩的名稱,如

新右派(指英國的政府革新).新治理(指美國的政府革新)。這股新

公共管理運動實與各國的行政改革有密切的關聯性。意即新公共

管理是在回應後資本主義社曾(又稱為後工業主義)時代,政府所

必須迎擊的挑戰,包括:

1. 刪減公共支出及抑制政府組織等作法的成效相當有限。

2. 公、私部門的界限日益模糊。

3. 資訊科技一日千里。

焦點五:新公共管理的要點。

【思考方向】

對新公共管理研究頗負盛名的胡德(C. Hood),指出新公共管理

的要點為:(1)授權公部門的管理者逕行「臨場的」專業管理,

以便進行組織主動的、透明的、裁量的控制,達成確定責任動線

的課責精神;

(2)重視績效的明確標準與衡量:透過訂定清楚的目標與績效標

準來正視目標達成的效率實現;

(3)注重產出的控制:依據績效的衡量作好資源的分配,著重成

果取向,而非程序的遵守;

(4)轉移公部門產品由分支單位負責:將大型組織分割成公司化

的單位,俾讓各單位有獨立的預算與職責基礎,並透過組織內部

和外部的特許制度(franchise arrangement)來獲致競爭優勢;

(5)引發公部門的更多競爭:透過雇用制度、投標程序、和其他

相互競爭的制度來達成降低成本和提高服務的關鍵;

(6)強調管理實務的「私有化型態」和「彈性的雇用和報酬」:

Page 10: 主題:公共管理的基礎理論 - web.thu.edu.twweb.thu.edu.tw/lu.bk/www/news/99-1.pdf · 主題:公共管理的基礎理論 焦點一:試說明Weber對傳統官僚制度的反省。

將私部門多年來實施有效的管理工具應用在公部門之中;

(7)講究資源使用的更多戒律和簡便:透過縮減直接成本、提高

工人戒規、嚴拒工會要求、減低順從成本等方式來查核公部門的

資源要求,並以較少成本提供較多服務。

他並將新公共管理的核心信念描述如下:將焦點放在管理而非政

策,以及績效評價與效率上;將公共官僚結構分解為許多附屬機

關(agencies),各機關之間根據使用者付費的基礎進行互動;利

用準市場(quasi-market)和簽約外包(contracting out)的方式來

塑造競爭環境;削減成本;以及一種強調產出目標、限期合約、

金錢誘因以及自由空間的管理型態。

焦點六:公共管理未來的發展趨勢

【思考方向】

(一)持續充實公共管理的概念內涵:公共管理此一學科領域的主要工作是為研究政府部門的革新

( Reform). 變革 ( Change ). 再造 ( Reinventing ) 創新

( Innovation ) 等。就現階段而言,其概念內涵暫時不宜作嚴格或

具體的界定。

(二)公共管理所採取的研究途徑是兼具實務性與規範性:公共管理的發展趨勢是為建構一個新而成熟的學科領域,因此其

所 1 取的研究途徑必須兼具實務性與規範性,既不落入狹隘的技

術理生窠臼,也不陷入空泛的玄學迷障,在發展具有實際效用的

治理工具的同時,也不忘建立其核心的價值規範。

(三)公共管理並不排斥私部門所使用的方法:公共管理學者認為,公部門與私部門兩者所使用方法,在實際選

擇使用時是講究「效用主義」的,兩者間的關係是互補而非完全

替代。

(四)重視虛擬網絡組織對公共部門所造成的影響:此一現象勢必對公部門的工作環境造成莫大的衝擊,因此公部門

應及早規劃適當的管理方法,例如電子公文認證的問題、資訊管

理或管制的問題及電子化政府的監督規制範圍。

(五)公共管理強調創新與系統思考:

Page 11: 主題:公共管理的基礎理論 - web.thu.edu.twweb.thu.edu.tw/lu.bk/www/news/99-1.pdf · 主題:公共管理的基礎理論 焦點一:試說明Weber對傳統官僚制度的反省。

公部門的管理者唯有具備創新與系統思考能力,才能夠提出正確

的變革方向,同時兼具整合者的技術能力,才能夠設計出新的治

理系統。俾利創新的政策產生實際功效。

(六)公共管理重視人力資源管理:自 1980 年代以來,先進工業化國家為因應全球市場化的競爭壓力

及愈來愈嚴格的服務品質要求,紛紛採取策略性的人力資源管

理,試圖藉由提昇公共部門的人力素質與工作力來提振國家競爭

優勢。其主要的變革方向,是將傳統的「人事管理」(PersonnelManagement) 轉換為強調分權化、彈性化及具有市場競爭機制的

「人力資源管理」( Human Resource Management)系統。

(七)重視服務的品質與績效的評估方法:在「顧客至上」的年代,政府施政必須講求服務品質的持續改善

和不斷的超越,而績效是民眾在接受政府所提供的服務時,首先

須考慮的重要面向。因此,公共部門管理者必須主動發展客觀且

有效的績效評估方法,以利藉由顧客滿意度來檢測政府績效,

(八)重視政府再造的方法與過程:政府再造(Reinventing Government)已成為一種「時代精神」,惟其

施行的方法或策略並無定律,重點在於必須配合公共部門的特有

生態環境,並且在過程中必須實事求是,不能淪為政治祭儀。

(九)公共管理強調價值調和與課責:公共管理強調價值的調和,不僅重視效率系統與效能,同時也重

視公平、正義和民主。再者,公共管理在主張行政自主化和彈性

化的同時,也強調課責(Accountability)的重要性;論者認為,唯有

發展客觀有效的績效測量標準,對政府部門的課責方能落實。

(十)公共管理重視運用組織發展的干預策略:1. 與人相關過程的干預(Human Process Interventions),包括人

際、團體及組織等三個過程途徑,希望藉由第三造干預、團隊

建立、領導及建構願景等管理策略進行干預。

2. 技術結構的干預,例如組織員額精簡、再造工程、員工投入及

工作設計等干預策略。

3. 人力資源管理的干預,例如績效管理、開發並協助組織成員發

揮潛力等的干預策略。

Page 12: 主題:公共管理的基礎理論 - web.thu.edu.twweb.thu.edu.tw/lu.bk/www/news/99-1.pdf · 主題:公共管理的基礎理論 焦點一:試說明Weber對傳統官僚制度的反省。

4. 以及策略性干預,主要是發展因應環境變遷和增強組織轉換變

革能力的干預策略。

焦點七:公共管理者(公經理人)應該扮演何種角色?具備何

種動機與技能?

【答題方向】

(一)角色:

米茲柏格(Mintzberg)在 1973 年發表《管理工作的性質》一書在該

書中將管理者所扮演者歸納為十種角色,並區分為三大類:

1. 人際角色:(1) 頭臉人物(figurehead) :為了象徵性意義,在法律上或形式上代表組織從事特定活動,如

會見賓客、主持宴會、代表簽約、剪綵、致詞等,以確立對內之

法定權威及加強對外之網路連結。

(2) 領導者(leader) :從事聘用、指揮、激勵、教導員工等活動,以期對內建立團隊。

(3) 連絡人(liaison) :從事組織內外個人或團體的接觸與聯繫,以便對內建立團隊,對

外連結網路,有助於獲取必要的資訊與協助。

2. 資訊角色:

資訊是重要的權力來源,資訊角色扮演得當,將使管理者成為所

有資訊的收發中樞,有助管理權力的提升與揮舞。

(1) 監理者(monitor) :

負責從組織內外獲取與蒐集資訊,特別是組織外部環境訊息萬

變,蘊藏著許多機會,也潛伏著許多威脅,這些資訊的獲取,攸

關組織的生存發展是管理者不可輕忽的工作。

(2) 傳播者(disseminator) :

從事傳播遞送資訊給組織內成員的活動,擁有良好暢通的溝通網

路,適時適量的資訊傳遞,是有效管理的基礎。

(3) 發言人(spokesperson) :

代表組織對外傳播遞送資訊,在面對外界利害關係人乃至於一般

大眾時,肩負著爭取善意、消弭疑慮的重責大任,以期塑造組織

Page 13: 主題:公共管理的基礎理論 - web.thu.edu.twweb.thu.edu.tw/lu.bk/www/news/99-1.pdf · 主題:公共管理的基礎理論 焦點一:試說明Weber對傳統官僚制度的反省。

正面形象。

3. 決策角色:

可說是管理者最重要的責任。

(1) 企業家(entrepreneur) :

從事發掘機會、化解危機、因應環境變遷、促進組織變革等作為,

以期為組織創造永續發展、長期發展的生機。

(2) 干擾處理者(disturbance handler) :

負責處理各種有損組織績效或組織發展的干擾,針對突發問題,

尋求矯正之道。

(3) 資源配置者(resources allocator) :

任何組織資源都是有限的,因此,如何配置組織的人力、財力、

物力、時間、資訊等資源,以有效地達成組織目標,是管理者一

項十分重要的工作。

(4) 談判者(negotiator) :

代表組織與相關人士,進行交涉、溝通夫、協調、議價、談判等

活動,為組織爭取利益。

(二)動機:

依貝利與威斯(Perry & Wise, 1990)的分類:

1. 理性動機指追求個人效用最大的動機。

2. 道德規範動機如為民服務,盡忠職守,社會公平等。

3. 感性動機: 如來自堅信計畫對社曾重要性的認同感,愛國心或

慈愛心等。

(三)技能:

凱茲(Katz)於 1974 年《哈佛企業評論》發表「有效的管理者所應

具備的技能」一文,提出三種管理者必備的技能,包括:(1) 技術性技能:管理者必須對其管轄的業務,有一定程度的瞭解與處理能力; 缺

乏此項技能,易被譏為「外行領導內行」,為基層管理者所應具備

者。

(2) 人際間技能:管理者如何能與上下左右間建立愉悅有效的溝通、協調、互動模

式,建立起信任與合作的人際關係之能力,晚近流行的「情緒商

Page 14: 主題:公共管理的基礎理論 - web.thu.edu.twweb.thu.edu.tw/lu.bk/www/news/99-1.pdf · 主題:公共管理的基礎理論 焦點一:試說明Weber對傳統官僚制度的反省。

數」(EQ)可說是此項技能的一項衡量指標。

(3) 概念化技能:係指具備宏觀視野,能從事形而上、抽象化、與策略性思維的能

力,又稱為直觀能力;此項技能為高階管理者所應具備之技能,缺

乏者易被批評「見樹不見林」。

焦點八:公共管理的三大問題(限制)?

【答題方向】

杜克大學班恩(Behn)教授則指出,倘若公共管理想要成為一門科

學,必須仔細認真地思考以下三大問題(3M 問題) 。

(一)微觀管理的問題(The Micromanagement)即公經理人如何突破憲政體制及程序規則等微觀管理的循環。這

是一個實際存在的問題,是一種如何治理的問題,例如公經理人

必須能夠協助立法者、政治首長及永業化的文官等,釐清政策制

定初執行過程中的權責歸屬。又如公經理人也必須能夠清楚界定

公共管理在引進企業精神後,其合法性及倫理的問題。

(二)激勵的問題(The Motivation Question)即公經理人如何激勵組織成員盡心盡力來達成公共目的。在委託

一代理的觀點下( Principal-agent Version),國會如何來控制行政首

長民選官員又如何來控制文官,以及公經理人如何激勵公職人員。

(三)測量的問題(The Measurement) :即公經理人要能夠測量出他們自己及機關的績效。包括輸入

(Input)、產出(Output)、結果(Outcome)及影響(Impact)等的測量。

同時,公經理人也要能夠協助公民瞭解如何能適當且實際地測量

出政府的績效,並據此確立本身的責任。

焦點九:公私部門在管理層面上的差異性?

【答題方向】(世新余致力老師版本,2000)(一)底線(bottomline) :

主張公私管理差異者,認為公共管理者很少有一清楚的底線。民

間企業經理則有利潤、市場績效或企業的存活為底線。

(二)時間水平:

Page 15: 主題:公共管理的基礎理論 - web.thu.edu.twweb.thu.edu.tw/lu.bk/www/news/99-1.pdf · 主題:公共管理的基礎理論 焦點一:試說明Weber對傳統官僚制度的反省。

主張公私差異者,認為公共部門管理者面對政治需求與政治時效

性,經常是只有相對較短的時間可做政策制定與改變;相對而言,

私部門經理人顯然有較長時間去做市場開發、技術創新投資與組

織重建等。

(三)人事任免:在政府中有兩層的官員,一為事務官,另一為政務官,事務官受

人事法規的保障,無論考試、任用、升遷、考績、解雇、退休等

均有詳細的法令規章,與一定的作業程序,公都門管理者很難從

效率的角度,善用人力資源。反之,私部門經理人對於人事的任

用、調任與免職皆有很大的權限,可對人力資源作較佳的利用。。

(四)分權與負責:主張公私差異者,認為公部門基於憲法分權與制衡的設計,公事

門組織的權力與責任是分散的,結果是任何一個公共政策的推

動,公共管理者均需不斷的與其他機關進行溝通協調。相對的,

私人企業則較無需花時間資源去做外部的談判。

(五)決策過程:主張公私管理差異者,認為私部門經理人在經過專注的研究後做

決定性的決策;相對的,公部門的管理者則是不斷的在「做」、「不

做決定」或重新修正,而且是處於草率的分散的狀況下: 公部門

管理者可能面對較多急迫性危機問題,暴露在來自各方利益的干

擾,因此其決策經常是無法如私部門經理人一樣,可以在有計晝

的時程表下進行。

(六)公開性與封閉性:主張公私管理差異者,認為政府管理必須攤在公共的目光監督

下,因此較為公開;相對的,私部門企業管理皆是在內部進行,不

需經過大眾的審視,因此較為封閉。

焦點十:請說明企業型政府的內涵以以及「5C策略」。

【思考方向】

一、歐斯朋(D. Osborne)與蓋伯勒(T. Gaebler)在一九九二年出

版的《新政府運動:如何將企業精神轉換至公務部門》(Reinventing

Page 16: 主題:公共管理的基礎理論 - web.thu.edu.twweb.thu.edu.tw/lu.bk/www/news/99-1.pdf · 主題:公共管理的基礎理論 焦點一:試說明Weber對傳統官僚制度的反省。

Government: How the Entrepreneurial Spirit Is Transforming thePublic Sector)一書中,提出企業型政府如何運作或治理的十項原則,俾能將官僚體系改變為富有創新精神的行政機構:

(一)導航式的政府:政府的職能在於引導領航(steering),而

非親自操槳。

(二)社區性的政府:政府將更多的決策權和公共服務的提供,

回歸社區自主處理。

(三)競爭性的政府:競爭機制是紓解官僚體制運作失靈的良方,

政府應將競爭的觀念注入公共服務與產出之中,以取代傳統獨佔

而造成的保守、浪費與無效率。

(四)分權式的政府:政府應將決策權下授以增加員工的自主權,

同時在適當的監督下,充分分權,讓地方政府發揮因地制宜的功

能。

(五)前瞻性的政府:政府能夠以遠見來治理國家,並重視事先

的防範優於事後的彌補。

(六)任務導向的政府:政府應以目標和任務為導向,而非以法

規命令為驅力,並注重任務的優先次序以便集中精力有效運用資

源。

(七)結果導向的政府:政府應對其施政結果負責,並以此作為

績效評量的標準。

(八)顧客導向的政府:政府的服務要以滿足顧客(人民)的需

求為優先,政府的施政績效和品質應由顧客(人民)的滿意度決

定。

(九)企業導向的政府:政府除了節流外,更要注重開源。

(十)市場導向的政府:政府面對不同的公共問題,可透過市場

機能的自律調理,以舒緩政府機構官僚化的現象。

二、歐斯本(T. Osborne)與傅瑞祺(P. Plastrik)從各國政府再造經驗中

歸納出五種再造策略一簡稱「五C策略」(FiveCs)之內涵,以作為

改變政府部門的參考。

1.核心策略(The Core Strategy) :

系指公共組織的職能集中在領航,而非操槳;換言之,行政體系應

重視正確的政策之制定,與施政方針之設定。

Page 17: 主題:公共管理的基礎理論 - web.thu.edu.twweb.thu.edu.tw/lu.bk/www/news/99-1.pdf · 主題:公共管理的基礎理論 焦點一:試說明Weber對傳統官僚制度的反省。

2.結果(後效)策略(The Consequence Strategy) :

後效策略的目的是建立在公平、客觀及科學的績效酬賞制度,以

利獎優懲劣,其若能發揮功能,是政府再造的策略中最具影響力

的策略。

3.顧客策略(The Customer Strategy) :

政府再造的精髓所在就是顧客導向,主要是在調整行政人員與民

眾互動的方式,以顧客導向的方式處理行政業務,強調公共組織

對顧客負責, .藉以提高行政體系對外在環境的敏感度以及回應

力。

4.控制策略(The Control Strategy) :

所謂控制策略指涉行政組織將內部重要性的決策權,逐級下授,

必要時可以對社區授能。

5.文化策略(The Culture Strategy) :

文化的策略是五項策略中最隱晦難明的策略,然而也是政府再造

策略中能否將成果持續的重要支點。所謂文化策略,是指三項途

徑之運用:

(1) 改變行政人員所習慣的工作內容及方法:即以顧客立場改變工

作習慣。

(2) 管理行政人員的情緒與壓力,發展新的心理契約:即對顧客應

有的感情承諾。

(3) 型塑贏家心態,發展必勝的心理模式:即強化員工自我控制,

邁向必勝目標。

焦點十一:黑堡宣言對企業型政府的看法。

【思考方向】

(一)面對時下崇拜市場機能、主張小而美政府、提倡以企業為

師的風潮,黑堡學者相當不以為然。首先,渠等認為資本主義與

市場經濟雖是追求社會福祉的必要條件,但卻不是充分條件,只

有市場、只靠市場是不夠的。資本主義給了社會航行的動力卻沒

有航行的設備、技術與方向,必須要靠政府代表集體社會意識,

掌握公共權威,來提供航行所需的設備、技術與方向。簡言之,

沒有市場萬萬不能,但市場絕對不是萬能,政府勢必要介入。黑

Page 18: 主題:公共管理的基礎理論 - web.thu.edu.twweb.thu.edu.tw/lu.bk/www/news/99-1.pdf · 主題:公共管理的基礎理論 焦點一:試說明Weber對傳統官僚制度的反省。

堡學者進一步申論,我們必須要有一個全新的行政體系才有辦法

面對各種政治與經濟上的問題,這是最起碼的要求。因此,如果

美國的公共對話一直還是侷限於討論政府在資本主義市場中的角

色,或老是研究如何減少政府介入的話,美國很難維持其在世界

上的優勢競爭力,亦無法不斷提高國民的生活品質。

(二)其次,黑堡學者認為公共行政迥異於企業管理,因為:(1)

常任文官不像企業經理人與對手們在競爭追求市場與利潤,常任

文官是與政治過程中的其他參與者競逐轄區、正當性與資源;(2)

公部門行政人員必須互動的對象,如市民與利益團體等,不同於

私部門的消費者與供應商,因為這些市民與利益團體對行政體系

的效能各有特殊的認知與期待;(3)行政體系運作所需的技能,

關注的焦點、以及工作的特質與私部門有很大的差異。宣言進一

步指出這些差距很大,因此, 在私部門很成功的管理者到了公部

門未必能有同樣的成就,反之亦然,故師法企業絕非解決公部門

諸多問題的良方。換個角度而言,公共行政與企業管理不同,乃

在於公共行政不但要能在較複雜的政治環境中有能力進行管理的

工作與展現管理的能力,更要有能力維持機關觀點、公共利益,

以及合於憲法規範的治理過程,這是公共行政與企業管理最大的

差異。

(三)何謂機關觀點?黑堡學者認為行政體系不是一塊思想言行

一致的巨石。公部門為了分工必須從事部門劃分,部門劃分後自

然會產生許多不同的立場與觀點,而各種立場與觀點都應有其表

達的權利與正當性。就像多元民主政治過程中充滿各個利益團體

競賽角力一般,行政體系充斥著各種不同的機關觀點相互競爭辯

論。而各機關可說是一個倉庫,儲存著專業知識,長年經驗、恆

久智慧及對部份公共利益的共識,機關內的人員則是這些知識、

經驗、智慧與共識的受托者。誠然,機關觀點可能淪為本位主義

或偏狹主義,但機關觀點不同於本位主義或偏狹主義,因為行政

人員除了要表達機關觀點外,更要注意到公共利益。

焦點十二:Osborne 與 Plastrik 認為政府再造存在哪些迷思?

Page 19: 主題:公共管理的基礎理論 - web.thu.edu.twweb.thu.edu.tw/lu.bk/www/news/99-1.pdf · 主題:公共管理的基礎理論 焦點一:試說明Weber對傳統官僚制度的反省。

(政府再造並非「萬靈丹」)

【答題方向】

(一)自由主義論者的迷思:係指改善政府績效之道,在於擴張預算支出及增加業務項目。事

實上,對設計不當的制度而言,無論投注多少預算均無法產出顯

有優良的成果,

(二)保守主義論者的迷思:係指改善政府績效之道,在於緊縮預算支出及減少業務項目。事

實上,對設計不良的制度而言,減少支出或可撙節稅收,但無法

提高政府績效。

(三)市場機能論者的迷思:

係指改善政府績效之道,在於師法企業的經營技術。事實上,企

業的喻象與管理技術固有助於政府的績效,但政府部門與企業組

織之間存在許多重大差異。

(四)員工取向的迷思:係指公務人員的績效,取決於財務資源的多寡。事實上,公務人

員若不改變使用資源的方式,則很難獲得較佳的績效成果。

(五)人力資源論者的迷思:

係指改善政府績效之道,在於甄補優秀人才。事實上,政府績效

不彰的原因並非人才不足,而是掣肘人才的不良制度。

焦點十三:新公共服務理論的內涵。

【思考方向】

新公共服務是根植於民主公民權(democratic citizenship)、社區與

公民社會模型(models of community and civil society)、組織人本

主義與新公共行政(organizational humanism and the new public

administration )、以及後現代公共行政( postmodern public

administration)等觀點所建構的。新公共服務強調民眾回應性以及

公民參與的過程,回歸(美國)政府設立的初衷而非僅在於追求

效率、生產力等理性的表現,強調憲政價值以及對人性問題的探

討,由人性觀點反思社會核心問題,真正地瞭解並回應公民需求,

以符合民主理論對政府的期待,其基本理念如下

Page 20: 主題:公共管理的基礎理論 - web.thu.edu.twweb.thu.edu.tw/lu.bk/www/news/99-1.pdf · 主題:公共管理的基礎理論 焦點一:試說明Weber對傳統官僚制度的反省。

1、服務於公民,而不是服務於顧客:公共利益是就分享價值進行

對話的結果,而不是個人自身利益的聚集。因此,公務員不是要

僅僅關注顧客的需求,而是要著重關注於公民並且在公民之間建

立信任和合作關係。

2、追求公共利益:公共行政官員必須促進建立一種集體的、共同

的公共利益觀念。這個目標不在於找到由個人選擇所驅動的快速

解決方案,而是要創立共同的利益和分享的責任。

3、重視公民權益勝過重視企業家精神:致力於為社會作出有益貢

獻的公務員和公民要比具有企業家精神的管理者能夠更好地促進

公共利益,因為後一種管理者的行為似乎表明公共資金就是他們

自己的財產。

4、思考要具有戰略性,行動要具有民主性:滿足公共需求的政策

和方案可以通過集體努力和合作過程得到最有效並且最負責地實

施。

5、承認責任並不簡單:公務員應該關注的不僅僅是市場;他們還

應該關注法令和憲法、社區價值觀、政治規範、職業標準以及公

民利益。

6、服務,而不是掌舵:對於公務員來說,越來越重要的是要利用

基於價值的共同領導來幫助公民明確表達和滿足他們的共同利益

求,而不是試圖控制或掌控社會新的發展方向。

7、重視人,而不只是重視生產率:如果公共組織及其所參與其中

的網絡,能夠基於對所有人的尊重而通過合作和共同領導之過程

來運作的話,那麼從長遠來看,它們就更有可能獲取成功。

焦點十四:在晚近的行政改革理論中,丹哈特夫婦(Robert B.Denhardt & Janet V. Denhardt)試圖建構新公共服務(NewPublic Service)來與新公共管理(New Public Management)加以對照比較。請敘述此二理論的核心要旨。

【思考方向】

自1990 年代開始,隨著時代的轉變,傳統公共行政理論所建立起

的科層官僚組織,對於當代複雜多變的問題捉襟見肘,再加上政

府資源的短缺致使官僚僵化的問題一一浮現。因此學者休斯

Page 21: 主題:公共管理的基礎理論 - web.thu.edu.twweb.thu.edu.tw/lu.bk/www/news/99-1.pdf · 主題:公共管理的基礎理論 焦點一:試說明Weber對傳統官僚制度的反省。

(Owen E. Hughes)認為自1990 年代之後,一項新的公部門管理

模式已出現在大部份先進國家中。這個新模式有不同的化身,包

括管理主義(managerialism)、「新公共管理」(new publicmanagement)、市場導向的公共行政( market-based publicadministration )、「後官僚典範(post-bureaucratic paradigm)、

或「企業型政府」(entrepreneurial government)。儘管此種新泰

勒主義目前仍然蔚為風潮,惟自2003年丹哈特夫婦出版《新公共

服務:服務,而不是掌舵》(The New Public Service:Serving, notSteering)一書而倡導「新公共服務」之後,,反而敦促行政學界認真細緻並且批判性地思考什麼是公共服務,為什麼公共服務很

重要,以及引導我們的行為內容和行為方式應該

是何種的價值觀。我們不僅要讚頌公共服務的特色、重要性和意

義,而且還要考

慮如何才可能更好地實踐這些理想和價值。

(一)新公共管理理論的核心要旨:

在新公共管理看來,科層官僚組織所提供的服務總是品質低劣,

同時亦無法符合消費者的需求或期望。基於新右派「私人就是好

的;公共就是壞的」哲學,不僅指出國家官僚組織應依比例地減

少,若無法如此,私部門的管理技術就該引進。進言之,,此理

論乃是為因應過去經濟不景氣、財政惡化、和正當性危機等險惡

環境,企圖克服雙環困境(catch-22 situation),提出解決良方,

希望釋放官僚,治理工作更具彈性、創新、與回應,並導入民間

活力,運用授能手段和非營利組織等來執行服務傳輸功能,以使

政府不必事必躬親,而擺脫萬能政府困境,實現「小而能,小而

美」的自由市場和新保守主義的理想。

(二)新公共服務理論的核心要旨:

不同於新公共管理主張驅逐科層官僚制與空洞化國家,丹哈特夫

婦指出構成新公共服務理論核心與實質有兩個主題:(1)促進公

共服務的尊嚴與價值;(2)重新肯定民主、公民權和公共利益的

價值觀,作為公共行政的卓越價值觀。甚至,他們希望像「民主」、

「公民」和「自豪」這樣的語詞不僅在我們的言談中而且在我們

的行為中都要比像「市場」、「競爭」和「顧客」這樣的語詞更

Page 22: 主題:公共管理的基礎理論 - web.thu.edu.twweb.thu.edu.tw/lu.bk/www/news/99-1.pdf · 主題:公共管理的基礎理論 焦點一:試說明Weber對傳統官僚制度的反省。

加流行。畢竟,公務人員並不提供顧客服務,而是提供民主。

簡單來說,新公共服務是根植於民主公民權( democraticcitizenship)、社區與

公民社會模型(models of community and civil society)、組織人

本主義與新公共行政(organizational humanism and the new publicadministration)、以及後現代公共行政( postmodern publicadministration)等觀點所建構的。新公共服務強調民眾回應性以

及公民參與的過程,回歸(美國)政府設立的初衷而非僅在於追

求效率、生產力等理性的表現,強調憲政價值以及對人性問題的

探討,由人性觀點反思社會核心問題,真正地瞭解並回應公民需

求,以符合民主理論對政府的期待,其基本理念如下:

1、服務於公民,而不是服務於顧客:公共利益是就分享價值進行

對話的結果,而不是個人自身利益的聚集。因此,公務員不是要

僅僅關注顧客的需求,而是要著重關注於公民並且在公民之間建

立信任和合作關係。

2、追求公共利益:公共行政官員必須促進建立一種集體的、共同

的公共利益

觀念。這個目標不在於找到由個人選擇所驅動的快速解決方案,

而是要創立共同的利益和分享的責任。

3、重視公民權益勝過重視企業家精神:致力於為社會作出有益貢

獻的公務員

和公民要比具有企業家精神的管理者能夠更好地促進公共利益,

因為後一種管理者的行為似乎表明公共資金就是他們自己的財

產。

4、思考要具有戰略性,行動要具有民主性:滿足公共需求的政策

和方案可以通過集體努力和合作過程得到最有效並且最負責地實

施。

5、承認責任並不簡單:公務員應該關注的不僅僅是市場;他們還

應該關注法令和憲法、社區價值觀、政治規範、職業標準以及公

民利益。

6、服務,而不是掌舵:對於公務員來說,越來越重要的是要利用

基於價值的

Page 23: 主題:公共管理的基礎理論 - web.thu.edu.twweb.thu.edu.tw/lu.bk/www/news/99-1.pdf · 主題:公共管理的基礎理論 焦點一:試說明Weber對傳統官僚制度的反省。

共同領導來幫助公民明確表達和滿足他們的共同利益求,而不是

試圖控制或掌控社會新的發展方向。

7、重視人,而不只是重視生產率:如果公共組織及其所參與其中

的網絡,能夠基於對所有人的尊重而通過合作和共同領導之過程

來運作的話,那麼從長遠來看,它們就更有可能獲取成功。

焦點十五:公共利益的指涉與功能

【思考方向】

(一)公共利益的指涉:

關於公共行政之公共對象究所何指的問題,傅德瑞克森(H. GeorgeFrederickson)認為雖無一致性理論,但有各種見解和看法,傅德

瑞克森將之歸納成以下五種模式:

1. 多元團體模式:社會上有許多利益相同的個人組成利益團體,競向政府施以

遊說及壓力,以謀自我利益的取得,增強及維護,「眾多團體

互動的淨值」就是公共利益。

2. 公共選擇模式:所謂的公共對象是指政府功能的消費者(Consumer),政府的

職責是在提供一個安全秩序的穩定環境好讓個人(消費者)依其功利理性,自由地追求計算自我利益。

3. 代議政治模式:公共對象是指民選首長及民意代表,他們因是民選,故最具

資格代表民眾。行政人員必須執行國會通過的法律,服從命

令,行使裁量行為時亦須唯首長是瞻。

4. 服務顧客模式:公共對象是指得到第一線行政人員服務或管制的個人及團

體,例如學校兒童是教育機關、校長、教師的服務顧客

(Client),犯罪者及受害者是檢警、監所人員的服務顧客(前者

一服刑;後者一保護),有所得者是稅務人員的服務顧客,心

理或生理障礙者是醫衛人員的服務顧客。

5. 公民資格模式:二共對象是指具有知識又肯參加公共事務的公民,這些「良

Page 24: 主題:公共管理的基礎理論 - web.thu.edu.twweb.thu.edu.tw/lu.bk/www/news/99-1.pdf · 主題:公共管理的基礎理論 焦點一:試說明Weber對傳統官僚制度的反省。

民」不僅關心個人利益,也會關心公共利益。行政機關應廣

開大門,透過多樣方法(社區集會、資訊公開、民意訪問等)去發掘、教育、擴大兼具功能與意願的公民,協助他們積極

參與公共管理。

(二)公共利益的功能:

公共利益在政策上可以發揮的功能,行政學者顧塞爾(C. Goodsell )歸納出下列四項:

1. 凝聚功能(unifying) :公共利益為符號,透過公共利益符號的運用,使公共政策「提出

的一方」與「接受的另一方」都能認可。

2. 合法化功能(legitimating) :政治體系透過公共利益的訴求與整合,能夠堅定公民的信心。對

既定的政策規劃,透過公共利益的訴求,可以整合、均衡各方利

益,並說服多數的公民,支持該項政策。

3. 授權功能(delegating) :因為公共利益缺乏準確、明定的範圍,內涵亦眾說紛紜、莫衷一

是。然而,公共利益的概念,卻能提供行政機構在法律或國會授

權的情形下,擴張權限,甚至更為精緻的運用。

4. 代議功能(representing) :在公共利益的概念下,提醒公民、公僕於政治運作的場合中,有

許多弱勢團體仍結構不良、組織鬆散,甚或並未選出他們的代言

人在國會中爭取利益。政策的制定,應以公共利益為依歸,才能

兼顧弱勢族群的需要。

焦點十六:何謂「治理」(Governance)?

【思考方向】

治理的意涵為何?對Robert O. Keohane & Joseph Nye 而言,治理

意指引導和限制團體集體性活動的過程和制度,其中包含了正式

和非正式的部分。相較之下,政府則是一種權威性的行動,並創

造了正式的職責。A. W. Rhodes 則對治理提出了七種內涵:(1)

作為公司治理的治理(governance as corporate governance);(2)

作 為 新 公 共 管 理 的 治 理 ( governance as the new public

Page 25: 主題:公共管理的基礎理論 - web.thu.edu.twweb.thu.edu.tw/lu.bk/www/news/99-1.pdf · 主題:公共管理的基礎理論 焦點一:試說明Weber對傳統官僚制度的反省。

management);(3)作為善治的治理(governance as good

governance);(4)作為國際互賴的治理(governance as international

interdependence);(5)作為社會控制論系統的治理(governance as

asocio-cybernetic system);(6)作為新政治經濟的治理(governance

as the new political economy);(7)作為網絡的治理(governance as

networks)。在敘述治理的七種意涵之後,Rhodes 並將治理簡要

定義為:「自我組成的組織間網絡」,而歸納其特性如下:

1、組織間的互賴。治理要比政府來的廣泛,它包含了非國家的行

為者。國家界線的變遷意味著公、私和自願部門間的界線之轉移

與滲透。

2、網絡成員的不斷互動。此乃導源於他們需要去交易資源和協商

共同的目的。

3、如遊戲般的互動(game-like interactions)。它係植基於信賴,

並以網絡參與者所協商和同意的遊戲規則為規範。

4、享有不受國家控制的高度自主性。網絡並不向國家負責,它們

是自我組合

的。唯國家雖不佔據主權位置,但它可以直接地和不完整地引導

網絡。

焦點十七:網絡治理。

【思考方向】一、起源

近年備受矚目的「治理」(governance)概念,被學界刻意與

「政府」區隔開來,一般意指政府與人民(所組成的組織或團體)

協同統治或管理的一種新的過程和型式。雖然治理的型式包括政

府官僚、市場等比較傳統的類型,但這個字彙的廣泛受到重視,

乃在於其隱含的「網絡」性質,意指政府與公民社會能夠統合成

為一個具有管理功能的網絡關係結構,透過這個結構的分工合

作,能夠有效促進法令計畫的周全性與執行的順遂;而此間非政

府組織和公民的參與,則對於提振政府能力以及限制政治和行政

的專斷獨行,有相當重要的貢獻。至於正式的定義則相當分歧,

其中聯合國「全球治理委員會」( Commission on Global

Page 26: 主題:公共管理的基礎理論 - web.thu.edu.twweb.thu.edu.tw/lu.bk/www/news/99-1.pdf · 主題:公共管理的基礎理論 焦點一:試說明Weber對傳統官僚制度的反省。

Governance)的說法具有相當的周延性:治理乃是各種公共與私

人機構管理共同事務諸多方式的總稱,它調和各種利益的衝突並

持續採取集體行動的過程;它包括正式機構與典章制度,也包含

經由成員同意並符合個人利益的非正式制度性安排。而英國學者

Rhodes則認為「治理」具有「自我組織(self-organizing)的意義,

是一種經由成員合作,建立彼此共同遵守的行為規範以達管理之

目的。

然而,「治理」一詞,基本上又可分自傳統意義與現代兩種

意義。依照傳統的定義,治理具有一種以國家為中心(state-centric)的思考模式,亦即國家經由政治機制的中介,引領(steering)社

會、經濟部門的統治作為。現代意義的「治理」則將社會相關制

度或機制(例如私部門之企業、非營利或志願性組織、社區公民

團體等)融入統治的過程,轉變為一種傾向以社會為重心

(society-centered),成員經由彼此互動、協商達成共識,進而

形成具有自治功能的網絡夥伴關係模式。Guy Peters稱上述治理的

傳統意義為「舊的治理」模式(old governance),現代意涵為「新

治理」模式。

二、原理:

目前公共行政研究上討論最多的公共治理模式,當推傳統「官

僚制度」、「新公共管理」理論以及稍晚發展之「(政策)網絡」

(policy networks)三者。就新制度論的觀點而言,政 府的存在

乃源於市場失靈;而就組織經濟學而言層級體系(hierarchy)的

優勢,則在於其能夠有效控制交易的不確定性,進而減低交易成

本。不論從那個觀點,政府以官僚體系提供公共財貨與服務,在

過去近百年之中一直有其正當性。然而,隨著全球化趨勢日益明

顯,且伴隨著各先進國家人口老化而來的社會福利負擔快速膨

脹,導致以往全能政府的概念受到莫大的挑戰。緣此,行政革新

運動,重新審視與界定政府的角色,並以創新的思維發展應對的

策略。新政府運動為主軸的一系列改革遂在先進國家迅速開展。

新公共管理非常仰賴市場機制,其基本前提在於透過個體經

濟理性與個人追求利益極大化的動機,達成所謂「公共利益」的

Page 27: 主題:公共管理的基礎理論 - web.thu.edu.twweb.thu.edu.tw/lu.bk/www/news/99-1.pdf · 主題:公共管理的基礎理論 焦點一:試說明Weber對傳統官僚制度的反省。

目的。根據Hood的說法,新公共管理的主要淵源為管理主義與新

制度經濟,同時Rhodes也認為,英國的行政變革在1988年以前是

由管理主義所主導,1988年以後至今,則是由新制度經濟的理論

思維取而代之。在此理論引導下,世界各國持續推動所謂的「4D」

行政變革:組織精簡(downsizing)、降低財政負擔(defunding)、

權力下放(decentralizing)與權力移轉(devolution)。

新公共管理或許是治療傳統行政弊病的一帖猛藥,卻非萬靈

丹。許多學者批評,例如新公共管理理論的基本內涵(經濟理性

的基本假設與管理主義)過於強調「顧客」式的回應,忽略了民

主治理應注意的核心議題,如公民精神(citizenship)、積極參與

以及民眾與政府機關協力行動(collaboration/partnership)等;新

公共管理只是提供一系列相互衝突的處方,缺乏理論的一致性與

整合。另外,新公共管理理論導引出全球化政府改造的風潮,也

激發學者探討新興國家(例如東歐)與初列已開發國家之林的國

家(例如台灣),有關新公共管理是否可與民主化改革齊頭並進

的討論。

為了導正新公共管理的問題,「網絡治理」的模式遂被發展

出來。例如新近對於民營化與契約外包的批評與省思,皆指出公

共治理應包括複雜的網絡內涵,需要政府與民間企業或契約對象

協調合作,因此其參與者不僅涉及國家、區域與地方各級政府,

也包含其他政治-社會各種組織團體,例如眾多特殊利益取向的利

益團體、私人企業、非營利組織、以及社區公民團體等。這些紛

雜的非政府組織團體,各有其本身特定的利益,而且在價值與目

標上,彼此經常衝突。如何將這些行動者納入決策制訂與執行體

系之中,而不致於危害治理的效率,就是網絡治理的精義所在

換一個角度觀察,有些學者試圖自「網絡」途徑詮釋「治理」的

內涵,認為其與目前官僚、市場二種制度機制相比較,網絡能夠

提供一個更為貼切、寬廣以及具體顯現現代國家、政府與社會關

係的有效分析架構。例如明確指出「政策網絡是公共行政研究治

理概念的核心」。Kettl 認為網絡理論提供現代治理一個嶄新的認

Page 28: 主題:公共管理的基礎理論 - web.thu.edu.twweb.thu.edu.tw/lu.bk/www/news/99-1.pdf · 主題:公共管理的基礎理論 焦點一:試說明Weber對傳統官僚制度的反省。

識架構:(一)增進瞭解治理的內涵,(二)政府應該如何順利運

作的基礎,(三)政府與各種非政府組織團體如何合作有效執行公

共政策,以及(四)促進政治權利與代表性民主的連結。而 Salamon

更將「新治理」(new governance)界定為公共行政的新典範,強

調公共行政的焦點必須從官僚轉移到網絡:從公、私部門對立轉

移到公私協力,從命令控制轉移到協商與說服,從管理技能轉移

為授能。因此,自網絡的途徑而言,在相當程度上,當代民主治

理實質上已經轉化成為一種經由公私部門策略性的協力與合作、

資源共享與共同協商,擁有自我管理能力的政策網絡架構;政府

與其他非政府組織或團體基於信任,經由資源的相互依賴,不分

階層平等對待,共同合作,一起承擔工作與責任,累積「社會資

本」(吳瓊恩 2002; 陳恆鈞 2002),達成協同治理的效果。

焦點十八:企業型官僚理論。

【思考方向】

一、李文(Lewin)和辛格(Sanger)兩人曾提出企業型官僚創造新環境

以利成功的八大步驟,茲將其分述如下:

1.容忍錯誤:

在公部門中,鼓勵創新的有形報酬很少,加以專業的風險深植其

中,假若制度上欠缺支持積極進取的機制,官僚惰性(Bureaucratic

Sabotage)、組織僵化的情形勢將無法避免。

2.具有創造才能與對承擔風險的支持:

高層管理者必須找出官僚體制中抑制創造才能的結構特質,設法

塑造基本價值(Initial Value)和獎酬系統(Reward System)來改變組織

的誘因。

3.賦予執行者自由裁量權與達成績效的責任:

管理者必須給予執行者自由裁量權來完成其所執行的計畫,並保

證其能達成清楚、具體目標的責任。

4.重視分析和評估:

分析的主要任務,是在創新方案實施中加以衡量,以達成方案設

計的盡善盡美;評估的工作則是藉由反饋系統,持續不斷探索創新

Page 29: 主題:公共管理的基礎理論 - web.thu.edu.twweb.thu.edu.tw/lu.bk/www/news/99-1.pdf · 主題:公共管理的基礎理論 焦點一:試說明Weber對傳統官僚制度的反省。

方案的執行情形,遇有錯誤立即修正,以保持此一方案的適用性。

5.藉由新的組織結構來加強彈性:

扁平式(Flat) 、較少層級(Less Hierarchical)的組織,是較能適應變

遷的組織結構,這種彈性源於其能自由地以非傳統的方式來組織

人員、資源與關係。

1. 獎酬制度能彈性的被使用:

除了基本的物質報酬外,尚有聲望共享(Credit Sharing) 、人民支

持,更重要是建立行政人員的尊榮感(Self-esteem)。

2. 建立外在的擁護者:

許多創新方案是政府部門無法獨立完成,必須藉由公、私部門協

力來加以促成,因此,將社曾資源導入公部門中,成為創新方案

成功的關鍵。

3. 媒體:建立受歡迎的公共形象:

欲尋求外在擁護者對方案的支持,或至少降低反對的聲浪,就必

須為機關或方案創造正面的公共形象,因此行政人員必須瞭解如

何使: 用媒體向大眾進行行銷與溝通的技術。

二、企業型官僚的行為特徵:

企業型官僚係指富於企業精神的公職人員,企業型官僚是由目標

和使命所導引,懷抱理想,並且積極革新。學者李懷適(Lewis)曾

依胡佛(J. Hoover) 、李克河(H. Rickover)及牟雪書(R. Moses)等三

人,歸納出五項企業型官僚的共同行為特徵如下:

(一)建立符合公共利益的志業:

企業型領導者善用組織內部的資源及力量,實現符合公共利益的

個人目標,並非僅限於加冠晉爵而已。

(二)專精某些社會關注的領域:

企業型領導者善於應用傳播媒體、國曾聽證會,以及其他各種會

議,倡導其專精的領域,在「理」與「勢」方面,成為某些公共

政策的代言人。

(三)善用組織力量:

企業型領導者曾積極運用組織中的影響力,爭取成功,而非消極

地坐視文官體系成為腐敗無能的淵藪。

(四)營造社會的信賴:

Page 30: 主題:公共管理的基礎理論 - web.thu.edu.twweb.thu.edu.tw/lu.bk/www/news/99-1.pdf · 主題:公共管理的基礎理論 焦點一:試說明Weber對傳統官僚制度的反省。

企業型領導者善用民意市場,塑造「舍我其誰」的形象。企業型

領導者不但擅長凸顯問題的重要性,亦善用自己的知識及能力,

塑造專業形象,以及民眾對其解決問題的信心。

(五)設法擴展專屬的公共政策範圍:

這種擴張組織的作法,雖然會悖離代議政府的民主程序,但卻能

有效降低不確定性、增加專業的自主性。

焦點十九:二十一世紀公共管理新典範—全觀型治理【思考方向】

(一)前言:

英國學者培利希克斯(Perri 6)首倡全觀型政府(holisticgovernment)之概念,主張下一世代之政府應為全觀型政府、預

防性政府、改變文化的政府及結果取向的政府。他並針對二十一

世紀政府所處之政治、經濟、科技、環境,提出未來全觀型政府

應該採行的制度變革。全觀型政府理念為英國工黨布萊爾政府所

採用,並提出聯合型(joined-up)政府之各項改革措施。不論從

理論層面或實務層面來加以衡量,全觀型治理( holisticgovernance)的理論可望成為二十一世紀有關政府治理的大型理

論(grand theory),值得行政學者廣泛加以研究。

(二)當前對未來政府形態的研究:

1. 第一種類型是以OECD 的研究成果為代表(OECD, 2001),

具體指出未來政府所面對的各種環境變遷,包括社會更加多

元、複雜、與分裂,媒體和教育改變民眾對於政府服務的品質、

課責與信任的要求。然而,他們對於未來政府的型態與特性,

基本上則保持開放的態度,並沒有提出具體的看法。

2. 第二種類型是Guy Peters 所提出來政府未來的治理模式

(Peters,1997)。他所提出來的模式包括:市場式政府、參與

式國家、彈性化政府、以及解制型政府。四種治理的願景都含

有公共利益的概念,也就是提供締造一個良好政府的綜合性理

念。不過,這四種模式仍然無法解答未來的政府所可能共同具

有的運作精神和原則。這四種政府型態都在描述政府在運作時

應該採用何種組織理念,而不是在描述未來政府在整體上所呈

Page 31: 主題:公共管理的基礎理論 - web.thu.edu.twweb.thu.edu.tw/lu.bk/www/news/99-1.pdf · 主題:公共管理的基礎理論 焦點一:試說明Weber對傳統官僚制度的反省。

現的型態或特徵。

3. 第三種類型是主張未來政府將呈現的共同模式。其中最明顯得

到共識的是有關電子化政府的理論。其次則是類如全觀型政府

的理論。政府運作透過資訊科技而全面電子化或稱數位化,已

是必然的趨勢,而不再是一種理論的構想。各國政府的電子化

只有程度、層次的差異,而不再是有與無的區別。全觀型治理

的核心關鍵則是有鑑於全球政經情勢已大幅、快速變動,全球

化、區域化的風潮日趨滲入政府的治理機制,快速變遷已成為

常態,政府的治理理念也必須採用全新的觀點以求適應。全觀

型政府是使政府能觀照全體,變成預防性、整合性、改變文化、

結果取向的政府,而能跨越政府層級、部會功能分裂的差距,

提供人民更好的服務。

(三)「全觀型政府」的意涵:

1. 反省過去:

全觀型政府這個觀念的起源,基本上在挑戰從十九世紀以來各國

政府的基本結構原則。各國政府在部會層級的組織,長久以來都

是以功能作為區分的標準,也就是功能分化是分部化的基本原

則。希克斯認為,在功能性組織的原則之下,政府的運作造成以

下的結果

(6 Perri, 1997: 26):

(1)高成本。

(2)錯誤行為的集中化。

(3)對於改變行為的做法認識粗糙。

(4)思考眼光短淺。

(5)過於注重治療而欠缺預防。

(6)欠缺協調、將惡化的問題丟在一起。

(7)評鑑的對象錯誤。

(8)負責的對象錯誤。

2. 界定:

全觀型政府應該做到(6 Perri et al., 1999: 11-14):

(1)政策層次的整合(policy-level integration):

Page 32: 主題:公共管理的基礎理論 - web.thu.edu.twweb.thu.edu.tw/lu.bk/www/news/99-1.pdf · 主題:公共管理的基礎理論 焦點一:試說明Weber對傳統官僚制度的反省。

政府所有的機關單位在政策制定的階段,即應為政策之整合,對

特定之政策目標與結果皆有共識和認同感。

(2)中央政府勇於創新並擴大授權:

對政府整合的工作必須有創新的勇氣,要能不斷地嘗試;整合工

作不是中央單方面的事,中央須擴大授權,地方政府也要主動參

與。

(3)審慎運用整合型預算(integrated budgets):

整體性預算(pooled budgets)可以克服政府各機關單位間不合作

的問題;但此種預算對整合工作不見得必要或有效。

(4)負責監督者須重塑新價值:

有責任監督者包含監察機關與政治人物、管制人員、監察委員、

地方委員會等等。監察機關的職權應順應民意,為產生有效能的

政府服務而施加監督的壓力與聯合參與(joined-up)的風氣;政

治人物也應緊盯政府整合工作之進度。

(5)中央與地方政府皆應傳播新知:

中央與地方都應設立學習單位,以負責傳播有關政府整合的新知

識。

(6)政府應更新資訊科技系統:

應發展全觀型政府所須之資訊科技系統並注意到個人隱私的保

護。

(7)人事行政之改革:

包括全觀型政府所需之考銓、薪俸等配套機制的革新。

(8)政治人物要勇於處理整合工作之困境:

例如要克服在運用資訊系統時,可能與個人隱私權的保護發生衝

突;說服民眾接受全觀型政府的觀念、讓民眾參與全觀型政府之

設計;由下而上原則的貫徹將使中央面對地方、基層的大幅擴權,

屆時將如何調整等議題或困境。

(四)「全觀型治理」的概念架構:

1. 三種面向的整合:

全觀型治理所牽涉的整合,包含三種面向(6 Perri et al., 2002:29):

Page 33: 主題:公共管理的基礎理論 - web.thu.edu.twweb.thu.edu.tw/lu.bk/www/news/99-1.pdf · 主題:公共管理的基礎理論 焦點一:試說明Weber對傳統官僚制度的反省。

(1)治理層級之整合。如全球與國家層級的整合(例如WTO 規

範的制定與執行)、中央與地方機關的整合、全球層級內環保與

資訊保護組織的整合。

(2)治理功能之整合。機關內功能之整合,如行政院之各部會,

或功能性機關間之整合,如健保與社福功能之整合。

(3)公私部門之整合。公部門業務採取委外、民營化、去任務化、

行政法人化等做法,運用更多非營利組織與私部門接軌,而使公

私合夥關係產生漸層合作的關係。

2. 比較:

(1)全觀型治理和傳統官僚型模及新公共管理等行政學典範的最

大區別在於,它是以解決人民的生活問題為政府運作的核心,相

對的,前面兩種典範都是以解決政府的問題為核心。由於要解決

人民的生活問題,不但要靠政府各個部會個別的努力,更需要政

府各部會協同的努力,因此就必須要有政府的整合型運作。

(2)政府的整合型運作牽涉到三個面向,包括治理階層的整合、

任一政府的內部治理整合、和公私部門的整合。在治理階層的整

合方面,它必須同時整合由地方政府、中央政府、到國際性政府

組織(包括區域性國際組織、全球性國際組織、及區域性政府,

如歐洲聯盟)的各種組織、法令規定和運作程序。在任何一個政

府的內部治理整合方面,則必須要改善或打破傳統行政所運用的

功能性分部化原則,使政府提供業務必須依賴各個功能不同機關

單位的傳統,改變成為政府各機關單位主動協調而且密切合作的

服務機制。至於在公私部門的整合方面,不但需要在法律制度方

面提供更多的協助,更需要文官體制和行政作業體系進行全盤

性、彈性化的調整,以便適應千變萬化的公私部門合作之需要。

(3)基於以上三種整合面向的需求,全觀型治理的體系之下,政

府的運作將以治理的結果為核心,擴大授權給各個治理單位,以

整合性的預算來達成任務,這種全觀型治理的境界,幾乎就類似

於人體的各種生理系統能夠自動自發的處理各種維護生命存續所

必須的生理功能一般,是複雜無比的。

(五)「全觀型治理」的改革策略—對我國政府再造的啟示

Page 34: 主題:公共管理的基礎理論 - web.thu.edu.twweb.thu.edu.tw/lu.bk/www/news/99-1.pdf · 主題:公共管理的基礎理論 焦點一:試說明Weber對傳統官僚制度的反省。

為了要達到全觀型治理的境界,必須採用三種改革策略,形成全

觀型治理的基礎建設,而這三種改革策略是互補而必要的制度。

這三種策略就是:運用資訊科技而形成的線上治理模式、整合型

政府組織、和主動型文官體系。這三種策略分別代表要達到全觀

治理所需要的科技基礎、組織基礎、和人員基礎。

1. 運用資訊科技而達成「線上治理」模式

從全觀型治理的角度來看,當人民想要解決他們的生活問題,不

論是有關於教育、就業、經商或者是其他問題,難道他們不能夠

以已經突飛猛進的資訊科技和網際網路技術,從單一的政府網站

得到他們所需要的服務?難道他們不能夠一次就得到所有服務,

而不論這些服務是來自於多少個政府的層級或是政府的單位?從

世界各國金融業務的普遍整合,到我國目前已經進行的金融控股

公司和其轄下金融機構可以提供單一銀行帳戶的整合性服務(並

且都是電腦即時服務)加以觀察,則政府電子化未來改革的方向

其實是非常清楚的。長期而言,政府的電子化改革要在網路技術

的基礎上,進行三種類型的整合,包括:不同政府層級的整合、

不同機關單位的整合、和不同政府網站的整合。這三種類型的整

合最後將達到整成為一個單一政府入口網站的階段。目前世界各

國政府都在朝這個方向而努力,例如我國、美國、新加坡等國家

都已經設置了單一政府入口網站,所差別的僅只是單一政府入口

網站(single governmental portal)所能夠提供的政府服務達到何

種整合的程度而已。

2. 建立整合型政府組織

全觀型治理的組織結構與運作,在核心決策部會單位方面,其特

色包括:

(1)以功能性分工為基礎的部會結構。

(2)增加資訊長、人力資源長、及企劃部等跨部會整合單位或辦

公室,和原來的功能性分工部會結構形成同心圓的整合型組織,

以強化溝通效果。

(3)政府組織運作的核心係以功能性分工的部會運作為主體,但

是同時融入跨機關單位的定期及不定期整合機制。

(4)以預算管制和行政目標管理為核心,定期評鑑政府業務整合

Page 35: 主題:公共管理的基礎理論 - web.thu.edu.twweb.thu.edu.tw/lu.bk/www/news/99-1.pdf · 主題:公共管理的基礎理論 焦點一:試說明Weber對傳統官僚制度的反省。

之程度。

(5)以資訊科技的制度化設計,建立政府業務整合之系統。

(6)各機關之業務應和其他部會、其他政府、服務之民眾、非營

利組織、壓力團體、國際性政府和團體,進行有效的連結、諮詢、

和磋商。

以上各項全觀型治理的組織結構和運作與傳統行政組織之不同,

乃在於它不但要依法完成功能性分工的業務需求,更要從解決人

民生活事件的角度出發,將政府業務整合的核心觀念,變成為內

建的機制。功能性分工的部會如果要以建立電腦軟硬體架構的科

學方式,進行研擬分工、協調的各種可能解決方案,將行政流程

充分予以電腦化的連結,則以目前的電腦科技而言,已不成問題。

在政府服務成為完全網路化的操作環境下,所有行政業務的溝

通、執行,已經突破各級政府的時空限制。就政府而言,跨越國

際性政府與機構、中央政府、地方政府、各種公部門機構的網路

平台,已足以提供毫無接縫的連續性、整合性服務。

3. 建立主動型文官體

如何達成整合型組織的運作目標,事實上幾乎全賴互相信任的主

動型文官體系來推動政府的公共事務。從全觀型治理的角度來

看,為了要達成政府體系的完全整合,提供人民即時、無線連結

的有形與無形服務,不但文官體系的主管階層應該有以上各種新

型態的行為價值觀,所有行使公權力或提供公眾服務的一般行政

人員都應該具有同樣品質的倫理行為。在全觀型治理的環境下,

文官的專業義務即是來自於他們有責任扮演一個有道德的公民角

色。行政人員既然受人民之付託而行使其權力,就應該維持政體

的價值(regime values),並且應該和民眾保持密切的互動。文

官應該要儘量相信公民、賦予公民參與的機會,並做出正確的決

定(Hart, 1984)。Hart 認為行政人員既在信賴的基礎上執行公

務,主張行政人員應該努力扮演成為「有道德的企業家」(moralentrepreneur)。

(六)結語:

全觀型治理的名詞在國內文獻討論上尚屬首見。全觀型治理相對

於傳統行政和新公共管理而言,是一個重大的行政學典範轉移,

Page 36: 主題:公共管理的基礎理論 - web.thu.edu.twweb.thu.edu.tw/lu.bk/www/news/99-1.pdf · 主題:公共管理的基礎理論 焦點一:試說明Weber對傳統官僚制度的反省。

也對行政學形成新的挑戰。全觀型治理的境界必須建立在綿密而

精細的資訊體系、組織體系、和人事行政體系之上,對於目前的

絕大多數國家政府而言,仍屬相當遙遠的目標。因此,如何形成

推動全觀型治理的動力,也許是一個更為根本的問題。從民主政

治的本質和發展來看,推動全觀型治理的境界,仍然必需要依賴

一個主動積極的公民社會,持續而有力的進行政治參與監督和行

政參與監督,進而對於政府的財政預算充分發揮影響力,建立合

理的財政收支計畫。只有在掌握了合理的財政預算控管之下,才

能夠形成全觀型治理的基本要求。

焦點二十:治理與公民社會的聯結—善治

【思考方向】

(一)前言:

自從1989年世界銀行首次使用「治理危機」一詞後,治理一

詞便取代「管理」,成為當代公共行政的主流觀點。而所謂「善

治」是從「治理」而來,強調政府效能在國家與市場機能有所不

足之時,必需有一個治理機制以協調並彌補,以保障公民與社會

的利益。以善治的觀點而言,強調政府與非政府部門對於公共生

活的管理,藉由合作而極大化公共利益。

(二)「善治」的意涵:

1.善治(good governance)就任何國家或地區而言皆是影響經濟發展

的重要因素。然而,如何使政府施政能夠達到善治,卻是上世紀

九十年代之後才逐漸成為學者討論的焦點。例如世界銀行(World

Bank)於1989年的報告中即提及在發展中國家進行經濟社會發展

計畫的過程中面臨部分國家所出現的治理危機。按照世界銀行報

告的歸納,所謂治理危機可歸納為以下兩點:(1) 部分國家的公

共資金遭到其國內菁英團體(local elite groups)的濫用; (2) 國際援

助團體在推動援助計畫時對當地的政治運作缺乏認知。因此,善

治至此成為國際援助團體對於發展中國家所欲輸出的概念

2.善治的概念由治理而來。根據亞洲開發銀行對治理的定義(Asian

Development Bank, 2006),治理的四個要素包含下述諸項:

(1) 問責 (accountability):政府官員就公眾對政府施政所

Page 37: 主題:公共管理的基礎理論 - web.thu.edu.twweb.thu.edu.tw/lu.bk/www/news/99-1.pdf · 主題:公共管理的基礎理論 焦點一:試說明Weber對傳統官僚制度的反省。

提出的質疑需提出回覆以及負起施政的責任。

(2) 公民參與(participation):在國家發展歷程中,能夠具備

公民的參與。

(3) 施政的可預測性(predictability):國家需具備有利於發

展的法治環境。

(4) 施政透明度(transparency):政府為公眾充分提供有關

施政的資訊,而且能夠向公眾清晰說明法規以及決策過程

4. 是以,善治的定義在於政府對於公權力是否能夠妥適行使,能

夠依法行政,同時也需對公眾的質疑即時回應,建立責任政治,

以及政府與民間的最佳協力夥伴關係。因此,善治的要件除了政

府依法行政,建立問責之外,民眾的參與是不可或缺的。換言之,

具備公民參與的公民社會,是善治的基礎,沒有健全發展和成熟

的公民社會,不可能有真正的善治出現。

5. 若針對善治的治理結構做剖析,吾人可以發現善治的治理結構

是一個以政府為主導的多元治理結構。在這個結構之下,政府雖

然扮演主導的角色,但是並非唯一的主體,除了政府之外,非政

府組織、企業與社會公眾皆是治理主體不可或缺的一份子。在善

治的治理結構之下,各治理主體所具備的職能如下:

(1) 政府:在善治的治理結構之下,政府雖然處於主導地位,

但是主要仍是藉由營造健全的法治環境,創造公平合理的競

爭環境,以促進市場機制的充分發揮,並促進治理結構的和

諧運作。

(2) 非政府組織(Non-Government Organization; NGO):在善治

的治理結構中,非政府組織主要職能在於彌補政府和市場的

不足,提供公共財(public goods)以滿足特殊利益群體的需要,

同時非政府組織也可執行部分政府職能。

(3) 企業:企業包含公營與私營企業兩個重要類別。無論是

公營企業或是私營企業,除了生產商品服務外,有時也透過

承包、特許經營、政府補助以及志願服務等方式提供公共財。

因此,就企業社會責任(corporate social responsibility)而言,必

須承擔公共責任,包括遵守法令規定,保障民眾及所屬職工

的健康、安全及福祉。

Page 38: 主題:公共管理的基礎理論 - web.thu.edu.twweb.thu.edu.tw/lu.bk/www/news/99-1.pdf · 主題:公共管理的基礎理論 焦點一:試說明Weber對傳統官僚制度的反省。

(4) 社會公眾:善治是「還政於民」的過程,因此社會公眾

對政府的監督是達成善治的重要環節。社會公眾對於政府施

政的監督有利於減少行政成本,提升行政效率。

焦點二十一:何謂政府再造?我國推動組織再造的具體作法為

何?

【思考方向】

(一)政府再造(Reinventing Government,簡稱REGO)或企業型

政府(Entrepreneurial government):係指政府應用新的方法來處

置有限的資源,達到最大生產力的效率與效能。申言之,企業型

政府係以體制內改革的「新右派」主張為經,以「市場經濟」、

「企業精神」為緯,透過「標竿學習」(Benchmarking)的方式,萃

取成效卓著的案例經驗,借用成功的企業精神經營技術,重新建

構行政文化、公務人力、權責歸屬、獎懲制度以及目標任務等層

面,期能活化文官體系,大幅提高其效率效能,適應能力、革新

能力以及治理能力,促成行政組織徹底轉型,實現政府的公共目

的,有效擴增公共利益。

(二)具體作法:

1.「去任務化」:即「解除管制」(de-regulation),指此項業務

應自國家任務中予以排除。

2.地方化:即「業務下放」,將現行中央機關辦理的業務,改由

地方辦理。

3.法人化:指將仍屬國家任務而無自任實施必要的業務,設立公

法性質的法人(即行政法人,指國家及地方自治團體以外,由中

央目的事業主管機關,為執行特定公共任務,依法律設立具人事

及財務自主性之公法人) 。

4.委外辦理:指將業務委託民間辦理,並可分為「機關委外」及

「業務項目委外」

焦點二十二:政府再造的理論模式與內涵

【思考方向】

一、理論模式:

Page 39: 主題:公共管理的基礎理論 - web.thu.edu.twweb.thu.edu.tw/lu.bk/www/news/99-1.pdf · 主題:公共管理的基礎理論 焦點一:試說明Weber對傳統官僚制度的反省。

學者杜尼克(Melvin J.Dubnick),試圖從近年來一般國家實施行政

革新的途徑進行分析比較,以釐清當代政府再造的策略途徑。

(一)最低限度的國家論:1. 最低限度國家論係以公共選擇理論為主軸,主張國家

的職能應儘量縮小,轉由市場機制及企業經營的方式

來運作。這種主張,具有反制官僚化(anti-bureaucratic)的傾向。以雷根政府(Regan Administration)為例,政

府的職權及重要性即經由預算緊縮、集權政策、精簡

而反應迅捷的政治建制等方式,予以極小化。

2. 最低限度國家論者,其立場係為「反制行政體系」

(antigovernment),主張大幅度刪減行政體系的預算及

人事,迫使政府在有限的資源下,構思有效的運作方

法,進行必要的變革;易言之,最低限度國家論者的

主張,係基於對政府的不信任,是一種「強迫式的策

略選擇」(push strategic option)。(二)管理再造的政府論:1. 此論是行政革新理論中較缺乏完整學術基礎與理論

架構的途徑,其以實際成功的企業經驗,作為政府部

門行政革新的借鏡。這種轉借均過程,固然排除了政

府的「公共特質/展現更寬廣的思想空間,卻也是管

理再造論者最受批判之所在。

2. 管理再造的政府論者並不排拒對政府的仰賴,屬於傳

統的因勢利導方式,主張以政治的手段以及成功的典

型,重新為政府尋找定位,藉由政治情勢的運用,導

引行政革新的方向;易言之,管理再造政府論者的方

法,是一種「牽引式的策略選擇」(pull strategic option).(三)解除管制的政府論:1. 此論與最低限度國家論不同,解除管制政府論者認

為,過多的管制及約束,足以妨礙文官體系的正常運

作,了提昇政府的績效,凡是不合時宜或繁瑣的法規

章程,均應予以刪除、簡化。

2. 解除管制政府論者則倡議「法規鬆綁」俾利行政人員

Page 40: 主題:公共管理的基礎理論 - web.thu.edu.twweb.thu.edu.tw/lu.bk/www/news/99-1.pdf · 主題:公共管理的基礎理論 焦點一:試說明Weber對傳統官僚制度的反省。

掙脫官僚體制的嚴密法規籓籬,提高行政的彈性。解

除管制政府論者認為,如果解除束縛之後的行政體系

依然未見效率提昇,即表示政府應考慮某些業務項目

必須進行移轉;易言之,解除管制政府論者的觀點,

是一種「釋放式的策略選擇」 (release strategicoptions)。

二、政府再造的內涵:

對於政府再造(Reinventing Government, REGO)的意涵,僅以「範

疇排除」(或稱負面表列)方式說明如下:

(一)政府再造與政治制度的改變無顯著相關:

1980 年代迄今的政府再造方案,係以行政部門為中心,與政治制

度無顯著相關。

(二)政府再造並非調整組織圖表而已,而是藉由改變政府組織

的任務目標、俸給制度、責任歸屬、權力分配以及行政文

化,重構行政部門的運作形式與內容。

(三)政府再造並不全然侷限於減少公帑的浪費、浮報以及濫用:

政府再造的目標,係在重塑政府部門,成為「效率導向、

永續革新」的公共組織。

(四)政府再造亦非政府精簡的同義詞,因為政府部門的績效與

組織規模並無顯著相關: 政府再造對行政部門的作用而

言,係為「塑身」而非「瘦身」只在追求組織結構合理化,

而非齊頭式的機關精簡。

(五)政府再造並不等同於民營化: 政府再造中重要的改革動

力,係源自「競爭機制」或「顧客選擇」非僅止於「轉讓

政府的所有權至民間企業」而已。因為,據各國的經驗顯

示,民營化如果僅從狹窄的技術層面著手改進,往往會造

成由「政府獨占」變為「私人獨占」

(六)內政府再造不只是提高行政效率的工具而已: 現代民主社

會的公民,不只要求政府撙節施政成本,更要求政府提高

施政品質。

(七)政府再造當然也不只是「全面品質管理」( total quality

management) 或 「 企 業 流 程 再 造 」 (business process

Page 41: 主題:公共管理的基礎理論 - web.thu.edu.twweb.thu.edu.tw/lu.bk/www/news/99-1.pdf · 主題:公共管理的基礎理論 焦點一:試說明Weber對傳統官僚制度的反省。

re-engineering)而已: 行政主管充分瞭解政府部門的「公共

特質」的差異,對企業管理技術與方法做適當轉化運用,

才是政府再造成功的前提要件。

焦點二十三:政府的未來治理模式(新治理模式)

【思考方向】

學者彼得斯(G. Peters)觀察 1980 年代及 1990 年代各國行政革新或

政府再造的實況,認為當代政府的新治理典範已經浮現,並可歸

納為四種明顯模式或特質。

(一)市場式政府(Market Government) :

1. 傳統治理結構最大的問題在於官僚體制的獨占性致使機關規

模和預算出現極大化,文官體系缺乏足夠誘因以致效率低落,

以及缺乏施政成本壓力導致過度生產等。

2. 市場式政府的特徵包括:採取分權化的組織結構; 運用企業管

理技術,並將功績薪給制(merit pay)改為績效薪給制(pay forperformance) ;引進市場競爭的誘因結構,並創造內部市場化的

決策機制;其提供的公共利益在於降低施政成本。

(二)參與式政府(Participant Government) :

參與式政府要對抗的是傳統官僚典範的層級節制體系,其主要特

徵包括:力求扁平化的組織型態;在管理上採用全面品質管理和團

隊建立的策略;運用諮議及協商的決策方式;其所提供的公共利益

在於增進公職人員和民眾對政府施政的參與度。

(三)彈性化政府(Flexible Government) :

彈性化政府的特徵包括:運用「虛擬組織/亦即政府的運作應著重

於系統的層次更甚於組織結構的層次;在管理方面,採取工作團隊

式的管理;其決策方式採取實驗性強的應變決策;其所締造的公共

利益在於降低施政成本和增進組織成員的合作團結。

(四)解制式政府(Deregulated Government) :

解制式政府的基本假定是,倘若能夠排除政府運作上的過度管

制,將使其更具效率功能。該模式對組織型態並沒有特別偏好; 在

管理方面,強調賦予管理者更多的自主裁量權;在決策方面,企業

型官僚應擔負更多決策責任;其所提供的公共利益為增進政府部

Page 42: 主題:公共管理的基礎理論 - web.thu.edu.twweb.thu.edu.tw/lu.bk/www/news/99-1.pdf · 主題:公共管理的基礎理論 焦點一:試說明Weber對傳統官僚制度的反省。

門的創新能力和行動力。

焦點二十四:新公共管理的概念目前已為世界各國政府普遍採

用,但也產生爭議,試說明其特性及其相關爭議。

【思考方向】

一、企業精神政府的設計策略與執行工具

(一)1992 年Osborne & Gaebler(1992)之Reinventing Government一書,帶動了理論與實務界對企業精神(Entrepreneurial Spirit)政府的關注風潮。當Reinventing Government 被視同「新政府運

動」的世俗化概念同時,反而使Reinvention 的深層意涵受到了忽

視。按Osborne 與Plastrik(1997)為回應若干論者,認為ReinventingGovernment 缺乏理論的基礎,及實際操作策略的批評,而給予

Reinvention 更精緻的定義為:公共系統及組織基本性轉型,藉建

構組織效果、效率、適應性及能力的劇烈增強,以達到系統「創

新」之目標。而此種轉型必須經由系統及組織的目的(purpose)、

誘因(incentives)、責任(accountability)、權力結構(power structure)與文化(culture)等面向之變革而得以完成。

(二)結合J. B. Say 及J. A. Schumpeter 對於企業精神與企業精神

管理的觀點,發現Reinvention 與企業精神所關注的重心均是「創

新」。因此,Osborne與Plastrik 亦精簡的將Reinvention 定義為由

「企業精神體系」(entrepreneurial systems)取代官僚體系的過程

(Osborne & Plastrik, 1997)。筆者同意若干論者所主張,在現代

公共行政中,若謂追求企業精神政府「取代」(replacing)或「去

除」(banishing)官僚體制,可能均為過度的描述,其不但在真

實世界中難以落實,也與公共行政發展沿革及現況不符。是故,

企業精神政府其目的應在「減弱古典公共行政之官僚體制特性」,

融合古典公共行政與企業精神兩者之長,共塑當代公共行政之新

輪廓。

(三)Osborne 與Gaebler 在1992 年所提出的企業精神政府應包

含下列十項特色(1)催化型(Catalytic)(2)社區導向(3)競

爭性(4)任務導向(5)績效導向(6)顧客導向(7)開創進取

Page 43: 主題:公共管理的基礎理論 - web.thu.edu.twweb.thu.edu.tw/lu.bk/www/news/99-1.pdf · 主題:公共管理的基礎理論 焦點一:試說明Weber對傳統官僚制度的反省。

性(enterprising)(8)前瞻性(anticipatory)(9)分權式(10)市場導向。而在這些特色中,依序可歸納出企業精神政府之可能

策略選案如下:

1.催化型政府可藉契約外包、扺用券、特許制、及稅收誘因,來

達到「導

航」替代「操槳」的轉型;

2.社區導向政府可藉社區之財政支援及授能,達到社區主義

(Communitarianism),減少官僚控制目的;

3.競爭型政府應充分推動公共組織內系統及外系統之競爭機制,

提昇生產力的量及品質;

4.任務導向政府是工作及法規之強力簡化(人事、預算及採購),

在法律範疇內,允許公經理人(public managers)之活化操作達

成目的;

5.績效導向政府改變「遵守法規及預算底線」之「組織目標錯置」

(displacement of goals),為強調「績效」及「成果」的工作文

化;

6.顧客導向政府建立「服務需求調查」、「顧客服務標準作業程

序」(e.g.英國公民憲章civic charter),提昇服務品質;

7.開創進取型政府不但強調財政的「必須」項目之節流,同時要

求開源。藉如開創基金(enterprise funds),留成分紅制(sharedearnings)及創新基金(innovation fund),配合使用者付費(userfees)及影響受益費(impact fees)來解決財政困境;

8.前瞻性政府調整預算、會計、報償制度、以達延長決策者決策

時間幅度之目的;

9.分權式政府在扁平化的層級組織中,授能第一線工作者,直接

面對顧客,並鼓勵其自我決策負責。

10.市場導向政府儘量利用現有市場機制,如污染費、環境維護費

及稅制誘因,去替代原有行政管制機制,以解決公共問題。

(四)由上述企業精神策略的回顧,可以瞭解(一)競爭(二)

誘因(三)市場機制是共同策略要素,而這些要素,則有賴組織

成員「創新」的基本原動力,得以激發,經由規劃及設計新制度

而達成公共問題解決之目的。

Page 44: 主題:公共管理的基礎理論 - web.thu.edu.twweb.thu.edu.tw/lu.bk/www/news/99-1.pdf · 主題:公共管理的基礎理論 焦點一:試說明Weber對傳統官僚制度的反省。

(五)誠如多數企業精神政府研究者之共識所示,政府部門絕無

法、也不該企圖如公司私部門的理念運作。由於公私部門基本使

命不同,政府企業化一項誤謬概念。但是我們卻應該致力於將企

業精神引介至公共組織(公共服務執行機構)。公共組織所生產

之財貨,不但可以如公司組織般的提供給「外部客戶」(externalcustomers)──人民及企業,並與之交易。同時,亦應可以扮演政

府「內部供應者」(internal providers),提供公部門自身服務,

並 與 之 同 樣 交 易 。 如 此 從 事 「 開 創 進 取 性 管 理 」

(enterprisemanagement),是企業精神政府之重要執行觀念。而

「開創進取性管理」的成敗關鍵有四項:(1)允許公共組織的商

業行為規範鬆綁(2)建立利潤中心制的自負盈虧體系(3)降低

行政規範管制(人事及預算)(4)加付競爭機制。

(六)通常「開創進取性管理」的執行工具有:

1.公共組織的「公司化」(corporatization),強調投資利潤的回

報,並有非政府獨占的董事會及公共管理人,以為組織績效負責;

2.開創基金會(enterprise funds)由經營收入,而非稅金成立循環

周轉基金會、貿易基金會等公共組織,直接提供公共服務;

3.使用者受益費(user fees)的課收對象,包括公共組織外部的顧

客及其他之公共機構;

4.撤除「獨占」(monopoly states),強化公共組織內外部之競爭

機制。

「開創進取性管理」可充份發揮(1)顧客導向(2)永續改善(3)績效導向(4)前瞻長程規劃(5)降低改善績效的政治干預

(simplifies thepolitics of improving performance)等優勢,使企業

精神政府得以貫澈落實。

二、企業精神政府的限制:

公共行政的內涵劃分為政策規劃的政務部門及政策執行的事

務部門,而後者強調的是「管理主義」的性質,這也尤其是新公

共管理(NewPublic Management)所呼籲的研究定向。雖然負責

執行的公共組織,在當前政府再造的理念下,因公共選擇理論

(Public Choice)、代理人理論(principal-agent theory)及交易成

Page 45: 主題:公共管理的基礎理論 - web.thu.edu.twweb.thu.edu.tw/lu.bk/www/news/99-1.pdf · 主題:公共管理的基礎理論 焦點一:試說明Weber對傳統官僚制度的反省。

本理論(transaction cost theory)所支持,而有「代理機構」(agencies)建構的國際風潮,但企業精神政府的引介,應不只是公共管理部

份所專有,同時亦應該推介於政策規劃的領域層次。然而,對於

企業精神政府的核心價值,卻是討論其設計限制的主軸。中外學

者對於企業精神政府的限制,均提出有價值的省思(江岷欽 & 劉

坤億,1997;江岷欽,1996)。不過由於各自方法論典範價值不

同,故亦引起相當有價值的學術對話。

(一)Bellon & Goerl(1992)對於企業精神政府提出四類型的反

思:

(1)企業自主性(entrepreneurial autonomy)與民主責任性

(DemocraticAccountability)之衝突

(2)公共企業願景(Public Entrepreneurial Vision)與市民參與價

值(Citizen Participation)矛盾

( 3 )企業隱密性( entrepreneurialsecrecy )與民主開放性

(Democratic Openness)(4)企業性風險承擔(Entrepreneurial Risk Taking)與民主公共

財處置責任(Democratic Stewardship)。

(二)Bellon & Goerl 的論點,同時俱呈「民主體制」與「企業

精神政府」的雙面優點及限制。這項價值的衝突,正如長久以來

民主社會中,有關「民主」與效率的辯論,似是體制內先在的價

值矛盾。

(三)而L. de Leon 則由倫理觀點,認為企業精神政府,內在有

不可欲之價值內涵;其包括:(1)本位自我主義(egotism)(2)自利性(selfishness)(3)剛愎自用(waywardness)(4)宰制

傾向(domination)及(5)機會主義(opportunism)。在這樣的

特性下,企業精神政府可能不只是會導致官僚體制的空虛化

(hollowing)(Lan & Rosenbloom, 1992),更可能形成如deLeon所言之「無政府」(anarchic)的政策問題。

(四)提出對「企業精神政府」更激烈的批評者如Larry Terry(1993)指出三項限制,包括(1)公共企業家偏好內部之宰制

(dominate people inside andbeyond their organization)(2)公共

企業家崇尚變革的創新,然而許多的變革未必帶來組織的正向發

Page 46: 主題:公共管理的基礎理論 - web.thu.edu.twweb.thu.edu.tw/lu.bk/www/news/99-1.pdf · 主題:公共管理的基礎理論 焦點一:試說明Weber對傳統官僚制度的反省。

展(3)公共企業家有反傳統的傾向,輕視存在甚至超過幾世紀的

價值及信仰。L. Terry 甚至據此提出結論:公共企業精神給予公

共行政的問題,會遠超過其所解決問題的貢獻。

三、公共企業精神限制的解決:

雖然各方論者對於「企業精神政府」各有不同之批判焦點,

但亦發現到其各自關懷的共識。例如宰制的特性、自利的個人動

機、及政府開放性等。然而筆者發現由於基本方法論上認識論及

世界觀之不同,雖然各家論者有發現問題焦點的共識,但對於解

決結論的知識態度,卻是南轅北轍。

(一)Bellon & Goerl 提出一種調合的方法論觀點,其等主張一

種強調公共參與(public participation)的「市民關懷之企業精神」

(civic-regarding entrepreneurship),用以解決過熱的個人自利動

機,並且化解部份「民主」與「企業精神」的衝突。而L. deLeon亦嚐試提出「倫理性的企業精神」(ethical entrepreneurship)。

此種修正後之企業精神,分別藉由:

(1)官僚體制之允諾公共企業家失敗的可能性及「嚐試方案」

(pilot program)(2)專業社區的價值及信仰規範價值

(3)甚至於在無政府網絡(an anarchic network)下,慎思後果──也就是對於「未來陰影之考量」(the shadow ofthe future),而可

以彌補企業精神在倫理面上之缺憾。

(二)對於極端的批判主義者而言,調合式的知識建議是必須給

予更加嚴厲的批判。筆者以為Terry 在企業精神政府之知識對話

立場上,即有過度否定性思考(negative thinking)之嫌。L. Terry認為如Bellone & Goerl 等人所提出之調合式世界觀,如「市民關

懷之企業精神」實在是一隻「披著羊皮的狼」(a wolf in sheep’s clothing),所以實不足取。然而此種批判式的結論,也正是方法

論上,批判主義論者,在接受大量反思性價值讚許外,所受到最

大批判的世界觀。回顧人類歷史(包括公共行政的發展史),經

常存在有「精神」與「物質」、「民主」與「效率」、「傳統」

與「現代」辯證並存的時代,應該是較為常態,而主流思惟獨霸

的情況,反倒是人類發展異象。要的是當我們發現衝突的價值時,

如何融合適中,尋找建設性之史觀──如Bellone & Goerl 與

Page 47: 主題:公共管理的基礎理論 - web.thu.edu.twweb.thu.edu.tw/lu.bk/www/news/99-1.pdf · 主題:公共管理的基礎理論 焦點一:試說明Weber對傳統官僚制度的反省。

deLeon 的修正式企業精神的嚐試對於公共行政之貢獻,應是遠大

於只是發現問題的陳述及批判之價值。

焦點二十五:請說明「轉換型領導」的意義、研究重點以及達

成之策略?

【思考方向】

(一)意義:

轉換型領導(Transformational Leadership)是柏恩斯(J. Burns)在

1978年《領導》一書中所提出,係一種能夠結合組織成員共同需

求與願望的組織變革過程;透過領導的作用,建立起人員對組織

目標的共識與承諾,基於共識承諾,領導者創造人員信念和行為

轉換的有利條件。

(二)構成要素 :

1.個別化關懷:關心個人。

2.心靈鼓舞:動機的啟發與精神感召。

3.才智激發:建立能夠激發組織上下才智的互動創造過程。

4.理想化的影響力:上下之間是相互影響的關係。

(三)研究重點 :

1.肯定人員有自我實現需求。

2.領導者透過激發與引導成員,喚醒成員自發意識與自信心。

3.從價值面、人性面、文化面、社會面探討領導現象。所以,領

導不僅是管理技術,亦是哲學觀念的實踐。

4.領導者扮演「組織意義的創造者」、「組織凝聚的締造者」、

「組織不安的解決者」、「組織成功的舵手」等啟發性角色。

(四)轉換型領導與交易型領導比較 :

轉換型領導並不否認領導是一種交易的結果,但更希望透過領導

者的魅力與遠見,從精神、觀念和道德層面獲得部屬的信仰和認

同,激發人員超越交易的現實關係,共同追求人格成長,並有效

達成組織使命。

(五)達成策略 :(林鍾沂老師版本)

邊尼思(W. Bennis)與納魯司(B. Nanus)於1985年出版「領導者」

一書中,論述達成轉換型領導的四項策略如下:

Page 48: 主題:公共管理的基礎理論 - web.thu.edu.twweb.thu.edu.tw/lu.bk/www/news/99-1.pdf · 主題:公共管理的基礎理論 焦點一:試說明Weber對傳統官僚制度的反省。

1. 透過願景引起注意 :

轉換型領導者為選定組織的方向,必須先在心理上為組織描繪出

一個含意且可能達到的未來狀態,亦即「願景」。此一願景可如

夢境般含糊,也可能像一個組織目標或任務說明那樣詳盡。但最

重要的是,願景對於組織而言,代表的是一個實際、 可行、迷人

的未來,並就某種重要層次而言,代表比現狀更好的情況。

2. 透過溝通傳達意義 :

美麗的願景必須是源自於組織整體的需求,必須由全體成員所「共

享」或「共有」,必須成為組織內一個新社會架構的一部分。惟

轉換型領導者如為新願景凝聚向心力,其扮演社會建築師的方式

相當多,包括發布、修訂招募新人的方法與目標;訓練員工響應

新的組織價值觀;以及採用或修改足以彰顯並強化新願景的共同

符號。

3. 透過定位尋求信任

「信任感」是維繫部屬與領導者之間關係的情感黏著劑,是所有

組織的基本要素,亦是維持組織的潤滑劑。能夠被信任的領導者

一定能使別人認識自己,並使自己的立場明確。

4. 自我肯定,施展自己的才能

肯定領導者自我的能力,必須包括下列三項要件:(1)瞭解自己

的優點和彌補自己的缺點;(2)培養與發展自己優點的能力;(3)

區分自己的優缺點和組織所需的優缺點。此外,一位成功的領導

者必須對事件的結果採取正面和樂觀的看法,而非將心思花在避

免錯誤、防患未然上。

焦點十四:何謂課責(accountability) ?它和「責任」(responsibility)有何不同?並說明課責的運作模式。

【答題方向】

(一)課責指的是組織中的某個人必須因為其決策或行動而

接受責難或獎勵。從最低層次的公務員到最高階層的

官員,每一層級的成員皆有受監督者課責的義務,課

責是一種外在的判斷標準,亦即外在的計算或順從。

(二)責任的意義較為模糊,而且是透過層級結構的另一個

Page 49: 主題:公共管理的基礎理論 - web.thu.edu.twweb.thu.edu.tw/lu.bk/www/news/99-1.pdf · 主題:公共管理的基礎理論 焦點一:試說明Weber對傳統官僚制度的反省。

方向進行;也就是說,每個人均對下屬的行動負有責

任,一個部會首長對所屬部會成員的行動負有責任。

由於為了何事負責,或者負責的範圍多大等問題無法

精確表示,使得責任的意義較為模糊不清。

(三)課責的運作模式:

1. 組織課責(organizational accountability) :係指官僚體系界限中的責任範疇,一般而言,命令不明確與

資源不充分是行為錯誤的開始。

2. 立法課責(legislative accountability) :立法、司法的課責範疇,當然法律也會隨時而轉化。

3. 政治課責(political accountability) :在大多數的個案中,政治課責通常包括官僚課責,因為政治

人物必須對其選民的承諾負責,此亦會造成行政責任的泛政

治化

4. 專業課責(professional accountability) :依賴行政人員的專業為準則,當專業與公共利益相衝突時,

最後則回歸公共利益。

5. 道德課責(moral accountability) :係指道德的有機體,經由建立社會規範而形成課責的來源,

道德課責也會有違抗命令的困境發生。

焦點二十六:學者對於課責的分類可分為哪些?【答題方向】

Romzek 與 Ingraham 提供了一個架構去瞭解課責多面向的觀

點。他們認為有四種不同類型的課責關係,主要分類的基礎

在於外部或內部,以及假定較高或較低的個人自主性,可分

為層級組織(層級節制)、法律、政治以及專業這四種類型的

課責關係,而前兩種具有較少的自主性以及較嚴格的監督要

求。

1. 層級組織(層級節制)的課責關係:

依靠的是管理與組織的命令,包括規則與標準作業程序,為了達

到這些標準成員必須為他們的績效負責。這種類型強調命令可以

Page 50: 主題:公共管理的基礎理論 - web.thu.edu.twweb.thu.edu.tw/lu.bk/www/news/99-1.pdf · 主題:公共管理的基礎理論 焦點一:試說明Weber對傳統官僚制度的反省。

透過規則、標準作業程序、監督、與組織的指令去告訴成員要做

什麼,此奠基於對個人嚴密的監督,因此工作自主性較低。

2. 法律的課責關係:

強調一些外觀上所期待的服從,或是績效的標準與仔細審查及監

督,成員被課予對他們績效的責任。此外,法律的課責關係亦牽

涉到對授權績效的細密外部監督,像是法律與憲法架構,這包括

了會計稽核與監督的公聽會。

3. 政治的課責關係:

基於更高層次的自主性。行為被視為是一種細部的層次,而主要

的考量是授與員工追求所指派的工作。政治的課責關係主張對於

關鍵的外在利害關係者的期待要有責任,像民選的官員、顧客或

其他機構。這些類型強調課責是基於行政人員都能充分的回應機

構的顧客、或是「消費者」的期待。

4. 專業的課責關係:

強調的是對於專業知識的責任與服從,提供個人的高自主性,基

於對適當實務的內化準則作出安排。績效標準是由專業規範以及

在同等級或工作團體中的普遍實務所建立。行政課責需要外部控

制也需要專業主義。公共行政中的課責是以內部及外部的方法達

成。內部控制是在機關內實施與建立;外部控制則涉及立法監督,

預算及稽核活動,在辦公室之中設置行政監察史(ombudsman)、新聞評論、顧客團體、利益團體及其他相關個體的監督。

焦點二十七:試比較「傳統模型的官僚課責」與「管理主義模

型的課責」?

【答題方向】

(一)傳統模型的官僚課責:

傳統的公共行政有其官僚的課責形式,透過層級結構,公務員在

技術上均向政治領導者負責,而最終則是向人民負責。此外,該

模型認為形式上屬於政治人物本分的政策事務,以及留給公務員

執行的行政事務之間,可以進行嚴格的區分。這種政治與行政二

分的模型是易於瞭解且具有吸引力的。任何行政行為都必須以政

治領導者為依歸,而公務人員僅是一項工具,去實現由政治領導

Page 51: 主題:公共管理的基礎理論 - web.thu.edu.twweb.thu.edu.tw/lu.bk/www/news/99-1.pdf · 主題:公共管理的基礎理論 焦點一:試說明Weber對傳統官僚制度的反省。

者所提出來的政策命令。

(二)管理主義模型的課責:

1. 以顧客為中心:在管理主義的模型中,課責系統的另一項改變就是改善與顧客之

問的關係。在傳統的官僚課責模型中,唯一的課責途徑便是透過

政治領導者。如今則有直接的連結關係通到民眾身上,公共管埋

者試圖將管理與顧客之間的關係,視為平常責任的一部分;而在直

接課責的途徑中,機關本身必須負責處理與顧客之間的關係,並

改善對他們的服務。。

2. 課責管理:(1) 相對官僚課責而言,一個更實際的途徑,便是採取「課責

管理。其觀念和私部門的方式相似,公共經理人對自己的

行動,以及所屬機關的行動,均負有課責的義務。他們不

能推稱所有的行動都是政治課責,而否認他們自己的責

任。在此種新課責型態中,直接連結官僚體制、顧客、立

法機關、媒體等之間的關係,而非總是要透過政治人物為

之。這種直接課責提供了比過去更大的優勢,因此,

Holme&Shand 認為「若被課責的人能真正的為自己的決策

負責,則課責性便會加強」。

(2) 在採用課責管理時有三個層面需要注意。

課責的改善,可透過明確列舉出政府所有組織中的確實業務內

容,其對成果的達成與否,應該是相當透明化的。但在傳統模型

中,從來沒有任何實際的方法可確定成果是否已經達成,因此公

務員可以在政治或大眾的注目下,將令人不滿的表現隱藏起來。

課責的個人面,而有別於組織面。管理者是一個對目標的達成

負有個人責任的人。管理者是要負責任的,它不像傳統模式只讓

課責發生在最高階層,尤其當管理者沒有採取適當的預防行動

時,就必須要負起責任。

第三個課責的形式就是回溯性的課責。傳統的模型總是有一些

回溯性的機制,特別是在財務上的廉潔, Behn 認為必須要為績

效儘可能的建立回溯性的課責。因此,公共管理者必須被信任能

去達成結果,並為其所做的負起形式責任。

Page 52: 主題:公共管理的基礎理論 - web.thu.edu.twweb.thu.edu.tw/lu.bk/www/news/99-1.pdf · 主題:公共管理的基礎理論 焦點一:試說明Weber對傳統官僚制度的反省。

焦點二十八:管理主義模型的課責可能會碰到哪些問題?【答題方向】

(一)管理課責與民主之間的緊張關係:管理主義下強調創新的績效作為,以為課責的標的,然而此種作

為可能將產生創新與課責的兩難困局,因為創新就必須突破法規

限制,難以進行有效的課責,而緊守法規的課責性,又難以產生

創造性的作為,甚至延伸出貝隆與葛爾(C. J. Bellone & G. F.

Goerl)所謂之企業自主對張(versus)民主課責、「個人遠見對張公民參與、企業秘密對張民主公開、擔負風險對張監護公共財等等關

於創新與民主政治之間的緊張關係。

(二)行政人員課責的對象為公民而非顧客:

管理主義模式中將行政人員視為企業家,將人民視為顧客,使得

行政人員角色狹隘化,行政人員不應只照顧到以短期利益為優先

考量的顧客族群,而應將焦點放在廣大的民眾身上,以實踐公共

利益。

(三)忽略法律規範:

行政人員為人民主權受託者,因此在執行政策時必須受到議會與

司法等機制之監督,尤其在管理主義模式下公共服務交由市場機

制提供,更需要接受議會與司法的監督,透過法律、資訊公開等

方式,以確保行政人員為憲政秩序下的正當參與者。

(四)管理主義引用私部門課責概念衍生的問題:

管理主義的課責概念源自於私部門,而私部門(公司治理)課責的

理論基礎就是「委託人一代理人」(principal-agent)理論,在該理論

下由於委託人與代理人處在資訊不對稱的情形下,將產生「不逗

當的選擇」與「道德風險」的弊病,亦即管理者的運作往往是為

了自身的利益,而非股東的利益,但這種現象在公部門仿效引進

後: 使問題更為明顯。

(五)公部門提供之公共服務難以量化:

公共管理模式強調提升公部門的績效,然而公部門所提供的公共

服務難以量化,如何建立具備可信度與效度之具體績效指標,當

作課責行政人員的工具,評定行政人員之績效,其可行性仍有待

Page 53: 主題:公共管理的基礎理論 - web.thu.edu.twweb.thu.edu.tw/lu.bk/www/news/99-1.pdf · 主題:公共管理的基礎理論 焦點一:試說明Weber對傳統官僚制度的反省。

觀察。

焦點二十九:何謂「創造性的解構」?其與課責之間存在何種

緊張關係?又該如何克服?

【答題方向】

創新是企業精神最核心的概念,惟在創新精神的實踐過程中又涉

及創造性解構與風險承擔兩項重要又衝突的困境,如何調和便成

為企業型政府的重要課題。

1. 創造性解構:根據熊彼得的觀點,所謂創造性解構,是指企業家在競爭激烈的

市場中,以創新的觀念與作為,解構原有產品的生命週期,開發

新的產品與服務,以便取得競爭優勢,確保利潤;杜拉克繼承了熊

彼得的觀點,認為公部門或企業部門都需要具有創新與企業精

神,公部門與企業部門企業家的責任不僅在提高組織績效,更應

該合力建構一個創新型的社會。

2. 創新與課責:企業型政府往往會陷入一種兩難的困境,倘若服膺種種既定的管

理程序,恐怕就難以創新、突破困境;假如大膽從事創新工作,則

有違反規範之虞。是以,企業型官僚經常擺盪在創造願景或遵守

管制、有效的執行政策,或是縝密的政策分析、把事情做好或是

便宜行事等進退維谷的困境中。

3. 克服之道:

(1) 雷文與珊卓(Levin & Sanger)為克服這種兩難困境,主張將企

業型政府的內部管理階層區分為兩類 :一是方案執行職

( Program Executives ) ,另一為行政執行長職 ( ChiefExecutives )。前者專司於計畫方案層次,係指官僚企業家、主

管企業家(屬高級事務官) ;後者專司於決策層次,係指政治企

業家和部分主管企業家(屬政務官)。(2) 這兩種管理階層在創新與課責上各盡其職能,達到「權責相稱」

的效果,使方案執行時成員樂於發揮新的職能,而不必畏懼創

新作為與課責相牴觸,同時也有權選擇創新方案的行政執行長

Page 54: 主題:公共管理的基礎理論 - web.thu.edu.twweb.thu.edu.tw/lu.bk/www/news/99-1.pdf · 主題:公共管理的基礎理論 焦點一:試說明Weber對傳統官僚制度的反省。

職,承擔更多的風險責任;其次,方案執行職的人員所提的創

新方案必須與他人競爭,除非有創新解構能力,否則,無法雀

屏中選,此一過程促使組織內部形成市場競爭的機制,俾利於

企業型政府發揮新功能。

焦點三十:新公共服務對政府再造的啟示?

【答題方向】

(一)政府再造的改革趨勢,將符合「鐘擺理論」(Pendulum Theory)所闡述的內涵,亦即行政改革的理念,是在 NPA 與 NPM的兩端進行擺盪,而八○年代的行政改革確立了鐘擺是擺

向新公共管理此端的,但基於鐘擺理論,吾人不排除朝向

新公共行政的價值主張進行擺盪修正的可能性,但必須強

調的是此種趨勢是漸進修正而非取代,新公共服務觀點的

提出,只能說是朝著價值的另一端修正,後續發展仍待實

務界的檢證。

(二)NPS 是 NPA 與 NPM 的修正論,更是公共管理新的研究途

徑,即所謂的 C-approach (Citizenship Approach) -強調公民

資格的研究途徑。至於 NPS 在實務上能否取代 NPM,應

由市場去決定。而這個市場的遊戲規則,正由政府、人民(公民資格或顧客導向),甚至更廣義的參與者(如國際價值觀)選擇型塑著。

(三)從學術研究的角度來看, NPS 的提出正可以在 NPA 與

NPM 相對立的價值中增加對話的可能、出現瓶頸時的另類

思考與決策基礎;惟我們所要強調的是,瞭解本身的限制與

對立面的思考邏輯,才能在光譜的兩端之間做好策略的抉

擇,而 NPS 的倡議,其意義正說明了這個「市場」(實務

界)的趨勢。

※補充:行政院組織改造調整原則(宋餘俠,2009)

一、機關(單位)運作範圍及部門化規定

(一) 中央行政機關組織基準法規範

基準法除規範一般機關或單位的組織設計原則外,亦就行政院組織之設

Page 55: 主題:公共管理的基礎理論 - web.thu.edu.twweb.thu.edu.tw/lu.bk/www/news/99-1.pdf · 主題:公共管理的基礎理論 焦點一:試說明Weber對傳統官僚制度的反省。

計,即行政院所設「部」之運作範圍有其劃分原則,以基準法第29條規

定,各部主管事務以中央行政機關應負責之主要功能為主軸,由各部分

別擔任綜合性、統合性之政策業務;基本政策或功能相近之業務,應集

中由同一部擔任;相對立或制衡之業務,則應由不同部擔任;各部之政

策功能及權限,應儘量維持平衡。

(二) 行政院及所屬各級機關組織調整作業原則

若以行政院為單一組織個體而言,其運作範圍的確立即所謂政府職能功

能的再檢討,將非屬中央行政部門所應擔當之範圍予以剔除,此即為去

任務化、委外化、行政法人化及地方化策略之立論理由。亦即在釐清該

組織運作範圍及其界線後再進行組織設計作業,而四化後之業務即為所

謂的「機關保留」部分,此即為行政部門據以逐級而下設計的範疇。

二、相依關係之協調原則

機關或單位之業務職掌內容不論是以接續依賴關係、相互依賴關係或組

合依賴關係而設計於同一組織之內,或因成本考量而設計於不同組織

間,皆有同時設計協調機制之必要性。前文提及將所有有依賴關係的單

位納於同一部門是組織解決環境不確定性的方式,但若確有因協調成本

過高而暫時無法納於同一部門內時,則可採標準作業流程、臨時任務編

組、常設委員會或階層體系等方式加以處理。

三、控制幅度與階層體系

在組織釐清其運作範圍及部門化的過程中,亦即將業務職掌採水平分工

集合在一起時,組織亦在進行垂直的分工,即建立上下間的指揮關係,

這也是因為控制幅度不宜過廣,且為了降低協調成本而有相對應的組織

層級節制概念的產生,也是某種意義上的降低協調成本概念。以基準法

為例,該法第22條規定:「機關內部單位應依職能類同、業務均衡、權

責分明、管理經濟、整體配合及規模適中等原則設立或調整之。」,而

考量業務單位及輔助單位之差異性,亦在同法第26條規定:「輔助單位

依機關組織規模、性質及層級設立,必要時其業務得合併於同一單位辦

理。」,以上皆為機關內部單位因考量控制幅度而規範之共同事項與標

準。

四、組織結構的權變與法規限制

如何調整行政組織架構則必須因應環境因素與行政機關施政需求而

定,因此,就行政機關法制設計上便要賦予行政機關得以權變調整組織

架構的彈性。以目前基準法的法制設計而言,該法第4條及第11條規定

中央三級以上行政機關必須以法律定之,中央四級行政機關方得以行政

Page 56: 主題:公共管理的基礎理論 - web.thu.edu.twweb.thu.edu.tw/lu.bk/www/news/99-1.pdf · 主題:公共管理的基礎理論 焦點一:試說明Weber對傳統官僚制度的反省。

命令訂之。又同法第29條、第30條、第31條、第33條對於中央二級機關

稱「部」、「委員會」及中央三級機關稱「署」、「局」之設立,以及

內部單位之設立有其數量之上限規範。

※補充:我國政府再造之因由(宋餘俠,2009)一、 政府組織及員額不斷膨脹,導致財政負擔增加

我國行政院所屬部會數目自35年制憲以來增多減少,目前已高達37個之

多,遠多於世界各主要國家的部會數(大多在15~25個之間)。就垂直

分工而言,部會之下又設三個層級,機關總數超過700個。機關數增多,

政府部門相對地必須配置更多的人力,不斷膨脹的員額造成了極大的財

政負擔。

二、 行政組織調整過於僵化,難以適應環境變遷

依據中央法規標準法規定, 國家機關組織必須以法律定之。因此,行

政機關

即便是增加一位編制人員,都必須透過修法的方式才能合法進用。所幸

在基準法完成立法後,此一僵化情形稍微獲得紓解,但對於達到「依環

境變遷適時調整政府組織, 即時挹注資源在民眾關注的施政議題」之

目標上,仍有相當距離。

三、 機關業務分工過細,協調界面過多造成民眾洽公成本增加

現代政府組織及員額規模龐大,除了增加溝通協調及財政成本外,也造

成民眾洽公的不便,實應以服務民眾為中心理念調整內部單位之組設。

雖然這可以透過行政程序及業務流程之簡化加以改善,但當這樣的改進

仍然無法提高效率時,就必須進行組織結構的調整。以行政院目前有37

個部會來看,為了處理跨部會業務協調而成立了二十多個任務編組,無

形中在行政院與部會間增加了一

個決策層級。如此仰賴跨部會協調機制的結果,造成部會無法負起完全

的政策責任;並且同一層級的機關過多與業務分工過細,也容易成為推

託與卸責的藉口。

※補充:「中央行政機關組織基準法」修正評議(陳鳳美,2009)

揭櫫世界各先進國家組織管理經驗,除日本訂有「國家行政組織法」及

韓國訂有「政府組織法」外,多未制訂基準性組織法律規範。同時,前

Page 57: 主題:公共管理的基礎理論 - web.thu.edu.twweb.thu.edu.tw/lu.bk/www/news/99-1.pdf · 主題:公共管理的基礎理論 焦點一:試說明Weber對傳統官僚制度的反省。

開日韓政府組織法律規範,未對機關組織及其內部單位設置訂有總量管

制限制。反觀我國基準法規定,對於各級機關、內部單位設置總數訂有

總量管制之限制,以及中央三級以上機關均需以法律規範其組織,實難

以適應瞬息萬變的環境變遷,以應新增政務需要迅速調整機關組織,造

成組織僵化、缺乏彈性與活力。因此,為因應機關組織彈性調整之實際

需要,我國應效法國外經驗,檢討鬆綁並簡化基準法。

一、鬆綁之必要性與迫切性

(一) 鬆綁機關總量管制

除了讓行政部門得以因應實際需要調整機關組織設計,也讓立法部門仍

可藉由審議二級機關組織法中有效監督其所屬三級機關設置之必要

性,而更有多餘心力致力於民生法案之審議,時可謂行政立法部門雙贏

之局面。

(二) 創設多元化組織設計

為提供政府因應社會多元需要,多元化組織設計,基準法將檢討明確定

義「機構」為「機關依組織法規將其部分權限及職掌劃出,以特殊設備

或設施及經營方式以達成特定目的之組織」,並增列社福為機構類型;

另為賦予獨立機關更大彈性,兼顧行政一體原則,有關其合議制成員專

兼任及人數規定及任命方式等,有必要考量個別獨立機關之特殊需要另

予規範;同時,也將致力推動行政法人制度,鬆綁現行人事及經費限制,

以應機關彈性用人實際需求,強化行政部門回應民眾應變能力。

(三) 兼顧中央與地方組織設計及人員交流

鑑於地方制度法業於96年7月11日修正公布,縣(市)政府一級單位定

名為處,所屬一級機關定名為局,二級單位及所屬一級機關之一級單位

為科。惟現行基準法規定,中央四級機關業務單位名稱為課,為兼顧中

央與地方行政機關組織架構設計及人員交流之衡平性,實有必要檢討修

正相關規範,以應實際需要。

二、修法重點

(一)增列外交機關為本法有特別規定而從其規定之適用對象,並增訂

機關不必逐級設立,惟明定隸屬指揮監督關係之規定。

(二)明確定義「機構」,並增列社會福利機構類型。

(三)增加一級機關副幕僚長人數為一人至三人,並明定至少一人應列

常任職務。

Page 58: 主題:公共管理的基礎理論 - web.thu.edu.twweb.thu.edu.tw/lu.bk/www/news/99-1.pdf · 主題:公共管理的基礎理論 焦點一:試說明Weber對傳統官僚制度的反省。

(四)增訂獨立機關合議制之成員應明定停職、免職之規定及程序,並

賦予行政院院長指定成員為首長及副首長之任命。

(五)配合地方制度法修正後,修正四級機關內部單位組設。

(六)鬆綁中央三級機關以下機關由行政院以命令定之,並刪除行政院

所屬部會、獨立機關、三級機關建置規模標準及其內部單位數之總數上

限。

(七)明定二級機關及三級機關設立臨時

性、過渡性機關之核定程序。

(八)修正條文施行日期由行政院另定之。

※補充:獨立機關與行政院關係運作說明依據司法院釋字第 613 號解釋理由書,獨立機關之存在對於行政一

體與責任政治原則有所減損,其設置應屬例外。唯有設置獨立機關之目的確係在追求憲法上公共利益,所職司任務之特殊性,確有正當理由足以證立設置獨立機關之必要性,重要事項以聽證程序決定,任務執行績效亦能透明、公開,以方便公眾監督,加上立法院原就有權經由立法與預算審議監督獨立機關之運作,綜合各項因素整體以觀,如仍得判斷一定程度之民主正當性基礎尚能維持不墜,足以彌補行政一體及責任政治之缺損者,始不致於違憲。

中央行政機關組織基準法(以下簡稱基準法)第 3 條第 2 款創設之獨立機關,自應符合司法院釋字第 613 號解釋意旨,必須有正當理由足以證立設置獨立機關之必要性,其建制目的在於因特定裁決性、管制性或調查性之公共任務,特別需要踐行高度專業化、去政治化或充分顧及政治與社會多元價值,乃授與獨立機關以合議制型態,依據法律獨立行使職權自主運作,除法律另有規定外,不受其他機關指揮監督。是以,獨立機關組織之運作必須遵循以下重要原則。1、獨立行使職權之意旨,依據司法院釋字第 613 號解釋,僅指在法律規定範圍內,排除上級機關在層級式行政體制下所為對具體個案決定之指揮與監督,使獨立機關有更多不受政治干擾,依專業自主決定之空間。故僅限於具體個案決定有所謂獨立行使職權可言,不包括政策決定領域。2、獨立機關依據法律主管之業務應以裁決性、調查性或管制性為主,不負責施政政策、產業輔導或獎勵等業務,若裁決性、調查性或管制性之個案決定涉及施政政策領域時,仍應與行政院各主管部會協調溝通,並接受行政院之政策決定。再者,其行使職權之個案決定雖不受行政院或其他行政機關之適當性及適法性監督,然以獨立機關仍屬中央行政機關,舉凡中央行政機關在性質上應一致遵行之行政管理事項仍應遵守,否則行政院作為憲法規定全國最高之行政機關,其完整之行政管理權限即難以維持。

Page 59: 主題:公共管理的基礎理論 - web.thu.edu.twweb.thu.edu.tw/lu.bk/www/news/99-1.pdf · 主題:公共管理的基礎理論 焦點一:試說明Weber對傳統官僚制度的反省。

為使獨立機關與行政院及其所屬部會機關間之關係明確釐清,確保獨立機關職權之順遂行使及行政院作為最高行政機關之行政權完整,茲就相關事項二者間之關係分述如下:一、獨立機關之人事任命

依釋字第 613 號解釋,行政院為國家最高行政機關,基於行政一體,必須為包括獨立機關在內之所有行政院所屬機關之整體施政表現負責,並因獨立機關施政之良窳,與獨立機關首長、副首長及委員之人選有密切關係,故行政院原則上擁有對獨立機關首長、副首長及委員之具體人事決定權。立法者雖有一定之制衡權限,惟仍以不侵犯行政權之核心領域,或對行政院權力之行使造成實質妨礙為限。應本於釋字第 613 號解釋,須考量行政一體與責任政治之原理,並依獨立機關之任務性質個別設計,不受限於基準法第 21 條之規定。

就獨立機關內部文官任用部分,仍應依現有各部會之制度運作,即如同各部會一般,獨立機關對其一級主管人選提出建議,行政院則有決定權。

二、獨立機關與行政院政務之關係獨立機關行使裁決性、管制性及調查性之具體個案決定時,

雖不受行政院指揮監督,惟若其執行業務有涉及施政政策或產業輔導等事項時,須先與主管部會機關充分溝通、協調後,或由負責督導相關業務之行政院政務委員主持協商,再報請行政院核定之;部會如遇有需協調事項時,亦應主動要求與獨立機關會商,並由行政院負最後政策決定之權責。

獨立機關之任務功能在超然獨立執行裁決性、管制性或調查性之個案決定事項,並不負有政策諮詢或政策協調統合功能,依行政院所頒獨立機關建制原則規定,獨立機關之首長,非為行政院之閣員,毋需參與行政院院會,惟若因院會討論議程有實際需求時,仍應受行政院院長指示列席行政院院會。此外,依據司法院釋字第 613 號解釋所揭示之行政一體原則及精神,獨立機關仍有義務依行政院之要求,參與行政院必要之會議,包括行政院所設之各任務編組會議,就其法定職掌範圍,共同完成公共任務,並遵守會議結論。

三、獨立機關主管法律之訂修獨立機關主管之法律案,包括組織法案與作用法案,其報院

審查程序均與一般機關相同,並應經行政院會議議決後,函送立法院。

如前所述,獨立行使職權之意旨,依據司法院釋字第 613 號解釋,僅指在法律規定範圍內,排除上級機關在層級式行政體制下所為對具體個案決定之指揮與監督,使獨立機關有更多不受政治干擾,依專業自主決定之空間。故僅限於具體個案決定有所謂

Page 60: 主題:公共管理的基礎理論 - web.thu.edu.twweb.thu.edu.tw/lu.bk/www/news/99-1.pdf · 主題:公共管理的基礎理論 焦點一:試說明Weber對傳統官僚制度的反省。

獨立行使職權可言,不包括政策決定領域。法律案均涉及政策決定事項,故如法律將獨立機關定為該法主管機關時,僅表示獨立機關對此等法案有草擬之權限,該法案之報院審查程序均與一般行政機關相同,獨立機關並無自行提出法律案於立法院之權限,其主管之法律案依據憲法第 58 條第 2 項規定,應經行政院會議議決之。

四、獨立機關應經行政院核定或同意之事項獨立機關作為中央行政機關之一,仍屬行政權之範疇,性質

上屬中央行政機關應共同遵守之通案事項,應經行政院核定或同意,例如:

(一) 處務規程依據基準法第 8 條第 1 項規定:「機關組織以法律制定者,

其內部單位之分工職掌,以處務規程定之」。復依該條文之解釋適用事項說明:「以處務規程定之者,由一級機關核定」。又基準法第 32 條第 1 項及第 34 條對於獨立機關業務單位及輔助單位之建制乃有所規定,其處務規程有關內部單位建制規定不得與之相違背,且鑒於此等內部單位之建制涉及各種高階職務人員(如參事、技監等)及主管人員(如處長、科長等)之設置,亦與編制、員額及待遇有關,有各中央行政機關一體適用及衡平性問題,故其處務規程仍應經行政院核定後,再由獨立機關自行發布並函送立法院查照。

(二)員額管理中央政府機關總員額法草案已就中央政府機關之總員額總

數最高限有所規定,獨立機關作為中央行政機關之一,其員額乃須計入行政院獲配之員額其配置員額總數應就行政院分配之總數定之。其餘員額管理事項並依該法規定辦理。再者,獨立機關相關職稱及官等職等之配置,公務人員任用法、各機關職稱及官等職等員額配置準則仍有其適用。鑒於此一部分涉及員額編制及官制官規,獨立機關與其他中央行政機關並無不同,其編制表仍應報行政院核定,並送考試院核備。至於人事費之編列,亦仍依現行預算員額審查方式,與其他中央行政機關為相同之處理。

(三)預決算處理依憲法第 58 條第 2 項及同法第 59 條規定,中央政府總預算

案及其他預算案(如追加減預算、特別預算)之提案權屬於行政院,其基本理念在於指定預算籌編權責機關負起促成預算獲得審議通過之責,並維護預算之完整性。獨立機關作為中央行政機關之一,應與其他中央行政機關為相同之處理,其預決算仍應納入中央政府總預算及總決算,由行政院提送立法院。

(四)施政方針及施政報告行政院依憲法規定,向立法院提出之年度施政方針及施政報

Page 61: 主題:公共管理的基礎理論 - web.thu.edu.twweb.thu.edu.tw/lu.bk/www/news/99-1.pdf · 主題:公共管理的基礎理論 焦點一:試說明Weber對傳統官僚制度的反省。

告,自應涵括所屬各機關施政範疇。獨立機關既為中央行政機關之一,亦須就其法定職權業務提出施政方針及報告,送行政院核定後,納入行政院年度施政方針及施政報告彙編,併送立法院。

(五)施政計畫依預算法規定,行政院部會均應提出年度施政計畫送立法

院。獨立機關既為行政權之一環,自亦須就其法定職權業務,與行政院部會同步提出施政計畫,俟送行政院核定後,納入年度施政計畫彙編,併送立法院。行政院部會對於獨立機關所提施政計畫內容,如認有涉及其政策性職掌時,自得要求與獨立機關會商及進行必要之調整。

五、其他行政管理事項均與一般行政機關相同,例如:(一)人事、會計、政風事項

獨立機關作為中央行政機關之一,如法律無特別排除規定,凡涉及人事、會計、政風一條鞭制度者,仍應依相關法制規定處理,與其他中央行政機關並無不同。

(二)其他行政管理事項1.刊登公報作業:行政院自 94 年 1 月起發行「行政院公報」,統一刊載行政院及各中央行政機關涉及人民權益之法令等重要事項。獨立機關依據行政程序法及政府資訊公開法訂定法規命令時,仍應送刊行政院公報,以維持中央行政機關公報之統一性。

2.出國報告管理作業:依據政府資訊公開法之規定,由政府機關編列預算派赴國外從事考察、進修、研究或實習所提出之出國報告,應主動公開。行政院研究發展考核委員會為便利機關內部管理出國報告並提供民眾便捷之查詢管道,已建立「公務出國報告資訊網」管理平台,掌握各機關出國計畫執行情形(http://open.nat.gov.tw/reportwork),獨立機關亦應依此機制辦理。

3.出版品管理作業:行政院研究發展考核委員會業已建立政府出版品管理制度,訂定政府出版品管理辦法及統一編號、基本形制、定價銷售等相關作業法規,供各行政機關遵循辦理。同時並 自 86 年 起 , 建 置 政 府 出 版 品 網 ( 網 址 :http://open.nat.gov.tw/gpnet),提供便利之工具以輔助承辦人員辦理政府出版品管理作業,並讓民眾得以迅速掌握所有機關完整之出版資訊,獨立機關亦應依上開規定辦理,並至政府出版品網登錄及維護書目資料。

4.施政管考作業:目前各行政機關所辦理之中程施政計畫、績效管理及評估等工作,行政院均訂有相關之規範,既不影響獨立機關行使職權之獨立性,且有助於其行政效率之提升,獨立機關亦應遵守。

Page 62: 主題:公共管理的基礎理論 - web.thu.edu.twweb.thu.edu.tw/lu.bk/www/news/99-1.pdf · 主題:公共管理的基礎理論 焦點一:試說明Weber對傳統官僚制度的反省。

六、有關國家通訊傳播委員會(NCC)之過渡處理方式國家通訊傳播委員會(NCC)組織法業於 94 年 11 月 9 日總

統令制定公布,惟有關委員產生方式因實質剝奪行政院之人事決定權,業經司法院釋字第 613 號解釋認定違憲,應於最短時間內加以修正。

在完成修法以前,為彌補其委員產生方式違反憲法權力分立、行政一體、責任政治原則及憲法保障通訊自由之意旨,包括對於其人事、預算,行政院得加以監督,並為適法之處理。涉及NCC 職掌之法案,不分組織法或作用法,由 NCC 或相關機關研擬,並應報請行政院簽請政務委員審查。不服 NCC 作成之行政處分,其訴願應向行政院提起。至於 NCC 內部單位組織、員額之規劃,亦應報行政院核定或同意。而關於業務權責劃分之事亦應依法處理,處理完畢後即處理人員移撥作業。

至於其他規劃中之獨立機關,是否符合釋字第 613 號解釋意旨,亦應一併加以檢討,俾符合憲法行政一體與責任政治原則。

主題:資源管理

焦點一:請申論「組織發展」的產生背景、意義內涵以及未來

發展趨勢。

【思考方向】

(一)產生背景:

1.1940年代後半期美國國家訓練實驗室的萌芽而產生一連串 的

「試驗室訓練法」(或稱「敏感性訓練」)。

2.行為科學家在「調查研究與回饋」方面的早期貢獻。

上述兩項動力均由麻省理工學院李文教授(K. Lewin)負責推動。

3.社會科學家從事「行動研究」的貢獻。

4.強調生產力和工作生活品質的研究。

5.重視策略性變革與組織的轉型。

(二)意義:

1.吳定教授:組織發展(Organization Development, 簡稱OD)係

以行為科學的理論與技術為理論架構,以特殊行動方案及執行技

Page 63: 主題:公共管理的基礎理論 - web.thu.edu.twweb.thu.edu.tw/lu.bk/www/news/99-1.pdf · 主題:公共管理的基礎理論 焦點一:試說明Weber對傳統官僚制度的反省。

術為基礎,目的在維持更新及改變人為組織制度與人際關係的有

計畫努力。

2.張潤書教授:組織發展係為增進組織效能,運用行為科學的知

識技術,經由外部專家的協助,採用適當的干預技術

(intervention),將個人的需要與組織的目標加以整合的一種有

計畫、目的之改變過程。

3.孫本初教授:組織發展的基本隱喻是視組織為一開放的系統;

其分析單元有個人、團體和組織;其干預的方法包括組織成員的

觀念和價值、組織過程、組織結構,以及整體系統的改變等;其

主要的核心價值在使組織成員學習合作與參與,以互相促進改

變;至於實施計畫性變革的目的在於增進組織的效能及健全。是

以,組織發展與計畫性變革、組織變革三詞在概念上是相通的。

4.傅蘭琪和貝爾(French & Bell, 1978):組織發展乃是一項促進組

織解決問題和革新過程的長期性努力,尤其是透過更有效及更協

同一致的管理方式來改變組織文化,藉由變革推動者(changeagent)的協助,採取應用行為科學的理論與技術。

(三)內涵:

1.研究焦點(focus)費藍德和布朗(Friedlander & Brown, 1974)二人認為組織是由「人

員」、「技術」、「過程與結構」三者所組成,其目的在完成組

織任務和人類自我實現的極大化。循此,其認為組織發展的研究

途徑包括:

(1)人際過程途徑(human-processual approaches)(強調人員、

組織過程):

人際過程的途徑強調人員的參與和組織的過程(例如溝通、問題

解決、決策等),透過此一途徑,組織成員可獲致他們各自的目

標和組織的目標。其干預方法包括調查回饋法( surveyfeedback)、團體間關係發展法(intergroup relations development)等。此途徑所涉及的相關知識包括心理學、社會心理學、人類學

以及團體動態和人群關係等。

(2)技術結構途徑(technostructural approaches)(強調技術、

組織結構):

Page 64: 主題:公共管理的基礎理論 - web.thu.edu.twweb.thu.edu.tw/lu.bk/www/news/99-1.pdf · 主題:公共管理的基礎理論 焦點一:試說明Weber對傳統官僚制度的反省。

技術結構途徑強調的是組織中的技術及結構方面的理論與干預方

法。此途徑的干預方法包括工作設計、工作擴大化、工作豐富化

等。所涉及的相關知識有工程學、社會學、心理學、經濟學以及

開放系統理論等。

(四)未來發展趨勢:(結合張潤書與孫本初教授的版本)

1.張潤書教授觀點:

(1)權變理論的強調與重視

(2)組織發展理論應發展共同理論

(3)擴充組織發展人才的知識領域

(4)組織發展具發展潛力

2.孫本初教授觀點 :

(1)理論方面 :

織發展的概念愈來愈廣

1950至1960年代組織發展研究焦點為組織社會面,並使用人際關

係的干預方法,包括敏感性訓練法、過程諮商法及團隊建立法。

1970年代組織發展研究焦點為則兼顧組織的社會面和技術—結構

面,並使用技術結構的干預法,例如結構設計、工作設計等。1980年代迄今,組織發展的研究焦點則大幅的擴散,不僅其概念範圍

變得愈來愈廣,而且使用的干預方法也愈來愈多,例如在人力資

源的研究領域中就使用酬賞制度、生涯規劃與發展,以及壓力管

理等方法。而在組織發展的概念與理論方面,不僅注意到組織結

構、組織與環境的關係,同時亦注意到組織文化以及策略規劃與

執行等,此導致一些新的干預方法紛紛出籠,例如:文化改變、

策略變革、自我設計的組織,以及轉換式的組織發展

(transorganzational development)等方法的使用。這些干預活動

的目的除了繼續追求組織成員的福祉及組織的效能以外, 尚擴及

對環境關係和外在利害關係人的關注,例如:政府、顧客、競爭

者等。

組織發展將趨向建立整合性的權變理論:

由於組織發展理論與干預方法的龐雜,亟需以兼具整合、權變的

觀點加以結合。目前已有多位學者從事這方面的努力,例如:康

明斯與休斯(Cummings & Huse, 1989)兩人在組織發展的診斷模

Page 65: 主題:公共管理的基礎理論 - web.thu.edu.twweb.thu.edu.tw/lu.bk/www/news/99-1.pdf · 主題:公共管理的基礎理論 焦點一:試說明Weber對傳統官僚制度的反省。

式中,整合了組織、團體以及個人等三個層面的干預活動;又如

迪起(Tichy, 1983)所主張的策略變遷管理(strategic changemanagement)模式乃是整合組織中有關技術、政治及文化等層面

的組織活動的管理模式;再者,布拉克與佩騰(Bullock and Betten,1985)兩人所提出的整合性的計畫變革模式(integrative model ofplanned change)亦是整合了階段(phase)與過程(process)兩

方面。

組織發展對於組織的看法將趨向於視組織為複合重疊的系統:

一般組織發展的教科書將組織界定為「開放系統」,而且強調組

織的共同目標。但布朗與柯維(Brown and Covey, 1987)兩人認

為未來的組織發展應該把組織看成為一個複合的重疊系統,並容

許組織具有多重的目標與服務對象。例如:政府機構、公用事業、

大學、醫院等組織就具有多 重的目標與服務對象。

(2)實務方面 :

從組織診斷走向組織學習:

1960至1970年代的組織發展強調組織的診斷,此時期在實務上採

用干預方法的目的,在使現存的組織結構和過程能夠運作得更

好,並幫助組織能偵測與矯正偏差的行為,此即阿吉里斯與史恩

(Argyris and Schon, 1978)兩人所稱的「單回饋圈學習」

(single-loop learning)。然而,在1980年代以後,有許多公司為

了要因應複雜且迅速變遷的環境,組織必須作大幅度的改換。此

種大規模的組織轉換,基本上是要改變組織的策略和價值,以便

能管理組織現存的結構與行為,這就是所謂的「雙回饋圈學習」

(double-loop learning)(Argyris & Schon, 1978)。歸結而論,

組織診斷僅是針對組織的功能,加以偵測錯誤並予枝節性地矯正

自己,此是屬於單回饋圈的學習。然而未來的組織環境將愈趨複

雜,而且變遷亦愈為迅速,所以,組織發展必須改採雙回饋圈的

學習方法,亦即針對組織的策略與價值,做根本上的變革,以創

新組織,適應環境。

從諮商取向走向企業取向:

比爾與瓦頓(Beer and Walton, 1987)兩人指出:組織發展中許多

較新的概念與方法皆強調,組織發展中的干預活動,應從傳統中

Page 66: 主題:公共管理的基礎理論 - web.thu.edu.twweb.thu.edu.tw/lu.bk/www/news/99-1.pdf · 主題:公共管理的基礎理論 焦點一:試說明Weber對傳統官僚制度的反省。

以管理顧問為中心走向由一般管理人員來負責。這種風潮已漸漸

興起,例如在管理的文獻中有關公司文化、策略變革管理,以及

轉換型領導(transformational leadership)等皆強調此種概念。而

且,一般管理人員愈來愈覺得「管理變革」(managing change)的必要性,所以,他們在變革的策略活動中,扮演更積極的角色,

並使用嚴謹的評估方法,促使組織發展走向企業取向。

從適應環境走向管理環境:

費佛兒與史連西克(Pfeffer and Salancik, 1978)兩人提出一種對

環境的分類方法,他們把環境分為三種:(1)系絡的環境(contextualenvironment):指間接影響組織的外在環境;(2)交易的環境

(transactional environment):指直接與組織互動的環境;及(3)創訂的環境(enacted environment):指組織所察覺到而自行描繪

出來的環境。由於「系絡的環境」和「交易的環境」均已經間接

或直接影響到組織,而「創訂的環境」則是組織所預見者,其尚

未影響到組織。目前組織發展的方法已從只注重組織如何及時地

適應其系絡的環境與交易的環境,走向以前瞻性的眼光去察覺「創

訂的環境」,並試圖將其改變為組織所喜愛的環境。例如:企業

組織常從事政治活動以影響政府的法律與管制的措施;或藉由垂

直和水平方式的整合活動以控制原料和市場。不過,私人企業組

織在從事此類「前瞻性」的活動時,必須考量成本與社會倫理的

問題。

焦點二:何謂「組織精簡」?

【思考方向】

組織精簡,作為一種干預技術,係指組織有計劃地減少組織中的

職位及工作,亦即縮減組織的規模。

1.推行組織精簡的步驟

一般而言,成功的組織精簡措施大都依循以下的步驟:(CummingsandWorley,2005:290-292)(1)釐清組織的策略

首先,組織的領導者必須指出組織的策略為何,並且說明組織精

簡與組織策略有何關係,其目的在告知組織成員,精簡本身並非

Page 67: 主題:公共管理的基礎理論 - web.thu.edu.twweb.thu.edu.tw/lu.bk/www/news/99-1.pdf · 主題:公共管理的基礎理論 焦點一:試說明Weber對傳統官僚制度的反省。

目的,而是達成策略目標的一項重組過程(restructuring process)。在整個過程中,領導者應讓員工有機會表達關心,提出問題並且

必要時能獲得諮商。

(2)評估精簡方案並做抉擇

策略一旦清楚了,整個精簡方案便可提出,同時加以評估。一般

而言,精簡可透過三個方法來做。但一項特定的精簡方案則可同

時涵蓋三種途徑。人力縮減措施目的在於短期內減少員工的人

數,方法包括自然耗損(即遇缺不補)、鼓勵提早退休、轉業服務

與資遣。組織重設計在於調整組織的結構以便為下一個階段的成

長預做準備。此一方案屬中期措施,可透過合併組織的部門,消

除管理階層以及重新設計工作來達成。系統的重設計(systemicredesign)是一長期性的措施,目的在改變組織的文化與策略取

向。它可以包含改變組織內每位成員的責任與工作行為,鼓吹持

續改善使之成為組織的一種生活方式。

不幸的是,有些組織常選擇像資遣等顯而易見的方法,因為比較

能快速執行。此舉會產生一種恐懼與防衛的氣氛,因為大家都只

注意誰會被迫離開。如果能夠檢視整體的方案並考慮組織整體的

情況,而不只是考慮某一部分的話,將可以緩和這種恐懼的氣氛。

再者,如讓員工參與此種精簡方案的話,則更能具有正面的效用,

因為它能激發一種迫切感,進而想出資遣以外的其他實施方案。

參與可讓員工更清楚精簡是如何進行的,因而能夠增加變革成功

的可能性,亦即不論採取什麼措施,員工均會認為是合理而公平

的。

(3)執行變革

此一階段即是組織人員裁減的執行。以下幾個實務的作法可供參

考以確保執行的成功。第一,執行命令由上而下確實掌控。做一

些困難的決策在所難免,寬廣的視野有助於克服人們保護自己業

務或門戶的本能。第二,明確指出無效率與高成本的領域作為精

簡的標的。那些眾所週知的人力閒置的領域如果絲毫未受影響,

將會大大打擊士氣。第三,將某些特定行動與策略連結起來。讓

大家知道,重組活動只是組織改善績效方案的一部分而已。最後,

經常透過各種媒介與員工溝通。此舉將使員工保有暢通的資訊、

Page 68: 主題:公共管理的基礎理論 - web.thu.edu.twweb.thu.edu.tw/lu.bk/www/news/99-1.pdf · 主題:公共管理的基礎理論 焦點一:試說明Weber對傳統官僚制度的反省。

降低他們的焦慮,使他們較易專注工作。

(4)照顧留任的與離開的員工的需要

大部分的精簡措施最後都涉及到人力的減少,因此,對於不論是

留任或要離開的員工都必須給予關切與照顧。對於留任而言,勢

必增加工作的負擔,再加上對於此次資遣以及未來可能被資遣所

產生的焦慮,極有可能發生學者所稱的「生還者症候群」(survivorsyndrome):自顧(self-absorbed)與逃避風險行為。這種人常存下一

次會被資遣的恐懼,對工作不熱心,對未來無確定感、對管理階

層有戒心,對被資遣的同事有內疚感。

對於這些留任人員,組織可以提供更多的溝通,告知組織未來的

願景是什麼,有些什麼目標與策略。強調他們的工作表現對組織

目標達成的重要性,使他們有一種被重視的感覺。當然提供他們

從事新工作所需的訓練亦是照顧的一項方法。高階主管可以透過

讓更多的員工參與決策來強化組織給予員工的重要訊息:員工對

於組織未來的成功與成長是重要的。

而對於被迫離職的人員而言,組織應提供一切必要的協助,包括:

轉業諮商(outplacement counseling)、個人與其家人的諮商、整套

離職金方案、尋找新工作的協助,搬家的服務以及工作的再訓練,

提供這些協助的目的就是期盼離職者能快速過渡到新的工作。

(5)以組織成長計畫作為精簡措施的“完結篇”

組織精簡的最後一個階段就是推行一個組織的更新與成長計畫。

許多研究顯示,未能盡快推出一個成長的方案,常是精簡計畫失

敗的主因。在推行更新與成長方案時,組織必須讓員工知道方案

的內容以及他們在其中的角色。雖然在精簡過程中,員工們經歷

了一段艱難的日子,但他們需要確知這個更新的方案能讓組織向

前行。

2、精簡的結果

許多實證研究的結果顯示,組織精簡的結果大多數是不好的2。例

如一次對1005家實施精簡的公司所做的調查顯示,只有不到一半

的公司指出,他們達成了當初所設定的成本降低目標。只有百分

之二十二的公司獲致所預期的生產力利益,因而大約有百分之八

十的公司必須重新聘請一些已被解職的員工。另一項對1142家實

Page 69: 主題:公共管理的基礎理論 - web.thu.edu.twweb.thu.edu.tw/lu.bk/www/news/99-1.pdf · 主題:公共管理的基礎理論 焦點一:試說明Weber對傳統官僚制度的反省。

施精簡的公司所做的調查結果顯示,只有三分之一的公司達到生

產力的目標。此外,此項調查亦顯示,在員工個人方面產生許多

的問題,如壓力與疾病的增加、自尊心的喪失,信任感與忠誠度

的降低以及婚姻與家庭的危機。另外,調查的結果亦顯示,組織

精簡在財務績效方面並未獲得有利的結果。例如,210 家實施資

遣的公司,其資遣後第一 年的財務績效雖然有所改善,但第二年

並未持續。這些結果顯示,資遣可能產生初期的財務績效改善,

但這些利益卻只是短暫的,且無法維持未資遣前的水準。

焦 點 三 : 組 織 重 組 (Reorganization) 、 組 織 再 造

(Reengineering)、政府再造( Reinventing Government)三者的概念與關係?

【答題方向】

1.組織重組:組織為了因應科技、政治等外在環境的影響,而所從事結構的重

新組合、設計與安排。如依據我國「中央行政機關組織基準法」

的規定,我國未來中央行政機關將設立 13 個部,4 個附屬委員會,

5 個獨立機關等共 22 個機關,行政院組織法即須配合修正,即為

組織重組的適例。

2.組織再造:組織再造乃是以顧客為導向的前提下,徹底、重新建構組織的作

業流程,而這種流程上的改變,亦可能導致組織結構上的改變,

例如再這工程的核心要素之一,資訊科技的進步(其他要素分為顧

客導向,以流程為中心,目標導向與系統性思考等), 像透過網際

網路的資訊共享,即可能改變原有的組織設計與安排。

3.政府再造:係以體制內改革的「新右派」主張為經,以「市場經濟」、「企業

精神」為緯,透過「標竿學習」的方式,萃取成效卓著的案例經

驗,借用成功的企業精神經營技術,重新建構行政文化、公務人

力、權責歸屬、獎懲制度以及任務目標等層面,期能活化文官體

系,大幅提高其效率、效能與治理能力,促成行政組織徹底轉型,

實現政府的公共目的,有效擴增公共利益。

Page 70: 主題:公共管理的基礎理論 - web.thu.edu.twweb.thu.edu.tw/lu.bk/www/news/99-1.pdf · 主題:公共管理的基礎理論 焦點一:試說明Weber對傳統官僚制度的反省。

焦點四:政府機關運用「策略性人力資源管理」時應注意哪些

原則(範圍)?又何謂 TLC 概念?

【答題方向】

一、 學者 Leonard Nadler and Zeace Nadler ( 1989 : 4)

將人力資源發展的範圍區分為包括三個活動領域:訓練

(Training)、教育(Education)、發展(Development),其定義如下:

(一)訓練(Training) :

乃是針對學習者目前工作所進行的學習。是為了改善員工目前的

工作表現,或增進即將從事工作的能力,以適應新的產品、工作

程序、政策、和標準等,以提高工作績效,其對工作的影響是立

竿見影在性質上雖是一種花費,同時也可視為投資。

(二)教育(Education) :

針對學習者未來工作所進行的學習。是欲培養員工在某一特定方

向、或提昇目前工作的能力,以期配合未來工作力的規劃,或擔

任新工作、新職位時,對組織能有較多的貢獻。嚴格來說,公務

人員的進修就是一種教育的功能,其目的不在對目前的工作技術

加以精進,而是對未來提供一種發展的潛能,機關與主管應該對

此加以鼓勵,並積極的提供各種可能的管道。

(三)發展(Development) :

不是針對學習者工作所進行的學習。其目的在獲得新的視野、科

技和觀點,使得整個組織有新的發展目標、狀態、和環境;除了組

織發展外,也包括個人的發展,培養繼續學習的意願,具備自我

發展的能力等。

二、 TLC 概念的提出:

John Naisbitt ( 1985)亦提出相同具有啟發性的概念,並強調工業社

會的學校教育重視 3R,即閱讀(reading)、寫作(writing)、算數

(arithmetic),但在變動迅速的資訊社會中則必須以 TLC 概念來取

代工業社會的 3R,方能有效因應當前多元複雜的社會。而所謂

TLC 的概念說明如次(吳瓊恩等人, 2005 : 212-214) :

(一)學習如何思考(Learning How to Think) :

所謂思考即是綜合與概化的能力,能分類推論,並區分事實與價

Page 71: 主題:公共管理的基礎理論 - web.thu.edu.twweb.thu.edu.tw/lu.bk/www/news/99-1.pdf · 主題:公共管理的基礎理論 焦點一:試說明Weber對傳統官僚制度的反省。

值,須要有哲學的基礎,此在學校教育中應成為必修課程須要有

哲學的基礎,此在學校教育中應成為必修課程。

(二)學習如何學習(Learning How to Learn) :

工業時代的官僚體制,其層級節制與專業分工體制,重視控制、

紀律、服從等價值,容易挫傷員工的學習意願,妨礙學習的勇氣,

而這正是 90 年代興起學習型組織的背景。

(三)學習如何創造(Learning How to Create) :

工業時代的產品是大規模生產的結果,因此一致化的價值成為主

軸,但此在資訊社會或後現代社會重視個別差異的時代,就變成

了問題。學習如何創造的能力,除了培養批判思考能力的哲學素

養外,還要時時向既有的規則提出挑戰,並在輕鬆愉快的氣氛下,

暫停理性推理的思維,使靈感從直觀中突然閃現,或者從事靜坐

(meditation) 也有助於創造力的培養。

焦點五:我國推動策略性人力資源管理之特點及修正建議?

【答題方向】

1. 由單一機關主導推動:

策略性人力資源管理由人事行政局主導,負責政策內容設計

與執行。此策略優點在於架構完整、便於整合,惟因政府各

機關不論在業務、層級、規模上均有極大差異,個別需求可

能於規劃中受到忽略。因此,在方案規劃完成後,必須不斷

透過反饋的過程。參酌各機關意見進行「政策設計」,方能避

免形式化的問題。

2. 以公部門需求為策略考量:

除了員工心理諮商係考量員工的需求而設計以外,其餘各項

措施均是以公部門本身的需求來考量,並且即便員工心理諮

商亦多是一種事後性質的補救措施,並無法反饋而對公部門

的措施進行調: 整,如此一來,各項措施是否能獲得公務員

普遍支持不無疑問。為避免此問題的發生,以利害相關人的

角度來分析公務員的需求,將之納入績效評量的「平衡計分

卡」當中,是應考慮的方向。

3. 人事框架下的細部修正:

Page 72: 主題:公共管理的基礎理論 - web.thu.edu.twweb.thu.edu.tw/lu.bk/www/news/99-1.pdf · 主題:公共管理的基礎理論 焦點一:試說明Weber對傳統官僚制度的反省。

傳統的公部門人事行政當中,由於各部門業務分工細緻,員

工各行其事,缺乏相互之間的競爭機制,文官管理法制也多

是一體適用,因此造成文官體系人才進用與流動的內部化及

封閉化。為了使得策略性人力資源管理能夠得到落實,必須

使各項僵硬的人事、管理制度予以放寬,使公部門在進用人

力、管理措施等均能得到更大的彈性。

4. 績效管理未能全面化:

績效管理的措施,包括運用大幅度的契約性人力資源,以機

關使命為方向的績效導向訓練、績效俸給與績效獎金制度等

等。目前,我國績效管理制度僅有績效獎金得以落實,並且

有流於形式之虞,未來應思索更大幅度進行績效管理制度的

落實。

焦點六:公務人力資本發展策略。

【思考方向】

(一)公務人力資源管理的新挑戰

政府治理模式的改變帶來對公務人員素質及能力的新要求。

鑒於人力資源是組織的基本構成要素,在新治理時代,民眾要求

政府要提供多元化的服務,新的工作內容出現,過去政府工作是

適用法令維持穩定,現在則需要提供不同的服務,面對新的問題。

因此,公務人力管理的轉型,是必要且無法避免的,主要包括「核

心價值的重塑」、「由傳統的人力資源管理發展為人力資本管理」、

「公務人員核心能力的提升」、「制定客觀、公平、公正、可行

的考核和激勵機制。其核心價值則是創新與進取精神(李逸洋,

2004)。所謂核心價值是組織員工所擁有、與在工作上獲得成功

最相關、最關鍵的基本認知、觀念、工作態度與價值觀念,其中

進取精神可以激發創新動機、提供創新的勇氣、昇華創新價值、

保證創新的執行,執行力是行政運作常被忽略的環節,在提出願

景與政策藍圖後,更要立刻付諸實際的行動,促成各項目標的達

成。沒有執行力,即使提出的創新方案再有效,也無法產生預期

的效果。由於認知及觀念為影響公務人員行為的基礎,因此,建

立公務人員數位時代應具備的核心價值乃成為公務人力管理的基

Page 73: 主題:公共管理的基礎理論 - web.thu.edu.twweb.thu.edu.tw/lu.bk/www/news/99-1.pdf · 主題:公共管理的基礎理論 焦點一:試說明Weber對傳統官僚制度的反省。

礎。

(二)發展公務人力資本的具體作法

1. 以「策略性人力資源管理」取代傳統人事管理 :

「策略性人力資源管理」係指人力資源管理與組織目標間的有效

聯結,各項人事作為的最終目的均在支援總體目標的有效達成、

創造組織績效與價值,以及發展一種能夠促進創新與彈性的組織

文化。它不同於傳統的人事管理。傳統人事管理較著重於「選才、

用才、育才、留才」的例行性行政作業,以及消極人事管制的功

能。反之,策略性人力資源管理則強調全觀性與目的性,運用創

新及彈性化的人力資源管理策略,以達成組織的目標。策略性人

力資源管理最重要的訴求之一,就是將人力資源管理的各項功能

(過程),從人力的甄選、運用、升遷、調派、訓練發展、薪資、

獎懲考核、裁汰退撫等,均能做有效的整合,彼此相互支援,以

協助達成組織目標。第二個層面是運用各種「人力資源管理策

略」,將各項人力資源管理功能予以統整,最終促成組織目標的

達成。基此,策略性人力資源管理強調策略管理,人事人員的角

色因而將有所轉變,也就是逐漸由傳統的人事管制功能,轉型為

積極的策略規劃與價值創造功能。人事人員將被賦予更多元的角

色,借用學者Dave Ulrich建構的概念,人事人員必須具備下列四

種角色與功能:「策略夥伴」、「員工管理貢獻者」、「轉型與

變革管理者」,及「行政管理專家」。

2.建構基於核心能力本位的培訓模式:

組織的培訓應以能力的提升為導向,要重視提升組織及員工核心

能力的培訓。

最早運用「核心價值」(core value)概念在人力資源管理上的是

美國國防部在第二次世界大戰期間,即開始使用「核心能力」作

為選用及拔擢軍官之依據,到了1990年代,在美國掀起一股「核

心能力運動」。影響所及,一時之間企業界競相針對其各項重要

職務建立核心能力專案並建構相關的管理機制;至於在政府部

門,因1992的《新政府運動》(Reinventing Government)一書及

1993年美國Gore副總統「全國績效評鑑」(NPR)報告,都提到

運用「核心能力」對於提升公務人力素質與政府效能的重要性,

Page 74: 主題:公共管理的基礎理論 - web.thu.edu.twweb.thu.edu.tw/lu.bk/www/news/99-1.pdf · 主題:公共管理的基礎理論 焦點一:試說明Weber對傳統官僚制度的反省。

引起各國政府越來越重視這個新興議題。,公務人員的核心能力

主要包括:「進取」精神、「創新」能力、「專業」技能。能力

本位培訓的理論假設是認為人是有理想的,他們想開發、學習並

運用新的能力。即「資訊不等於知識、知識不等於能力、能力不

等於核心能力」。沒有賦予運作內涵的資訊並不是知識;知識只

存在組織之中,只存在組織中人們相互協作的運作過程之中。在

人們為完成某一運作任務而從事的互動協作、思維溝通、彼此構

築思想時,才形成了知識。同時,只有當知識成為現實的生產要

素時,才能成為一種能力。因此,各種專業知識本身不等於生產

力,也不等於能力。

3. 建立、完善績效考核機制:

公部門最重要之任務,即在於達成政府施政目標,而公務人

員正是執行政府施政計畫與方案之重要成員。而績效考核是組織

管理工作中的一項重要任務,是保障並促進組織內部管理機制有

序運轉、實現組織各項管理目標所必須的一種管理行為。因此,

如何使得公務人員瞭解施政目標,進而有效規劃並執行施政計

畫,並定期確認公務人員執行公共部門計畫與方案之進度與成

效,也就成為公務人力資源管理之重要一環,而其所顯示出之意

義,即在運用績效管理中之績效考核措施來為組織提供有效之回

饋機制。

4. 建立與健全公平的激勵機制

如何不斷增強公部門的活力和公務人員的素質,提高效率、

減少開支、改善服務,為政府樹立良好的形象,已成為一個各國

普遍的重大課題。要妥善解決這些問題,除了要有一系列的規章

制度來規範公務人員的行為外,如何進一步激發公務員的工作積

極性是一個十分重要的關鍵環節,這就要建立和完善有效的激勵

機制,通過各種有針對性和有效的激勵機制,使公務人員不僅能

滿足其基本需要,還能向更高層次的目標奮鬥,才能真正使得公

務員成功地履行其的義務,為政府組織提供更優質的服務。

5. 提升公務人員的忠誠度

Page 75: 主題:公共管理的基礎理論 - web.thu.edu.twweb.thu.edu.tw/lu.bk/www/news/99-1.pdf · 主題:公共管理的基礎理論 焦點一:試說明Weber對傳統官僚制度的反省。

組織的優勢在於人才,人才作用的發揮首先在於人的忠誠。依據

專業調查機構有關於員工忠誠度調查發現,能培養員工忠誠度的

幾大因素包括迎合員工的興趣所在、給予員工發展機會、提供員

工適當職位以及合理的薪酬,其中存在於組織內部的公平對培養

和提升公務人員的忠誠度非常重要。而要建立高忠誠度還必須培

養員工的歸屬感。

6. 實施外包策略,提高人力資本管理效率

公務人員的績效評估目前剛剛起步。政府對績效評估認識和

重視程度不夠,所謂的政府績效評估實際上長期處於流於表面和

形式化,考核的結果一般都是每個公務人員的績效差別不大,不

能真正發揮激勵作用;評估的內容也比較單一,主要係就一些事

務性的工作進行評估,而缺乏對公務人員創新能力方面的評估。

因此如果實行外包,通過聘用專業機構運用較為科學的評估方法

對公務人員的績效進行評估可以在相當程度上解決低效或無效的

問題,一方面專業機構的評估技術較為成熟,另一方面,作為第

三方的專業評估機構可以站在中立的角色進行評估,有利於規避

公務部門自己制定評估規則的同時又用這種規則對自己進行評估

的遊戲,缺少第三方監督所帶來的弊端,如考核標準的科學性、

評估過程的公正性、考核結果的有效性等問題。

焦點七:管理核心能力的建構。

【思考方向】

配合台灣民主轉型的潮流與全球知識經濟時代的來臨,我國政府

當前最大的挑戰之一,乃在於如何獲得與培育優秀的公共管理人

才。無論是邁向二十一世紀、提昇國家競爭力,行政體系扮演著

不可或缺的角色,而優秀的管理人才乃國家所迫切需要的知識資

產,更是實現這些政治願景的基礎。為全面提昇我國公共管理的

品質、強化行政主管的管理核心能力,我國政府在「挑戰2008:國家發展重點計畫」之「整合政府終生學習資源」計畫中,將主

管職務管理核心能力的選定、評鑑及培訓列為主要內容之一。

※Katz 的經典之作:

二十世紀七○年代,管理學大師Katz(1974)於《哈佛企業評論》

Page 76: 主題:公共管理的基礎理論 - web.thu.edu.twweb.thu.edu.tw/lu.bk/www/news/99-1.pdf · 主題:公共管理的基礎理論 焦點一:試說明Weber對傳統官僚制度的反省。

(HarvardBusiness Review)中,發表了〈有效的管理者所應具備

的技能〉一文,文中提出三種管理者必備的技能:技術性技能、

人際間技能與概念化技能。

1.技術性技能(technical skills)所謂技術性技能,也就是作業能力。管理者必須對其所管轄的業

務,有一定程度的瞭解與處理能力。誠然,有時管理者無須也無

法全盤瞭解其所轄業務,而是可以透過部屬之協助而對其所轄業

務有所瞭解,進而善盡指揮督導之責。然而,一個對其所轄業務

一無所知的管理者,恐怕無法與部屬溝通,部屬也無法給他必要

的協助,甚至很可能譏之為「外行領導內行」。所以一位管理者

應具備某種程度之技術性技能,其重要性可說是不言而喻。

2.人際間技能(human or interpersonal skills)如果說技術性技能是「處事」的能力,人際間技能談的便是「待

人」的功夫。在一個組織裡,一般的管理者可說是上有長官,下

有部屬,還有左右一些不相隸屬的同事,如何能與上下左右建立

愉悅有效的溝通、協調、互動模式,建立起信任與合作的人際關

係,可說是管理者十分重要的一項技能。人際間技能的發揮,當

然不是只侷限在組織之內,管理者與組織外相關人士如何建立善

意、互信、互惠的關係,也是一位管理者成敗關鍵之所在。晚近

十分流行的「情緒商數」(EQ; emotion quotient),可說是管理

者的人際間技能之一項衡量指標,而從「情緒商數」此一概念受

到的重視程度,我們不難想像人際間技能對管理者的重要性。

3.概念化技能(conceptual skills)所謂概念化技能,係指具備宏觀視野,能從事形而上、抽象化與

策略性思維的能力。一位優秀的管理者必須要能瞭解國內外政

治、經濟、社會、文化、科技的現況與趨勢,並從組織之中超越

跳脫出來,將整個組織視為大環境中的一個小單元來衡量評估該

組織在大環境中之利基(niche),進而建構願景(vision),提

出策略,為組織的永續經營與發展,規劃出美好的藍圖。從以上

描述可以想見概念化技能對一位管理者的重要性,我們通常批評

一些人「見樹不見林」,可說是一位缺乏概念化技能之管理者的

最佳寫照。

Page 77: 主題:公共管理的基礎理論 - web.thu.edu.twweb.thu.edu.tw/lu.bk/www/news/99-1.pdf · 主題:公共管理的基礎理論 焦點一:試說明Weber對傳統官僚制度的反省。

※美國政府核心管理能力簡介:

美國聯邦政府人事管理局於1990 年提出「領導效能架構」

(Leadership Effectiveness Framework),以規劃不同層次主管人

員應具備的獨特能力與共同的基本能力。共同的基本能力包括:

口頭溝通、書面溝通、解決問題、領導、人際技能、自我導向、

彈性、果斷力、以及工作所需的技術能力。基層主管人員應該有

管理多元化工作人力、處理衝突、促成團隊建立、影響或協商、

以及人力資源管理等能力。中階主管人員應該培養創造性思考、

規劃與評估能力、顧客導向、內部控制與廉潔的能力、以及財務

管理與技術管理等能力。組織的高階主管人員則應該具備宏觀的

遠見以及對外部環境的洞悉能力。在此架構下,每一層級

的主管人員均以較低層次的能力作為基礎,而後再發展出更高一

層級職務所需的能力,即可避免無法勝任新職,或是擔任較高層

級的職務卻缺乏基層經驗與基本能力的困境。

1998 年起則修正為五大面向並選定所需的二十七項能力如下。

(一)變革領導:1. 創意與革新;2. 持續學習;3. 對外界的覺

知;4. 彈性直

觀;5. 激勵服務;6. 策略性思考;7. 願景領導;8. 壓力克服力。

(二)員工領導:1. 衝突管理;2. 文化覺知;3. 正直與誠實;

4. 團隊建立。

(三)成果導向:1. 責任感;2. 顧客服務;3. 果斷力;4. 企業

精神;5. 問題解決能力;6. 技術專業力。

(四)企業敏銳:1. 財物管理;2. 人力資源管理;3. 科技管理。

(五)聯盟建立與溝通:1. 影響力及談判能力;2. 人際能力;3.口語溝通;4.夥伴關係;5. 政治技能;6. 書面溝通。

※ 我國政府建構管理核心能力簡介

鑒於當前世界各國及企業界愈來愈重視建立核心能力,我國政府

於「挑戰2008:國家發展重點計畫」之「E世代人才培育計畫:

整合政府學習資源」子計畫中,已將核心能力的選定、評鑑及學

習等列為內容。同時,行政院並於2004年1 月5 日核定修正「公

務人員終身學習推動計畫」肆、「實施策略」之四、「建構導引

公務人員有效學習機制」(二)規定,界定公務人員業務推動應

Page 78: 主題:公共管理的基礎理論 - web.thu.edu.twweb.thu.edu.tw/lu.bk/www/news/99-1.pdf · 主題:公共管理的基礎理論 焦點一:試說明Weber對傳統官僚制度的反省。

具備的管理及專業核心能力,擬訂訓練計畫,規劃實施訓練。而

為落實相關工作,行政院人事行政局爰辦理行政院所屬機關核心

能力之選定作業,第一階段係以管理核心能力為規劃重點,經選

定中、高階主管職務各六項管理核心能力,並經行政院於2004 年

3 月18 日函知各機關,另配合研訂「行政院所屬機關中、高階主

管職務管理核心能力評鑑量表」,於同年7 月5 日函送各機關參

考運用,作為評估主管人員訓練發展需求之依據。

調查結果顯示:在高階(第十二職等)主管部分,受重視的管理

核心能力項目有創意型塑願景、策略分析、變革與危機處理、團

隊激勵與領導、跨域協調、績效管理等六項。創意型塑願景、策

略分析、變革與危機處理係屬前瞻規劃項目;團隊激勵與領導、

跨域協調係屬人際互動項目;績效管理則為執行實踐項目。在中

階主管人員部分,受重視的管理核心能力項目有顧客導向服務、

知識管理與運用、流程與時間管理、衝突折衝與溝通、指導與經

驗傳承、目標設定與執行等六項。顧客導向服務、知識管理與運

用、流程與時間管理係屬執行實踐項目;衝突折衝與溝通、指導

與經驗傳承為人際互動相關的項目;目標設定與執行則係前瞻規

劃項目。

焦點八:組織信任—社會資本的觀點【思考方向】

壹、社會資本的核心價值—信賴

一、社會資本的重要特徵:

雖然學者學科背景相異,但是可以知曉社會資本是一種非實體的

資原(resources),能使網絡中的多元參與者相互信賴協澗,以利共

同達成目標與解決問題。並且社會資本具有下列六項重要特徵

(Lesser, 2000 : 7) :(一)社會資本雖然有著不確定的回報,但經由投資可以期待未

來會實現。例如組織可以投資在協商上,以建立非正式網

絡與創造面對面的互動關係,增加信賴程度。

(二)社會資本是具有供給的轉換特性。社會資本被運用在多數

生產過程上。例如,一個組織職能轉換給另一個組織,進

Page 79: 主題:公共管理的基礎理論 - web.thu.edu.twweb.thu.edu.tw/lu.bk/www/news/99-1.pdf · 主題:公共管理的基礎理論 焦點一:試說明Weber對傳統官僚制度的反省。

而達成獲取利益的目標。

(三)類似自然與人力資本,但是不像財政資本;社會資本需要維

持主要的生產力。在社會資本中沒有提供時間、能源或其

他資源。個人間的連結需要長時間的培養,如同鋼鐵所產

生的氧化作用一樣。然而,不完全像人力資本的地方是,

維持社會資本需要至少兩個團體才能成形。

(四)類似人力資本,但是不像自然資本;社會資本無法預測貶值

率。社會資本就像是組織的知識,可促進生產力提升,隨

時間增加而有上升之趨勢。然而,許多組織可能捨棄無關

的技術(例如,大變革的技術使得現今技術淘汰),但是可

能無法預料到社會資本正迅速流失的價值。如公司預期之

解雇發生,則其社會資本的創造價值可能降低。

(五)社會資本是公共財。特別是它不是個人所擁有,而是依附

在所有網絡成員中。其他形式的資本(例如天然、人力與財

政資本)則傾向個人擁有。

(六)社會資本是存在行動者間的關係網絡,不是行動者本身。

社會資本不是空洞的,它是依靠個別行動者互動而創造的

價值。為了適應環境變遷,社會資本可以凝聚集體行動。

貳、組織的信賴投資

組織管理者如何建立信賴關係將是焦點之所在。換句話說,信賴

關係的建立唯有組織管理者發揮「由上而下」(top-down)的積極作

為,才能克盡全功。一般說來信任投資之策略可分為下列四個面

向:

一、個體層次上之信任投資:

在個體層次方面,重點在於人力投資模式,此點意味管理者重視

員工專業技能訓練,以提高人員素質。大部份的組織,人員受教

育的機會很多,並且能提高個體學習的智慧和能力。

二、團隊層次上之信任投資:

就人力資本的投資而言,團隊建立是希冀能遠成「自我管理」的

一連串過程。在這樣的環境下,團隊管理者們往往傾向於提升團

隊的技術發展,特別是在交叉訓練中,需要重視發展團隊的活動

性與對組織生產及業務過程的理解能力。因此,為了達成技術訓

Page 80: 主題:公共管理的基礎理論 - web.thu.edu.twweb.thu.edu.tw/lu.bk/www/news/99-1.pdf · 主題:公共管理的基礎理論 焦點一:試說明Weber對傳統官僚制度的反省。

練,團隊管理者往往通過一系列的活動來增加他們的責任感,培

養自我管理的能力。

三、組織層次上之信任投資:

雖然在個體和團隊層面上的信任投資,累積起來就是對全組織的

信任投資。但在組織層面上明顯需要一種管理能力來打破「合作

之困境」,使合作重新開始。是以,組織層次上的信任投資,不僅

需要在個體和團隊上投資,而且還希望將其影響擴散至整個組織

中。換言之,管理者有責任視自己為「好將軍」(管理者)並成為

「好士兵」(員工)的榜樣。

四、網絡層次上之信任投資:

現今公部門運作已突破傳統疆界之限制,造成公私部門間互動更

加頻繁。此外,公共事務議題日趨複雜化,為了有效解決難以治

理之問題,公私夥伴關係已成為一種互動態樣。因此,公部門必

須對私部門或非營利組織投資信賴,達成互惠之回報。

主題:方案管理

焦點一:績效管理(Performance Management)專題探討1

【思考方向】

壹、前言:

績效管理刻正成為學術界和實務界所正視的課題。「有評量,才有

管理; 沒有評量,就沒有績效」一直被視為管理的格言:唯弔詭的

是,誠如韓第(Charles Handy)所言: 「量得更多容易,量得更好卻

難」。因此,如何設計一個聰明而負責的績效管理制度,乃為大家

一致的努力目標。

貳、績效管理之意涵:

一、績效管理的要素:

績效管理和一般管理方式有別,乃在於特別強調系統的整合,並

著重對組織績效的所有面向進行宏觀的控制、審核、與評估,箇

中的主要因素為:

1 台北大學林鍾沂教授見解。

Page 81: 主題:公共管理的基礎理論 - web.thu.edu.twweb.thu.edu.tw/lu.bk/www/news/99-1.pdf · 主題:公共管理的基礎理論 焦點一:試說明Weber對傳統官僚制度的反省。

1. 詳明清楚而可衡量的組織目標(此即目標管理);2. 系統的使用績效指標或組織績效的衡量標準,以評估組織的產

出;3. 應用個別成員的績效考評,以助益成員間努力的調和,並導向

於組織的目標;4. 使用績效的激勵如績效獎金,以獎酬對組織目標有特別貢獻的

個人成就;5. 將人員和財政的資源分配予以聯結,使其形成年度的管理或預

算循環;6. 就每一規劃循環之結束實施定期考核,俾實現目標並暸解績效

是否比預期或好或壞的理由,且透過反饋的形成以幫助新的循

環的開始。

二、績效管理的聯結架構:

(一)它將單位中不同的管理體系如預算、用人、績效評量、和

個人績效考評體系加以聯結;(二)它將高階的願望和低階的服務傳輸給予接合;(三)它把決策制定的核心與負責執行政策、處理顧客的單位予

以有效的聯結;(四)它透過個人的績效獎金和組織單位之優先順序的轉移,而

把工作的獎酬制度給予結合。

綜言之,若缺乏此一聯結過程,組織就會形成浪費、重疊、和沒

有效率。

三、績效管理的重要性:

在行政學的發展上,傳統公共行政、新公共行政、新公共管理、

黑堡宣言和新公共服務等雖對公共行政的價值與使命各有不同的

界定,卻也未將績效服務棄之不顧。細究原因:(一)績效乃是政府責任的表現:不論是政府被形容為「萬能政

府」或「小而能政府」最終的指南端在於人民對政府施政

表現的感受。換言之,「政府的大小的問題根本不是問題,

真正的問題是政府能否有績效」亦即,組織(包括政府機關)欠缺績效獲績效太差將被視為無能的表現。

(二)績效深具統攝性與變動性:在傳統的公共行政中,績效被認

Page 82: 主題:公共管理的基礎理論 - web.thu.edu.twweb.thu.edu.tw/lu.bk/www/news/99-1.pdf · 主題:公共管理的基礎理論 焦點一:試說明Weber對傳統官僚制度的反省。

為透過管理工具,不折不扣地貫徹施政的目標,這種強調

執行力的貫徹即被認為績效優良。及至新公共管理,認為

政府的功能在於導航而非操槳,於是減低管制、契約外包、

推動民營化、建立公私夥伴關係,遂被頌為政府施政的一

項美德。一方面,政府須專注核心職能的提昇,另一方面,

又得做好「去任務化」的配套,這才算是展現績效的精義。

最後,新公共服務雖然強調政府的功能不在導航而在服

務,若行政能以公共利益為重,傾聽民眾的需求和聲音,

策略思考,民主行動,那麼政府的作為就被視為有績效的。

由此看來,績效的意涵並非一成不變,它宛如變色龍一般,

能夠隨機的調整。

(三)績效是人民評估政府最易識別的服務標的: 一個政府施政

績效的良窳,雖難以鉅細靡遺的完全解釋和評估,唯當百

姓接觸時,卻能得到八九不離十的整體印象,所以在每個

時代,績效管理關係著政府能力的表現,導致其非常在乎

民眾心目中的形象和評價。

參、公部門應用績效管理的難題

一、行政部門所提供的多屬服務,而非產品,這使得續效衡量較

屬不易:

由於服務講究服務熱情、犧牲奉獻、用心體貼、默默付出、

長期關注、一次到位、無關利益等特性,而使得量化的衡量備感

棘手。反之,績效管理為達課責要求,於是稽核的概念從最初的

財務,延伸到非財務方面,並對程序和制度進行鉅細靡遺的審查。

無論績效優劣,都要完全依據被認為十分精準的績效指標來衡量

的。這種被 Michael Power 稱之為「稽核爆炸」(audit explosion)的現象,不但取代或邊緣化了舊有的負責制度,更使得原本重視

大體的服務格調淪為細微的瑣事。

二、公部門經常欠缺較為精確和有效的績效衡量,而產生所謂的

「績效迴避症候群(performance aversion syndrome) :

組織成員在彼此相安無事,無需競爭的組織氣候中,最易令人產

生偏安心態,例如,每年的年終考績以輪流方式,被認為最不會

引起爭議和維持和諧的最佳方式。此種組織成員冀求一團和氣,

Page 83: 主題:公共管理的基礎理論 - web.thu.edu.twweb.thu.edu.tw/lu.bk/www/news/99-1.pdf · 主題:公共管理的基礎理論 焦點一:試說明Weber對傳統官僚制度的反省。

避免「績效傷人」的文化傳統,在今日強化績效管理的潮流下仍

產生一定的宥限作用。昔日戴明曾經指出:「績效評估制度鼓勵了

短期績效,卻大大削弱了長期規劃,引發恐懼,破壞團隊合作,

製造同事間敵對的氣氛和助長辦公室政治」。

三、組織與評估的相互對立:

行政機關的特性,如依法辦事,遵守程序規則之強調,以及

人事上種種的措施,如職位分類制,平等就業機會、權益平等促

進行動(affirmative action)、工作規範、工會主張等均有形或無形

限制了績效之評量,甚至難對績效良窳的員王施予嚴格的獎酬或

制裁,尤其是組織面對多元的服務對象,又在相互衝突的目標中,

進行客觀與公正的績效評量,更會惹來爭議。

四、容易爭功諉過:

績效管理和績效獎酬制度的實務,往往使得每個單位或人員為了

增加自身的績效而爭功諉過的現象,例如員警為了自己的績效而

逕行跨越轄區抓人已是爭奪績效屢見不鮮的事實;再者組織成員

也僅就組織績效所要衡量的相關指標加以重視,而忽略了對無關

衡量之業務本質和能力的重視。亦即員工只做要檢查的工作,而

漠視應該完成的任務。

肆、績效管理的策略地位:

績效管理的有效實施應建構在四個主要層次的前提上:(一)評量工具需被視為可信與有效的:

一般而言,一個良好的評量工具應包括以下幾種特性:

(1) 完整性(completeness) :指標能否適切的反應總目標?例如,投

資報酬是衡量收入的良好指標。而品質、準確性和即時姓,常

為服務衡量的有用指標。

(2) 即時性(timeliness) :當需要衡量時,評量工具能夠快速地被選

取,而不會被限縮在特定的日程或扮演「事後諸葛」(autopsy)。換言之,衡量要能夠靈敏地反映當下的問題,而非僅止於年度

上的績效評量。

(3) 能見性(visibility) :衡量項目是否能讓被評量者公開地追蹤、檢

視。如果評量系統不夠透明,恐怕沒人會曉得何種績效要被考

察、追蹤; 欠缺透明或能見度將使評量系統不易被採納。

Page 84: 主題:公共管理的基礎理論 - web.thu.edu.twweb.thu.edu.tw/lu.bk/www/news/99-1.pdf · 主題:公共管理的基礎理論 焦點一:試說明Weber對傳統官僚制度的反省。

(4) 成本(cost) :如果能夠利用他種目的所蒐集到的資訊,則所花的

費用將會相對便宜。否則,如擬獲得職員的意見反映而自行實

施一年三次的調查,就要花費昂貴的成本,是以一份調查研究

能夠與其他文件交叉連結,便可降低成本。

(5) 可釋性(interpretability) :評量工具應該容易地被理解,而且使

用的資訊容易在組織內部或跨組織間自由流動。假如兩位管理

者同時對同一評量有不同的詮釋,則此一評量指標將會被另一

方視為太過主觀。

(6) 重要性(importance) :評量是否與重要的組織目標形成連結,設

若品質、及時性和準確性看似重要,但若忽略如公正、永續發

展等其他因素,評量恐會流於偏頗。

(7) 時程的平衡性(time balance) :評量系統應能反映長程與短程的

目標,而非只迎合短期目的卻犧牲了長期目標,否則,問題恐

有被短視化之虞。

(8) 動機的平衡性(Motivational balance) :舉例而言,所有的評量系

統必須在競爭活力與合作團隊之間找到平衡。

(二)績效評量應和高績效組織形成密切的連結:

每位經理人都在努力尋求正確的方法來執行評量的工作,就像是

在尋找基督於最後的晚餐上所使用的聖杯一般。例如投資報酬率

就被視為最歷久不衰的評量指標;財務的盈虧也被看成績效的底

線。但是就公部門組織而言,績效管理更應促成高績效組織特性

的展現。因為,組織若能展現卓越、有機的特質,它被期待達成

「良好績效」的可能性,遠比傳統機械的組織來得高。

(三)績效管理能夠反映組織的使命價值:

績效管理的成敗之關鍵,即在於組織是否發展出一個具統合性的

前提,那就是組織的使命價值。組織的使命價值不但可以讓組織

保持持續前進,不致迷失方向,而且在資源配置上亦可向一些說:「不」,專心致志於使命達成,不會發生使命偏離的問題。

(四)組織的績效管理應該簡要而且講究大體:

有了目標和績效評量之後,每個人都會覺得成功的定義變得很具

體明確。但績效管理若變成為「鉅細靡遺」的精細管理,則不但

招來反感,反而未見成功。是以如何將績效管理設計為適中而大

Page 85: 主題:公共管理的基礎理論 - web.thu.edu.twweb.thu.edu.tw/lu.bk/www/news/99-1.pdf · 主題:公共管理的基礎理論 焦點一:試說明Weber對傳統官僚制度的反省。

體的規範,便成為它運作完善與否的精義。究言之,也許我們為

自己構築的嚴格管理制度,其實只會傷害信任,而不是維護信任。

伍、績效管理的相關配套

一、策略管理(strategic management):1. 何謂策略管理?簡言之,它可被視為組織資源分配的重點式取

捨,以形成製造差異化、與眾不同的動態過程。是以,只要科

技不斷的更新,市場不斷的變化和環境不斷的變遷,組織就須

與競爭對手進行一場策略遊戲。最後的勝出者必定要使出競爭

者無法模仿的策略本領,創造出獨特的價值。

2. 簡言之,組織如何將資源做成有效的配置,以開創獨特的績效

成果,並向有些不甚相關的任務說「不」將是應用策略管理以

使組織績效勝出的基本關鍵。這也就是 Magretta & Stone 一再

指出策略與績效之關係乃為同一錢幣之兩面;與眾不同,才能

優於競爭者; 策略是一種讓績效更優異的邏輯。

二、全面品質管理:

在一九五○年代,當時美國管理先知戴明即利用統計管制方法觀

察流程運作,而逐漸推展其全面品質管理運動。所謂全面品質管

理顧名思義,即在財貨和服務的生產上對品質從事全面性和持續

性的關注。其中「全面」是指每一作業部門均應戮力追求產品品

質;「品質」意指迎合甚或超越顧客的期待;「管理」則指發展和

持續組織的能力去穩定地改進品質。

三、工作團隊:

為何團隊應用成為管理學的熱門話題?有幾個因素值得重視:1. 首先,有證據顯示當需要多樣的技巧、判斷與經驗才能完成任

務時,團隊的表現通常會比個人表現來的更好;2. 想把組織改造的更有競爭力、提生效能效率時,通常採取團隊

的方式,員工的才華才能做到更妥善的利用;3. 管理者發現團隊遠比傳統的部門或其他型態的固定工作團

體,更具環境的調適力和反應能力,這是因為團隊具有迅速組

合、展開工作、調整重心和解組的能力;4. 團隊具有工作激勵的效果,它提供員工一個可以參與決策的運

作管道,因而可提升工作動機並促成組織民主化。

Page 86: 主題:公共管理的基礎理論 - web.thu.edu.twweb.thu.edu.tw/lu.bk/www/news/99-1.pdf · 主題:公共管理的基礎理論 焦點一:試說明Weber對傳統官僚制度的反省。

四、標竿管理:

謂標竿學習,是指就工作流程的運作績效來典競爭對手從事系統

性的比較或和某種產類的最佳典範作一比較,而形成「見賢思齊」

的作用。它可分屬內部和外部的比較。例如,它會對組織內部績

效卓著的單位之作法加以參考,也會和外部的卓越組織加以效

法。職是之故,標竿學習通常都以「最佳實務」(the best practice)連結在一起。Peter F. Drucker 為此以「創意性模仿」做了最佳詮

釋。它說明了某種具有模仿本質的策略,當採用創意性模仿之後,

會比最初從事該項創新的人還要了解該項創新。

陸、結語

績效衡量現在正是公共管理下的熱門課題,不論何種國家型態、

公私部門幾乎都在談論典實施著績效管理。唯績效管理猶如哈佛

大學教授海菲斯(Ronald Heifetz)所描述的是種「走在剃刀邊緣」,

當使用時不可不慎。有鑑於英國目前推動的績效管理制度,業已

使行政官僚作業負擔加重,苛求極高,而且「稽核爆炸」與「比

照辦理」等方式,更使組織陷入癱瘓的困窘中,所以 0' Neill 認為

應建立「聰明而負責的續效管理制度」,而非顯微鏡下的精細管

理,這才能贏得民眾的信任。

焦點二:績效管理與組織學習(世新徐仁輝教授見解)

【思考方向】

壹、前言

一、 如何測量與改進政府績效的努力,在 1950 年代績效預算

制度採行時即已開始,經過半個世紀,到最近二十餘年更為

積極。原因是世界主要工業國家皆面臨經濟衰退、財政困窘、

人民拒納更多稅、以及要求政府服務更好的情況,因此只有

努力提升政府績效,才能讓政府課責。

二、 各 國 採 行 的 績 效 基 礎 預 算 (Performance-basedBudgeting)、績效管理(Performance management)或結果導向管

理(Managing for Results)等制度就是試圖改進政府績效。不

過,提升績效的努力是需要長期與持續的,例如需要時間來

訂定績效指標,並需要更多的時間來讓行政官僚與民意代表

Page 87: 主題:公共管理的基礎理論 - web.thu.edu.twweb.thu.edu.tw/lu.bk/www/news/99-1.pdf · 主題:公共管理的基礎理論 焦點一:試說明Weber對傳統官僚制度的反省。

熟悉如何使用這些績效資訊。又由於政府機關目標與公共服

務本質特徵,各國實施績效管理制度皆面臨一些困難;公務

員也仍然繼續受到原有價值與文化的影響。因此績效管理制

度的推動有賴於組織學習機制的建立。

貳、績效管理的意義與議題

一、績效管理的重要性

政府再造強調顧客導向型的政府,必須讓政府施政能適時滿

足民眾的需要,因此在施政績效評估方面亦由過去重視「投

入」(input)的控制,轉變為強調「成果」(result)的測量,例

如對於計畫的產出(output)、結果(outcome)、服務品質、民眾

滿意度等。藉由以成果為導向的績效管理,將使各機關之施

政或計畫執行的成果成為客觀的課責標的,有助於檢測政府

對人民需求的回應能力。換句話說,政府是否表現良好是看

公民可以從政府的支出計畫中獲得什麼,判斷政府的績效不

是看其企圖而是看其成果。

(一)績效管理

經濟合作暨發展組織(OECD)指出績效管理包括公司管理、績

效資訊、評估、績效監控、評量、與績效報告等。在新績效趨勢

環境下,管理的流程嚴格定義為規劃績效目標與目的、彈性管理

以達成目標、績效測量與報告、並將績效資訊用於規劃資金、設

計、營運、獎賞與處罰等方面(OECD,1995)。美國布希(Bush)總統自 2001 年上任後大力推動「總統管理議

題」(President’s Management Agenda, PMA),其包含五項重點議

題:(1)人力資本之策略管理,(2)具競爭力的採購,(3)提升財務表

現,(4)強化電子化政府,以及(5)預算與績效之整合。這五項議題

目的在於提升各機關計畫的績效資訊品質,促使國會與行政部門

於制定管理及資源分配之決策時,能考量並結合績效資訊,以實

質提升政府整體的施政績效,向納稅人證明政府運作之效率及成

果。

(二)績效基礎預算

傳統預算制度在強調預算的控制功能,因此注重資源如何配

Page 88: 主題:公共管理的基礎理論 - web.thu.edu.twweb.thu.edu.tw/lu.bk/www/news/99-1.pdf · 主題:公共管理的基礎理論 焦點一:試說明Weber對傳統官僚制度的反省。

置在不同的支出項目上,要求預算作業的合法性,對納稅義務人

負責。績效基礎預算制度強調的是計畫產出與結果,因此更注重

預算制度的規劃與管理功能,追求資源在不同的計畫間作配置,

以提升資源的使用效率。

1980 年代澳洲與紐西蘭是最早引進績效基礎預算制度的國

家,一開始是將預算按產出編製,目前更進一步按結果編製。1990年代以後多數 OECD 國家也陸續採行績效基礎預算制度,試圖將

績效目標與預算過程結合。

(三)績效管理的目的

1. 政府引進績效管理與績效基礎預算的主要目的可以歸納

有四項:改進效率、改進預算決策、改進財務透明度與課

責、節約支出(OECD,2004)。如美國的「政府績效與成果

法」 (Government Performance and Results Act,簡稱

GPRA),英國的「財務管理法」(Financial ManagementInitiative),澳洲的「計畫管理與預算」 (ProgrammeManagement and Budgeting),紐西蘭的「財政法」(PublicFinance Act)等,這些立法的目的皆在利用績效資訊改進

預算決策,以提升行政效率、透明度與課責。

2. 績效資訊對於政府是非常重要的,它可以幫忙管理與控制

公共服務,也可以讓民意機關監督行政官僚是否有效率使

用預算,它也滿足了民眾對於政府課責的要求,因此政府

有義務公開績效資訊並對其負責。施能傑(1999)指出政府

實施績效管理的最大意義在於改變行政文化,讓公務員以

正面積極的態度去面對績效管理的價值,雖然績效測量可

能帶來爭議性,仍會更負責的去有效率使用資源。

二、績效管理的問題

一般政府績效管理常見問題,可以歸納有如下(徐仁輝,

2004):(一)目標設定一致問題:

政府機關目標與私人企業的目標有很大的不同,私人企業以

追求利潤為主,政府機關的目標經常是無法量化的。加上政府機

關的利害關係人有許多,正如同多重委託人般,彼此的目標可能

Page 89: 主題:公共管理的基礎理論 - web.thu.edu.twweb.thu.edu.tw/lu.bk/www/news/99-1.pdf · 主題:公共管理的基礎理論 焦點一:試說明Weber對傳統官僚制度的反省。

是衝突的,因此如何透過政治妥協,取得共識並非易事。而目標

的確定,卻是績效評估的先決條件,如無法確定目標,即無法訂

定績效指標。

(二)績效指標量化問題:

績效指標無法量化是績效評估的老問題,對於無法用金錢價

值衡量的項目,如何予以評估?如何具體化?對於那些不知如何

量化的重要計畫,如果勉強為之,是否造成衡量結果的嚴重扭曲?

(三)資訊的完整性問題:

計畫執行的資訊可能是片面的與泛政治化的,而蒐集資訊經

常係為不同目的,這使得績效評估無法正確理性且客觀地進行。

又蒐集績效資訊來源的不同會產生績效衡量基礎不一致的現象。

(四)評估結果的回饋問題:

評估結果是否將作為計畫改進的參考、以及未來計畫取捨與

預算編列之考量因素。

三、績效基礎預算之問題

績效評估與預算作業的結合有其限制,原因有以下三項:

(一)績效與財務資料的技術性限制

依 Pollitt(1999)的分析,直接將績效評估資訊用於預算目的有

技術性困難,例如只有部分施政計畫,特別是可以提供市場價格

的財貨或服務計畫,較容易進行績效測量,才可以獲得較可信的

單位成本。又許多公部門活動,例如研究、濟貧、環保等,他們

的結果需要在經過一段時間的未來才能顯現,而非在同一會計年

度內可以觀測。

(二)績效資料本質上無法為預算決策提供充分的基礎

績效指標代表計畫的整體表現,欲將其做為決策依據,必須

對於為何產生此績效結果的原因予以檢討。例如計畫結果績效不

佳的原因為何?係因為計畫不具體呢?還是管理不良?顧客群不

對?地點不對?外在不利的大環境造成?人員配置不當?預算不

足?績效測量不易?或其他因素?首先必須探究其原因,如果是

預算不足,增加其預算反而可以導致其績效提升。反之如果是績

效評估良好,代表其運作適當,增加其預算可能所帶來的額外利

Page 90: 主題:公共管理的基礎理論 - web.thu.edu.twweb.thu.edu.tw/lu.bk/www/news/99-1.pdf · 主題:公共管理的基礎理論 焦點一:試說明Weber對傳統官僚制度的反省。

益有限。總之不宜將績效資訊與預算作機械式的聯結,而必須探

討績效未如預期的原因,再作預算決策。

(三)對未來方向作決策僅依賴過去的績效基礎係不適當的

績效資訊指的是過去的表現,而預算決策則是針對未來擬進

行的活動作決策,以過去的資訊作為決定未來方向的唯一基礎是

否適當?事實上,績效測量應該只是政策與資源決策的考量因素

之一。不宜完全以過去的績效表現決定未來的政策之優先順序,

例如決策可能需要考量的是利害關係人的偏好、或是有無適當的

替代方案等(Perrin, 2003)。四、我國績效管理問題:

1.策略績效目標與衡量指標連結仍待加強

2.績效目標與衡量指標的挑戰度不足

3.施政計畫與預算編列尚難結合

4.施政管理及評估資訊建立不夠完整

5.評估專責單位及評估人員專業能力仍待持續提升

6.人力及經費指標應著重組織績效提升

7.機關績效與激勵制度仍待持續結合。

參、績效管理與組織學習之間的關係

組織學習理論中有所謂的單圈學習(single-loop learning)與雙

圈學習(double-loop learning),前者係在現行任務、目標與策略

下,尋求績效的改善,後者則針對目標、策略與政策予以重新評

估。績效管理的學習應屬於雙圈學習,才能讓組織績效的提升獲

得成功;同時這學習不只是一種結構性的學習機制,而應是文化

性的學習。分述如下:

一、績效管理與員工的內在動機

(一)人非機械,正式的管理與控制制度只有在要求公

務員執行特定行為時有效,好的管理應該是以最

大化內在動機,與需要最少的正式控制為目標。

事實上正式的管理制度是反生產力的,因為它們

降低了內在動機;管理者必須認清正式制度對於

人類行為的影響是有其限制的。

Page 91: 主題:公共管理的基礎理論 - web.thu.edu.twweb.thu.edu.tw/lu.bk/www/news/99-1.pdf · 主題:公共管理的基礎理論 焦點一:試說明Weber對傳統官僚制度的反省。

(二)激勵公務員注重績效可以利用制度設計引進外在

壓力,如利用會計與審計系統,加強計畫預算績

效的考核。再如績效評估制度的改進,利用問卷

或其他社會科學工具對政策的影響進行衡量。至

於引進競爭機制以加強員工改善效率的壓力也是

一種方式,如將特定服務外包,或重新檢討政府

角色,將民眾視為顧客,績效測量以顧客的滿意

度為基礎。

二、績效管理與組織學習

績效管理本身就是一個學習過程,就組織策略管理而言,組

織決策者可以透過績效資訊,進行策略評估與選擇,學習制定策

略目標與計畫,進而改善組織績效。就個人層面言,績效待遇制

度對員工產生制約學習的增強行為,即工作績效良好者可以獲得

更好的績效獎金報酬。

三、 文化性的學習

組織學習理論文獻強調組織文化對學習的重要性(Schein,1992),學習是基於共享的經驗、規範與了解,因此學習的方式包括高度

員工授權、參與、自由裁量,透過這種文化性的學習強調共享的

功能性規範。

(一)結構性與文化性學習

結構性的學習方式則係指對於結構與程序性安排建立制度,

讓組織可以系統的收集分析儲存傳播與使用資訊,有助組織效

能。結構性的學習較吸引改革者,因為可以透過命令將組織的結

構與程序方面予以改變,結構性學習也符合學習的理性分析理

論,強調收集儲存與分配資料。

(二)學習論壇

1. 大都數政府在實施績效管理時皆發展策略規劃與績

效測量,並產生大量績效資訊,問題出在不知如何使

用資料。獲得很多資料卻不會使用的原因在於缺乏將

資料檢查並解讀的程序,即所謂的學習論壇(learningforums)。學習論壇指的是鼓勵參與者詳細檢查資

訊,考慮其影響,並決定它如何影響未來行動。如果

Page 92: 主題:公共管理的基礎理論 - web.thu.edu.twweb.thu.edu.tw/lu.bk/www/news/99-1.pdf · 主題:公共管理的基礎理論 焦點一:試說明Weber對傳統官僚制度的反省。

只是單純的提供數量資訊,這種程序是不會發生的,

例如多數管理者花時間去與人們互動收集口頭資

料,但卻不去思考這些量化資料。

2. 學習論壇的程序是批判性的,可以藉學習調整組織行

為。組織成員決定參與那些程序,取決於什麼是他們

認為對組織最適當的,若組織文化勾勒出資料的考量

程序是種適當的組織行為,他們就會去學習分析那些

資訊。在學習過程中成員的價值評估與解釋資訊會受

到文化偏見的影響,結構性學習就是忽視了文化面,

一般績效管理也是只注重法令與系統。所以美國國會

政府會計局(GAO,2003)最近公佈績效管理的成功,

必須產生一種認同自我檢查與學習的評估文化。

肆、組織學習機制的建構

績效管理制度的設計應注重透過學習論壇進行績效資訊的討論,

而非只是注重資訊的產生,如果沒有這種學習論壇,績效管理制

度是不完備的。從組織學習的觀點言,大多數政府績效管理皆偏

重於績效資訊的產生與傳遞,而忽視了建立學習機制來評估與使

用這些資訊。

一、 具體化學習論壇

(一)學習論壇強調學習是件經常事務,透過論壇產生對話,對

話是學習的前導。Senge(1990)指出對話就是允許參與者表

達個人意見,並產生共識。他建議成功的對話應包括假設

的中止、建構對話的基本規則、成員的積極參與、平等對

待參與者、成員願意提出組織急迫議題等。Argyris 與

Schon(1996)認為使用對抗性的議題資料會導致防衛性反

應而非學習,因此應使用公正與開放非對抗性的資料。

(二)行政院人事行政局(2004)所擬定的未來精進績效獎金暨績

效管理制度的策略性作為之一為:「強化績效管理之內化與

學習,協助各機關塑造績效導向的組織文化」,其具體做法

是辦理績效獎金種籽師資研習營與開設績效管理相關課

程。這些做法目的即在增進各機關首長與一般同仁對於績

效管理與績效獎金制度的認識,進而促成學習論壇與對話

Page 93: 主題:公共管理的基礎理論 - web.thu.edu.twweb.thu.edu.tw/lu.bk/www/news/99-1.pdf · 主題:公共管理的基礎理論 焦點一:試說明Weber對傳統官僚制度的反省。

的產生。透過學習論壇可以對於績效指標的選擇與資訊的

收集,產生集思廣益功效,建立更完善的制度與作業方式。

二、 將文化融入學習

(一)績效管理不只是單純的產生一大堆績效報告資料,而不去

分析使用與從中學習。學習過程不能擺脫組織文化扮演的

角色,如何將傳統依法行事的官僚文化,轉變為目標或績

效導向的組織文化並非易事。充分的授權、鼓勵溝通、移

除人事會計的限制、強調任務導向團隊等方式,皆是強化

員工追求績效的內部動機與認同感的方法。只有建立員工

自我評估與檢討的學習文化,才能透過雙圈學習機制,讓

績效管理產生最大效益。

(二)在追求機關預算最大化的官僚文化下,機關首長關心的是

如何爭取最大預算,並執行預算;注重的是預算投入面控

制,而非產出或結果面。只有在績效導向的行政文化下,

機關首長爭取預算係為了追求機關績效的實現,因此會以

績效資訊作為預算編製的依據,認真實現績效基礎預算。

三、 機構為中心的組織學習

(一)如果績效管理與績效評估制度的規範,能因應機關的屬性

不同而有差異,即可建立以機關為中心的組織學習;同時

組織文化的改革大部分也是在機關層級形成的。各機關自

然會將實施績效管理視為機關本身的改革機會,鼓勵組織

成員踴躍學習與對話,悉心檢討組織面臨的重大議題與現

行目標,利用績效資訊來重新策劃目標、策略與計畫;同

時利用績效資訊建立機關自己的激勵與回饋制度。

(二)以機關為中心的組織學習可以讓各個機關透過不斷的自我

對話,發展出衡量該機關計劃產出或結果的具特色之績效

指標,不僅有利於績效評量作業,克服「評估資訊不完整」

與「績效指標待提升」等問題;更可以因有較具體的績效

指標,而與預算的編列密切結合,貫徹績效基礎預算的理

念。

Page 94: 主題:公共管理的基礎理論 - web.thu.edu.twweb.thu.edu.tw/lu.bk/www/news/99-1.pdf · 主題:公共管理的基礎理論 焦點一:試說明Weber對傳統官僚制度的反省。

焦點三:公部門組織長期以來均使用企業管理的方法或技術,

以期改善組織運作的效率。近年來,更引進許多企業管理大師的

概念或技術,並應用在公部門的管理當中。試就所悉,舉出一個

已被廣泛運用的新管理方式,並說明其運用的方式以及有何效

能?

【思考方向】(二)標竿管理(benchmarking)1. 定義:

標竿管理就是將本企業各項活動與從事該項活動最佳者進行比

較,從而提出行動方法,以彌補自身的不足。換言之,即是將本

企業經營的各方面狀況和環節與競爭對手或行業內外一流的企業

進行對照分析的過程,是一種評價自身企業和研究其他組織的手

段,是將外部企業的持久業績作為自身企業的內部發展目標並將

外界的最佳做法移植到本企業的經營環節中去的一種方法。實施

標竿管理的公司必須不斷對競爭對手或一流企業的產品、服務、

經營業績等進行評價來發現優勢和不足。

2. 分類:

可以據所針對的企業運作不同層面將標竿管理分為三類,即戰略

層的標竿、操作層的標竿和管理層的標竿。

(1)戰略層的標竿:是將本公司的戰略和對照公司的戰略進行比

較,找出成功戰略中的關鍵因素。

(2)操作層的標竿:主要集中在比較成本和產品的差異性,重點

是功能分析,一般與競爭性成本和競爭性差異有關。

(3)管理層的標竿:涉及到分析企業的支撐功能,具體指人力資

源管理、營銷規劃、管理信息系統等(MIS)。其特點是較難用

定量指標來衡量。

3.效用:

(1)做競爭對手的標竿,有助於確定和比較競爭對手經營戰略的組

成要素。

(2)通過對行業內外一流企業的標竿,可以從任何行業中最佳的企

業、公司那裡得到有價值的情報,用於改進本企業的內部經營,

建立起相應的超越目標。

Page 95: 主題:公共管理的基礎理論 - web.thu.edu.twweb.thu.edu.tw/lu.bk/www/news/99-1.pdf · 主題:公共管理的基礎理論 焦點一:試說明Weber對傳統官僚制度的反省。

(3)作跨行業的技術性的標竿,有助於技術和工藝方面的跨行業滲

透。

(4)通過對競爭對手的標竿分析,與對客戶的需求作對比分析,可

發現本公司的不足,從而將市場、競爭力和目標的設定結合在一

起。

(5)通過對競爭對手的標竿比較,可進一步確定企業的競爭力、競

爭情報、競爭決策及其相互關係,作為進行研究對比的三大基點

(三)平衡計分卡(The Balanced ScoreCard,簡稱 BSC)1. 起源:

是績效管理中的一種新思路,適用於對部門的團隊考核。在 20世紀 90 年代初由哈佛商學院的羅伯特·卡普蘭(Robert Kaplan)和諾朗諾頓研究所所長、美國復興全球戰略集團創始人兼總裁戴維·諾頓(David Norton)發展出的一種全新的組織績效管理方法。平衡

計分卡自創立以來,在國際上,特別是在美國和歐洲,很快引起

了理論界和客戶界的濃厚興趣與反響。

平衡計分卡被《哈佛商業評論》評為 75 年來最具影響力的

管理學,它打破了傳統的單一使用財務指標衡量業績的方法。而

是在財務指標的基礎上加入了未來驅動因素,即客戶因素、內部

經營管理過程和員工的學習成長。

2. 意涵:

BSC是一套從四個方面對公司戰略管理的績效進行財務與非財務

綜合評價的評分卡片,不僅能有效克服傳統的財務評估方法的滯

後性、偏重短期利益和內部利益以及忽視無形資產收益等諸多缺

陷,而且是一個科學的集公司戰略管理控制與戰略管理的績效評

估於一體的管理系統,其基本原理和流程簡述如下:

(1)以組織的共同願景與戰略為內核,運用綜合與平衡的哲學思

想,依據組織結構,將公司的願景與戰略轉化為下屬各責任部門

(如各事業部 )在財務 (Financial)、顧客 (Customer)、內部流程

(Internal Processes)、創新與學習(1nnovation&Learning)等四個方面

的系列具體目標(即成功的因素),並設置相應的四張計分卡。

(2)依據各責任部門分別在財務、顧客、內部流程、創新與學習

等四種計量可具體操作的目標,設置——對應的績效評價指標體

Page 96: 主題:公共管理的基礎理論 - web.thu.edu.twweb.thu.edu.tw/lu.bk/www/news/99-1.pdf · 主題:公共管理的基礎理論 焦點一:試說明Weber對傳統官僚制度的反省。

系,這些指標不僅與公司戰略目標高度相關,而且是以先行

(Leading)與滯後(Lagging)兩種形式,同時兼顧和平衡公司長期和

短期目標、內部與外部利益,綜合反映戰略管理績效的財務與非

財務信息。

(3)由各主管部門與責任部門共同商定各項指標的具體評分規

則。一般是將各項指標的預算值與實際值進行比較,對應不同範

圍的差異率,設定不同的評分值。以綜合評分的形式,定期(通常

是一個季度)考核各責任部門在財務、顧客、內部流程、創新與學

習等四個方面的目標執行情況,及時反饋,適時調整戰略偏差,

或修正原定目標和評價指標,確保公司戰略得以順利與正確地實

行。

3. 效用:

平衡積分卡不僅是一種管理手段,也體現了一種管理思想,就是:

(1)只有量化的指標才是可以考核的;必須將要考核的指標進行

量化。

(2)組織願景的達成要考核多方面的指標,不僅是財務要素,還

應包括客戶、業務流程、學習與成長。自平衡計分卡方法提出之

後,其對企業全方位的考核及關注企業長遠發展的觀念受到學術

界與企業界的充分重視,許多企業嘗試引入平衡計分卡作為企業

管理的工具。

進言之,實施平衡計分卡的管理方法主要有以下優點:

(1)克服財務評估方法的短期行為 。

(2)使整個組織行動一致,服務於戰略目標 。

(3)能有效地將組織的戰略轉化為組織各層的績效指標和行

動 。

(4)有助於各級員工對組織目標和戰略的溝通和理解 。

(5)利於組織和員工的學習成長和核心能力的培養 。

(6)實現組織長遠發展 。

(7)通過實施 BSC,提高組織整體管理水平。

焦點四:何謂民營化?類型?有何限制?

【思考方向】

Page 97: 主題:公共管理的基礎理論 - web.thu.edu.twweb.thu.edu.tw/lu.bk/www/news/99-1.pdf · 主題:公共管理的基礎理論 焦點一:試說明Weber對傳統官僚制度的反省。

一、民營化的意義

(一)「民營化」一詞最早出現在1969年杜拉克(Peter Drucker)所撰《斷續的年代》一書,《韋氏英文辭典》第九版,將民營化

定義為:公共部門私有化的動作,係指公營事業之所有權或控制

權,由政府部門移轉到私人部門的一種過程。

(二)薩維斯(Savas)認為,民營化是指針對政府部門所擁有的資

產及資產經營的相關活動,逐步降低政府的影響力,增加私人影

響力的一種過程;代表政府重新思考其角色定位,以反應社會需求

的一種努力。

(三)廣義的民營化:係指政府部門降低其對國民經營干預的所

有活動,包括政府將國營事業的經營權或所有權,部分或全部的

轉移給民間,透過市場機能以改善經營體質、提高產質,使企業

發揮其最高的效率。而狹義的民營化:係指「公營型態的解除」

(divestiture),將公營事業的部分股權或資產出售給民間,政府將

收入回繳國庫或另行運用,使國營事業的所有權得以順利移轉。

二、民營化的類型

(一) 撤資(disvestment):將公營事業的資產移轉給民間,可

透過出售、無償移轉、清理結算等策略來達成。

(二) 委託(delegation):政府將部分或全部公共財貨勞務的生產

活動,委由民間辦理,而政府擔負起監督的責任。其委託方式包

括簽約外包、給予特許權、財務補助、發行抵用券、售予強制權

等。

(三) 替代(displacement):民間私人部門取代原由政府所提供的

各項生產或服務活動。其替代情形可區分為「政府功能不足的替

代」、「政府全盤撤離的替代」、「政府為解除管制所進行的替

代」。

三、民營化的限制

(一) 主權與正當性的問題:如外交、法律、司法審判、稅賦徵

收等不適合民營化。

(二) 攸關國民生計之事務:如都會交通、公共醫療、兒童及老

人照護都市、最低生活水平維持等事物需慎思民營化的適切性。

(三) 公共責任的追究:民營化造成公、私部門界限模糊,相對

Page 98: 主題:公共管理的基礎理論 - web.thu.edu.twweb.thu.edu.tw/lu.bk/www/news/99-1.pdf · 主題:公共管理的基礎理論 焦點一:試說明Weber對傳統官僚制度的反省。

造成公共責任模糊及課責的困難。所以,政府如何透過法制化來

規範民間「準政府」的義務及責任,成為限制之一。

焦點五:試說明公共管理的三 P五 D 原則(府際治理的原則)?

【答題方向】

公共管理的新經營理念就是活化參與(Participation)、運用民營

化( Privatization)及營造協力(Paltnerships)三 P 關係的重新建

立。而三 P 的基礎就是五 D :移轉(Devolution)、民主化

( Democratization ) 、分權化( Decentralization ). 解除管制

(Deregulation)、發展(Development),有了五 D 基礎工程,三 P

才能啟動發酵。三 P是「三位一體」同一核心但分三個面向

切入,而用之於實務案例時,卻可能是三 P行動原則整體意

涵的發揚。

(一)三 P的公共管理行動原則:

1.活化參與(Participation) :

從公共行政和公共管理的發展而言,「公民參與」(Citizen

participation) 及 公私部 門 的「 合 作生 產」 (Public/Private

coproduction)的觀念及作法在先進國家已被廣泛運用。。私部

門及公民透過民主政治所保障的機會提供知識及情感參與公

共問題的「集體解決」(Collective problem-solving)。「公民參與」

及「合作生產」的作法,不僅為公共事務的管理提供一種解

決問題的方式,同時更是激勵參與者對「公民資格」的驕傲。

2.運用民營化(Privatization) :

民營化亦可視為官僚規模精簡(Downsizing bureaucracy),引進

市場機能(Market mechanism),把更多社會資源放在民間: 民

間可以在市場機制中發揮創意與活力。

3.營造協力(Partnership) :

現代的公共管理相當注重組織內部的社會建造(Social

construction in organizations),使公共組織更具與外部環境調適

和結合的親和力。相同的,現代化私部門的經營亦必須注重

與民眾和政府互動,即所謂的私部門運作的「雙重公共性」

( Duality of publicness)。亦即公部門和私部門共同學習到在公

Page 99: 主題:公共管理的基礎理論 - web.thu.edu.twweb.thu.edu.tw/lu.bk/www/news/99-1.pdf · 主題:公共管理的基礎理論 焦點一:試說明Weber對傳統官僚制度的反省。

共服務的提供方面,經營者可能不是主要演員,高度支持的

民眾和大家願意協力合作的精神和行動格外顯得重要。

(二)三 P的基石:五 D

1. 移轉(Devolution) :

係指生產功能的轉移及社會責任的共負,生產角色的分享,

將原本政府統籌的公共業務,逐步釋出,亦可視為私部門社

會責任的承受。

2. 民主化( Democratization) :

透過法治文明及責任制度的營造使民間公民力、社會力及人

文力釋放出來,民間社會力量民主理念之深化而成為強而有

力的資源,其對社會整體發展、國家競爭力多有助益。

3. 分權化( Decentralization) :

中央對地方的釋權,增加地方自主性,讓地方政府能來富有

彈性、靈活運用,而不畏懼與私部門接觸往來,而是讓基層

政府有快速回應多元社會意見的能力,同時解構多元的社會

實情。

4. 解除管制(Deregulation) :

降低政府干預,使政府機關的活動受市場機能的規範;或是減

少政府介入,放鬆政府對企業不必要的控制。。

5. 發展(Development) :

發展意涵包括三個層次,一為基本需求的滿足,包括物質的

需求;社會需求,以及精神與道德的需求。二為選擇的自由,

確保各種消費和服務的自由選擇權,消除外在的不合理限

制。三為創造一個有益於民眾提高自尊和自重的環境,包括

各種制度和系統的建立。

焦點六:知識型政府的建構。

【思考方向】

為因應動態、複雜與多元的知識經濟時代,建構知識型政府是最

可能強化政策運作、改善政府施政能力的關鍵。知識型政府的意

涵是透過推動政府機關內部的知識管理與研發創新,型塑新的行

政或組織文化,並創新既有的政府組織結構、決策制定、溝通協

Page 100: 主題:公共管理的基礎理論 - web.thu.edu.twweb.thu.edu.tw/lu.bk/www/news/99-1.pdf · 主題:公共管理的基礎理論 焦點一:試說明Weber對傳統官僚制度的反省。

調、服務方式、法規制度以及人力資源應用等模式,進而建立以

知識為核心價值的組織架構,運用政府的知識、智慧及創意資本,

提供智慧型服務,提升政府的策略規劃能力與豐富質優的公共價

值,助益於提升國家競爭力。知識型政府的特質大致可歸納為學

習型組織、知識管理及研發創新等三大構面。(一)學習型組織

即是將「學習」的動機、成效,應用在「組織」上,使組織發揮

最大的功能,目前各機關加強終身學習認證為其主要活動之一。

(二)知識管理,即是掌握隱性知識與顯性知識的轉化與運用,

以擴大組織的知識擴散與分享,並為政府與社會各界擴大接觸與

加強互動的絕佳方式。

(三)研發創新在高度競爭的知識經濟時代,是獲取競爭優勢的

最大泉源。又政府近些年來在推動技術創新、產學研發合作專利

權數提高與購買先進技術產品等有相當進展。

知識型政府在全世界仍處萌芽期,尚未有大量的成功經驗構建定

型的發展模式。以下數項未來可能的發展方向,可供參照:

(一)國家基礎建設上從NII 到KIIKII建設的重點包括:促進知識、智慧與創意自由跨界流通的知識

平台、激發知識創造、轉換、組裝、整合、保護與利用智慧資本

的國家研發創新體系建立、推動多元價值的社會發展、建立知識

分享與創意的社會架構與文化、智慧資本、社會資本與創意資本

的累積、政府與企業及公民社會的知識價值建立等有關創造無形

知識資產高價化的基礎建設。

(二)政府角色職能─從「知識機器」到「知識機場」

政府的角色職能將從提供知識經濟發展動力,供應知識原料,並

且管理知識流量的一部龐大的、複雜的「知識機器」,逐漸蛻變

為一座能夠創造吸引國內及全球人才、智慧、創意在台灣群聚匯

集自由流通、起降的「知識機場」或創意的「知識花園」。此涉

及政府角色之轉變,其動能來自高效能、高智慧的文官群。

(三)政府的任務─從公共事務管理到公共價值創造

知識型政府的主要任務將從公共管理轉型為利用知識創新,為社

會創造公共價值的最大化─社會的互信、政策的創新、智慧型的

服務、知識導向的決策,洞燭機先的策略能力及時間價值的創造

Page 101: 主題:公共管理的基礎理論 - web.thu.edu.twweb.thu.edu.tw/lu.bk/www/news/99-1.pdf · 主題:公共管理的基礎理論 焦點一:試說明Weber對傳統官僚制度的反省。

等公共價值。但其核心不外要知「知機而發,不可失機而悔」,

達到窮變通久的境域。

(四)政府人力運用─從人事管理到智力管理

建立人員跨界流通變動所需的終身學習、多重專長轉換、創新核

心能力培育、知識創造分享誘因等助益智力創造的環境,讓每一

位公務人員皆蛻變為具備「3Q」(IQ、EQ與CQ,C 代表creativity)的新世代知識工作者,讓渠等都能盡情的發揮創意,貢獻知識生

產力與創造力。甚至「4Q」或「5Q」,不一而足。所以,公務人

員不但要提昇專業能力或培養第二、第三專長外,亦需追求才德

兼備與處事圓融,方能因應時需。

(五)政府組織運作─從階層組織到「知識化組織」

21 世紀的政府組織型態,誠如杜拉克(Peter Drucker)所言,仍然

是一個組織分層、階級嚴明的制式化的組織,但將是澈底的「知

識化的組織」,是一個知識社群網路林立、管理階級角色重整、

知識專業人員匯集、知識與創意跨界自由流通的組織,亦是以知

識為核心的知識導向型或知識密集型政府組織。所以,各社群之

聯結、協力支援與調控及激勵,是必要建立的機制。

(六)從知識管理到「知識治理」

透過知識管理活動、開放性知識交換平台的建立、知識社群的建

立、擴大權力的下授與民眾的參與公共政策研議、行政程序的公

開與流程透明化等,將有助於「知識治理」的進一步體現。當然,

治理在於如何促使決策更接近民意或政府效能目標之達成。要

之,如何由國家上層理念發展方向與具體目標,落實融入機關各

層級每一位成員的腦中與行為運作之中,組織成員非僅知悉該機

關組織目標,尤其應知道大到知識型政府的發展方向,以及小到

知識管理的策略與方法,經由理念心智的改變,達到行政作為的

改變,因應變革管理需要。

焦點七:我國當前政府再造工程對公務人員的期待—形塑「知

識工作者」

【思考方向】

一、知識工作者的崛起

Page 102: 主題:公共管理的基礎理論 - web.thu.edu.twweb.thu.edu.tw/lu.bk/www/news/99-1.pdf · 主題:公共管理的基礎理論 焦點一:試說明Weber對傳統官僚制度的反省。

知識革命與知識經濟的來臨,組織的基本資源大致有四種:「科

技」、「人力」、「實體」、「財務」等資源。由於近年來人類科技文

明的躍進,改變了經濟發展的模式,「知識革命」已經來臨,21

世紀將是「知識經濟」的時代,「知識資本」將成為組織的第五種

資源。

二、二、知識工作者的特徵

依據杜拉克(Drucker)綜合歸納以下五點有關知識工作者的特徵:

(一)知識工作者與組織的關係:

1. 組織的基礎不是權力的運用而是信賴關係的建立,知識工

作者與組織是共生共存的平等關係,他們是組織的資產,

而非成本。

2. 知識工作者的工作生涯要比組織的平均壽命還長,具有移

動性,來: 去自如,成為不折不扣「四海為家」的人。

3. 知識工作者往往並非全職人員,他們依靠「自我管理」維

持信譽。

(一)知識工作者與主管的關係:

1. 知識工作者與主管之間是一種夥伴關係,但他們需要主管指引

工作的方向、標準和價值觀,以衡量表現與績效之間的關係。

2. 對知識工作者不能使喚,只能說服,領導這群人需要完全不同

的技巧。

(二)知識工作者的工作性質:

1. 必須對自己的貢獻負責。

2. 2.工作要不斷創新。

3. 主持續的學習和教導,成為工作中的一部分。

(三)知識工作者激勵的誘因:

知識工作者激勵的誘因較偏向於「自我的實現」及「靈性的需求」

的滿足,其激勵的方式為:

1. 從工作中得到的滿足,要比從薪資上得到的滿足更為重要。

2. 知識工作者需要面對挑戰,和組織共同賦於使命,並追求績

效。

3. 知識工作者不能以財務方式來衡量其貢獻,應激發他們對組織

的「效命」

Page 103: 主題:公共管理的基礎理論 - web.thu.edu.twweb.thu.edu.tw/lu.bk/www/news/99-1.pdf · 主題:公共管理的基礎理論 焦點一:試說明Weber對傳統官僚制度的反省。

(四)決定知識工作者生產力主要的因素:

決定知識工作者生產力主要的因素是根本態度的改變,其細節則

表現在以下六個因素:

1. 工作的任務或使命。

2. 知識工作者必須要有自主性,並能自我管理。

3. 不斷的創新再創新。

4. 持續不斷的學習與不斷的教導。

5. 質量並重,甚至於獲得最佳品質時才決定工作的產量。

6. 知識工作者是資產,而非成本。

焦點八:「多元化管理」的理論與實務(陳其祿,2008)一、概念起因

(一)多元化管理概念的出現主要與人口、經濟結構的轉變,以及全球化等因素有關。換言之, 隨著人口的增加、經濟活動的多元性,以及全球跨域之流動,世界各國無不面對一更為複雜且多元的工作人力群體。(二)此一群體的特徵即是有著高度的「異質性」(heterogeneity),不僅在性別、年齡、族裔,甚至是文化、價值、語言、宗教、性取向與身心狀態上皆有著極大的差異。(三)形成此一狀態的原因除人口本身之增長與結構之變遷外,譬如壽命的延長、女性就業人數的增加、族群的融合等,更有著經濟結構轉型所產生的驅動力,如團隊合作(teamwork)的管理需求、全球市場的整合發展,以及因組織合併(merger)或聯盟(alliance)所產生的多元化工作團體之需要。(四)此外,多元化工作團體的形成與人權觀念及實務的進步亦有密切關連,這是因為平等的就業機會(equal employmentopportunity)、弱勢之保障(minority protection)與對差異性的尊重(valuing differences)等皆是建立多元化工作團體之前提要件。而此類觀念與制度實務之改革也已逐漸蔚為風潮,自然更促進了多元化人力工作團體之興起及發展。二、概念界定:多元化管理的意涵主要是指透過領導溝通、組織設計及價值態度變革之相關行動,來達成對一個異質性工作團體的管理,其除了

Page 104: 主題:公共管理的基礎理論 - web.thu.edu.twweb.thu.edu.tw/lu.bk/www/news/99-1.pdf · 主題:公共管理的基礎理論 焦點一:試說明Weber對傳統官僚制度的反省。

希望能建立一個平等及相互尊重的工作環境之外,更欲促成不同成員潛質及特色之發揮,以謀取工作人力的最大綜效三、主要功能

(一)消極的多元化管理

1.多元化管理的第一個發展階段係以消除工作環境中的不平等因

素及對少數或弱勢之權益進行保障為主要目標。此種發展最早可

追溯至美國1960年代起所通過的各種「就業機會均等」(EqualEmploymentOpportunity)之民權法案,這些法案主要是針對以往

因偏見( prejudice ) 及歧視(discrimination)等因素而受到工

作機會限制之女性與少數族群進行僱用機會的保障,以杜絕和預

防不當的僱用歧視與限制。

2.嗣後在1970年代則有更進一步的少數與弱勢族群就業保障的民

權法案通過,即所謂的「弱勢優先」(Affirmative Action)法案,此類法案除強調公平的就業機會之外,更要求雇主必須能

僱用一定數額比例(quotas)的少數或弱勢族群,以確實達成組

織多元化及弱勢保護之目標。申言之,「弱勢優先」法案係賦予

雇主更高的規範要求,以使少數及弱勢族群無論在工作機會的爭

取及實際就業的數量上,皆能獲得確切的保障。

(二)積極的多元化管理

1.多元化管理的目標並非僅是為了矯正工作環境中的不公平與歧

視限制,其更希望能積極運用多元化工作團隊中的差異特色來獲

取工作之綜效。就前述消極面的多元化管理而言,其只能被視為

是被動的法令順服,還未能真正體現與發揮多元化工作團隊之優

勢。因此若欲確實做到積極的多元化管理。

2.最重要的係應「重視差異」(valuingdifferences)並營造一個「多

元友善的工作環境」(diversity-friendly environment);此處雖然

只是強調對差異的重視,但卻已與消極面的多元化管理有很大的

不同,蓋「就業機會均等」與「弱勢優先」等措施只是將單一化

組織(monolithic organization)(即對異質性成員採取排斥態度

並抗拒多元化觀念及實務的傳統組織)的限制藩籬予以打破,但

並未使組織及其主流成員能真正認同與珍視異質性與多樣性對其

組織的貢獻。

Page 105: 主題:公共管理的基礎理論 - web.thu.edu.twweb.thu.edu.tw/lu.bk/www/news/99-1.pdf · 主題:公共管理的基礎理論 焦點一:試說明Weber對傳統官僚制度的反省。

3.譬如美國1978年的「文官改革法」(CivilService Reform Act)便已將文官團隊的多元性視為是一種文官改革的重要目標,明確

肯定不同族群成員對公共行政之貢獻。此外,又如我國考試院所

公布之「國家考試性別平等白皮書」、「身心障礙人員考試制度

白皮書」、「原住民族考試制度白皮書」等皆有明確重視不同族

群差異及機會均等之目的。

三、多元化組織

(一)積極的多元化管理除了對於成員的異質性給予重視且營造

一個對多元化友善的工作環境之外,其更進一步的作法便是積極

地去應用和發揮成員異質性的特色,並使其成為組織的資產

(assets)與競爭力的來源。換言之,積極的多元化管理係將組織

的多元化特色轉變為最重要的組織資源;對組織而言,成員的異

質性不僅只是一個被接受的狀態,更是組織所欲追求的一項價

值,因為在當代競爭的環境中,唯有多元的成員、觀點及技術方

能克服組織所面對的各種問題,也才是組織競爭力的核心關鍵,

所以積極的多元化管理本身就是在追求多元化,亦即在創造一個

「多元文化的組織」

(二)為了能使組織更具多元化或建立一個「多元文化的組織」,

積極的多元化管理係從人力資源的招募、選用、訓練,組織的設

計與工作的安排,甚至是組織策略的規劃與願景(目標)的建構

等皆應有多元化因素的考量,其最終目的在促使所有成員皆有自

我發展(self-development)之機會,充分發揮其不同之潛能,同

時在一尊重差異與友善的環境中,能成為一具有多元文化且合作

共生的整體性系統( holistic system),並展現問題解決與工作之

綜效。

主題:政治管理

焦點一:何謂危機管理?行政機關的危機管理活動,應包括那些

內涵?試分析說明之。【答題方向】

Page 106: 主題:公共管理的基礎理論 - web.thu.edu.twweb.thu.edu.tw/lu.bk/www/news/99-1.pdf · 主題:公共管理的基礎理論 焦點一:試說明Weber對傳統官僚制度的反省。

一、危機管理之意涵:

1.危機管理:為減少災難發生時,人民生命財產之損失,發展並執行動員相關

人力、物,力各項措施之過程。

2.組織危機管理:組織為了避免或減輕危機情境所帶來之嚴重威脅,所從事之長期

性規劃及不斷學習、適應的動態過程。

二、危機管理的活動:

(一)危機爆發前之運作與活動:1. 危機感應系統:組織若能針對早期的危機警訊加以察覺,並採取適當的因應

措施來遏止其發生,那麼組織便能將危機消弭於無形而達到

善良管理的最高境界。

2. 危機計畫系統:危機計畫系統的目的就是在事前對可能發生的潛在危機,預

先加以研究討論,以發展出應變的行動準則。

3. 草擬危機計畫說明書:所謂草擬危機計畫說明書,就是對危機情境作沙盤推演的過

程(此即危機處理劇本)。4. 危機訓練系統:最主要的目的是想透過此種訓練的過程,使其成員能夠培養

出分析的能力與知識取得的能力,並從中學習及培養獨立判

斷的能力,以便其在危機的情境下能作出創造的決策,並能

以彈性的行動來解決危機。

(二)危機發生時之運作與活動

1.危機管理小組:危機管理小組是一個智囊團,它是由各種對危機情況十分明瞭,

並能針對特殊個案作出評估的專家所組成。

2.危機情境監測系統:組織中的危機情境監測系統應對危機情境加以監測,其可運用特

有的監控技術及良好溝通網路,對危機作追蹤並將所得的情報向

危機管理小組報告,使得該小組能夠掌握可靠的訊息來對危機情

Page 107: 主題:公共管理的基礎理論 - web.thu.edu.twweb.thu.edu.tw/lu.bk/www/news/99-1.pdf · 主題:公共管理的基礎理論 焦點一:試說明Weber對傳統官僚制度的反省。

境作評估。

3.危機資源管理系統:為了能有效解決資源運用的問題,組織平時便應設立危機資源管

理系統,包括資源的種類、數量、配置地點等,從而建立資源管

理系統的資:料庫,以供危機管理小組運用。

(一)危機發生後之運作與活動:1. 成立評估系統並進行評估:

成立該小組主要目的在於負責對整個危機管理活動作評估及調查

的工作,以供組織修正危機計畫時參考。

2. 加速復原工作的進行:危機發生後,組織對其內、外部遭受到傷害的利害關係者,應予

以適當的救助與補償。

3. 從教訓中學習與危機管理的再推動:最主要的工作還是要從危機事件中學習教訓,並將此學習回饋至

危機前的準備工作,以利危機管理活動的再推動。

焦點二:危機管理的概念模型與實務。

【思考方向】

一、危機準備模式

危機準備模式(Crisis Preparedness Model)係建構在 Reilly

(1993)的模型基礎上,其包括三個主要階段,依序為:

1.一般準備階段,該階段中尚包含兩個重點:(1)危機的策略規

則、(2)有益於危機管理的文化;

2.早期預警訊息的交流階段;

3.危機管理階段,其中重點為:(1)迅速的回應、(2)資源動

員,以及(3)有效的資訊流通等。

二、危機管理六階段

危機管理是策略管理中一個很重要的部分,其本質在確保組

織的穩定和生存,並追求組織目標的過程中繼續成長。有效的危

機管理是需要系統化及訓練化,而此「二化」則是建立在對危機

的警戒、管理上的敏感度、組織反應的敏捷能力,以及組織認知

Page 108: 主題:公共管理的基礎理論 - web.thu.edu.twweb.thu.edu.tw/lu.bk/www/news/99-1.pdf · 主題:公共管理的基礎理論 焦點一:試說明Weber對傳統官僚制度的反省。

到謹慎規劃的重要性,因此提出「危機管理六階段模式」。此六

階段分別為:

1.克服(coping)

2.重新思考(rethinking)

3.創意改善(initiating)

4.察覺(sensing)

5.干預(intervening)

6.防堵(sandbagging)。

三、公私協力模型之應用

FEMA 曾應用公私協力模型於美國西雅圖(International

Strategy for Disaster Reduction, 2005),藉以進行災害防救及危機管

理之行動,並且具有相當之成效。其成功的原因在於:公部門及

民眾確認、理解彼此雙方的需求;增進對地區的熟悉及增強地區

減災的能力;維持地方參與和廣泛涉入的信念。而此次行動之目

標即在於社群安全之自治,使所有成員均能對社群有所貢獻。由

此我們可以知道公私協力模型在災害防救上有其可資應用之處,

而其定義如下所示:公私部門協力乃指政府部門與公民、營利組

織與非營利組織之間處於平等地位,以信任為彼此協力之基礎,

以公民參與及其附加價值為協力追求之導向,藉由雙方資源的交

流以達到公共利益之目的,且在協力過程當中經由雙方契約的訂

定,確立彼此的權責,以建立公私協力的公共服務責任網。

肆、整合性危機管理系統(IEMS)

「整合性危機管理系統」(Integrated Emergency Management

System,IEMS)(FEMA, 1983)即是以多目標途徑為主要考量,

其基本假設在於各類型的危機均有共同的危機特性。IEMS 將危機

管理分為以下四大階段的政策規劃及執行過程:

一、紓緩政策(mitigation policy)

此一階段之政策包括規劃足以減輕災難損害之各種因應措

施。危機管理中之紓緩性政策可分為兩種類型(1)結構性

(structural)—例如推動興建水壩防洪計畫、改善鎮暴裝備及技

術計畫等;(2)非結構性(un-structural)—如研擬房屋建築法規

增強抗震能力,訂定災難保險給付規則,規劃土地(或山坡地)

Page 109: 主題:公共管理的基礎理論 - web.thu.edu.twweb.thu.edu.tw/lu.bk/www/news/99-1.pdf · 主題:公共管理的基礎理論 焦點一:試說明Weber對傳統官僚制度的反省。

使用規則,及改革獎勵及處罰性稅制誘因(tax incentives and

disincentives)。

二、準備政策(preparedness policy)

此類型政策主要是在發展因應危機的運作能力。其計畫包括

1.設計危機運作計畫,2.建立危機資訊溝通網路,3.建立緊急

事件處理中心,4.設立危機警報系統,5.緊急事件處理人員訓

練計畫及模擬,6.資源管理計畫。

三、回應政策(response policy)

此一階段的政策特色在於強調當危機已無可避免地轉換成災

難時,所應採取之行動。例如1.醫療救援系統,2.緊急事件處理

中心之運作,3.救難及撤離計畫,4.災民收容,5.第二波災難發生

可能性預防措施等。

四、回復政策(recovery policy)

短程回復政策包括重建基本民生支援系統,例如將水源、電

力恢復至最起碼之運轉程度。而就長期回復政策言應重建交通運

輸系統、污染放射物之控制、疾病衛生控制等。

焦點三:非營利組織可以透過哪些方式對政治產生影響?

【答題方向】(孫本初,2005)

非營利組織可以透過多種方式對政治產生極大的影響:

1. 聯盟策略:

所謂聯盟(Coalitions),是團體間為達成某一公共政策的目標,而存

在的明顯工作關係。公益團體聯盟活動的存在係一種吊詭的兩面

性。一方面,公益團體因為尋求組織的壯大,故十分樂意接受其

他團體的聯盟提議,但另-一方面,強勢的公益團體由於不願被搭

公共政策的便車,弱勢的公益團體亦不願永遠籠罩在強勢團體的

羽翼下,故又常常會表現出分離的傾向。

2. 資訊策略:

基於對政府決策資訊不足的認識,或對於決策者專業能力的憂

心,公共利益團體主動提供有效的資訊,協助政府作出更趨理性

的可行政策,是為資訊策略。

3. 困窘策略:

Page 110: 主題:公共管理的基礎理論 - web.thu.edu.twweb.thu.edu.tw/lu.bk/www/news/99-1.pdf · 主題:公共管理的基礎理論 焦點一:試說明Weber對傳統官僚制度的反省。

所謂困窘策略,是指揭露政府不良的決策,引發社會輿論的聲討,

以刺激政府部門給予改革,或利用政府官員對某議題的強烈反

應,使原有未受關注的議題擴散開來,再利用說服的技巧,使社

會大眾依照非營利組織所希望的方向來型塑民意,以形成政策壓

力。

4. 遲滯策略:

指非營利組織透過大眾傳播媒體與學者,對政策表示反對,或透

過具有影響力的民意代表在議案審查的各階段對法案提出修正的

意見,藉此從事討價還價的行動。

5. 訴訟策略:

司法是政策執行中的一個環節,透過訴訟的提出與裁決,除了能

夠使被主張的公平與正義獲得實現外,法官在釋法過程中也往往

對於政策價值給予不同的解釋,而實際影響了公共政策的執行。

6. 選區壓力策略:

非營利組織在從事壓力策略時多會依下面幾種方式進行: (1)刊登

廣告,以大篇的廣告來吸引大眾的興趣,喚起公益團體的支持者。

(2)利用發行刊物來表明所支持或反對的候選人,或其在議會中的

發言記錄(3)選擇會員中較具有影響力的人士,動員其寫信或打電

話給予其選區中的議員(4)要求團體中的會員寫信或打電話來表

達立場。

焦點四:非營利組織的挑戰以及期待?

【答題方向】(孫本初,2005)

儘管非營利組織是公民參與的具體表現,可彌補市場機能與政府

職能之不足然而在實際運作上,非營利組織或者因為經營的壓

力,或者因為政治的考量,有時會偏離非營利組織的原有「使命」,

或者面臨經營的效率問題,因此面臨相當大的社會挑戰與壓力。

1. 非營利組織必須面對的是社會要求課責的挑戰:

現代社會已邁入「課責的時代」、「透明化的時代」,在「政府」與

「企業」均納入社會課責的行列當中(例如強調企業的社會責任、

政府的公開透明等),非營利組織亦有必要向公眾或相關政府部門

展示其服務的效率與效能,證明其妥善運用有限社會資源的能力。

Page 111: 主題:公共管理的基礎理論 - web.thu.edu.twweb.thu.edu.tw/lu.bk/www/news/99-1.pdf · 主題:公共管理的基礎理論 焦點一:試說明Weber對傳統官僚制度的反省。

2. 非營利組織必須克服志願性社會服務失敗(Voluntary Failure)的

壓力:

隨著社會的發達與服務需求的膨脹,所謂營利性的社會服務機構

(如老人安養中心)紛紛成立,加入服務的競爭,競爭帶來服務品

質的比較。所以非營利組織一如其他各類型的組織,必須面對開

放系統的環境壓力,適時地進行計畫性的組織更新和變革,這些

變更包括禾基本使命的再界定、服務對象的確立、經營策略的調

整以及內部管理的檢討等,唯有如此,非營利組織才足以迎接新

的挑戰。

3. 非營利組織必須克服專業化的壓力:

Salamon(1994)強調,非營利組織並非政府失靈或者市場失靈的替

代品,而是具有特殊專業能力的部門,因為這樣的專業能力,非

營利組織可以和政府或企業負起不同的社會貢任而成為三足鼎立

的狀態。然而,截至目前為止,非營利部門專屬的專業能力所指

為何?似乎仍不容易定義,相較於企業的「生產效率」、政府的「公

平分配」,非營利組織如何找到一個可以相區隔的社會功能定位,

是日前非營利組織專業化的挑戰。

焦點五:政府與第三部門的協力關係

壹、第三部門的發展

在市場經濟體制下,我們透過營利部門提供的商業交易行為來得

到物質滿足;並從政府部門獲得安全的滿足。此外,隨著公民意

識的抬頭,如雨後春筍般的「第三部門」產生,且蔚為一股不可

忽視的力量,第三部門並非如營利部門以利潤為導向,也非如政

府部門以控制為目的,它是由社會的公民所發起,經由理念與使

命的傳達和實際的行動,為社會的另一些公民服務,而其中並沒

有金錢的利益,取而代之的是人道的精神,而此即是第三部門的

精神。

貳、第三部門的功能

在價值多元變遷急速的社會中,第三部門之所以存在並展現獨特

的功能,實因為其扮演各種積極的社會角色所致(Kramer, 1981;

Page 112: 主題:公共管理的基礎理論 - web.thu.edu.twweb.thu.edu.tw/lu.bk/www/news/99-1.pdf · 主題:公共管理的基礎理論 焦點一:試說明Weber對傳統官僚制度的反省。

江明修,1994):

(一) 先驅者:第三部門能敏感的體驗社會需求,以組織多樣與彈

性的特質發展具有創新的構想,適時地傳遞給政府。

(二) 改革與倡導者:第三部門深入社會,實際瞭解政府政策的偏

失,運用輿論或政策遊說等具體行動,促成社會變遷並尋求政府

的改善措施。

(三) 價值維護者:以倡導、參與改革精神改善社會,並主動關懷

弱勢團體。

(四) 服務提供者:發揮彌補(gap-filling)的角色,經常選擇政府

未做、不想做或較不願意直接去做的,但是卻符合大眾所需要的

服務來做。

(五) 社會教育者:利用刊物、舉辦活動、透過媒體的宣傳等方式,

負起傳遞特定人群需求的資訊,藉此嘗試提供新的觀念改革社會

大眾,以及決策者對社會的刻板印象或漠視態度,並補充正規學

校教育體系之不足。

參、政府與第三部門的協力關係一、公私協力的意涵:

(一)江明修、鄭勝分(2002)指出,公私協力是公民或第三部

門參與公共服務的重要方式,其目的不僅試圖將民間創業精神及

成本效益分析,帶入政府的服務功能中,更重要的是邀請民間組

織,在基於公民參與和共同承擔公共責任的自覺下,與政府共同

從事公共事務執行和公共建設工作。事實上,即使參與者都是私

部門,只要合乎公共利益,均可視為「合夥」關係(Waddock,1984)。Bailey(1984)則提出一較靈活的定義,認為公私協力

乃來自於超過一個部門之利益聯盟所為之策略行動。

(二)吳英明(1984;江明修、鄭勝分,2002)也認為,公私協

力係指特定事務的參與者形成一種不屬於政府也不屬於私部門,

而是屬於公私結合而成的關係,其參與者對該事務之處理具有目

標認同、策略一致及分工負責的認知與實踐,且雙方為了共同目

的,彼此信任。如果用音樂比喻,則公私協力就如同演奏一首交

響樂般,音符和音符間環環相扣,樂章和樂章間緊密銜接,也唯

有演出者同心協力,才能夠合奏出悅耳的曲目。

Page 113: 主題:公共管理的基礎理論 - web.thu.edu.twweb.thu.edu.tw/lu.bk/www/news/99-1.pdf · 主題:公共管理的基礎理論 焦點一:試說明Weber對傳統官僚制度的反省。

二、政府與第三部門關係模型:

由上述可知,政府與第三部門公私協力是解決政府財政負擔(特

別是人事成本),以及民眾需求的有效策略,但是政府與第三部

門之間應該是何種關係呢?吉爾登等﹙Girdon and Kramer &Salamon, 1992;陳定銘,2003﹚探討第三部門與政府關係時,認

為第三部門參與公共政策,可以從兩個層面加以區分,一是服務

經費的提供與授權,另一是實際服務的輸送者,並發展出四種的

關係模式。

(一) 政府主導模式:政府為經費與服務提供者,此為一般所謂的

福利國家模式。

(二) 雙元模式:政府與第三部門各自提供福利服務的需求,兩者

並無經費上的交集,而是處於平行競爭的範圍(各有其範疇),

同時由政府和第三部門提供資金與傳送服務,但是各有其明確的

範圍。

(三) 合作模式:典型的合作模式是由政府提供資金,第三部門負

責實際的服務傳送。但是相反的情形亦有可能,此種模型廣布於

美國。

(四) 第三部門主導模式:第三部門同時扮演資金提供與服務傳送

的角色。

焦點六:試說明電子化政府的定義、運作方式與推行之可能困

境。

【答題方向】

(一)電子化政府的定義:

1.「電子化政府」是一九九三年美國政府「運用資訊科技改造政

府」報告中提出的概念,強調「利用資訊科技來『革新』政府」。

2.除了結合政府各部門業務電腦化的成果,更企圖透過網際網路

塑造一個提供民眾各種天候服務的電子化或網路化政府,進而提

昇政府生產力與效率。

3.電子化政府即是政府機關運用資訊與通信科技形成網相連,並

透過不同資訊服務設施(包括電話、網際網路、公用電腦站等),

對機關、企業及民眾在其方便時間、地點及方式下,提供自動化

Page 114: 主題:公共管理的基礎理論 - web.thu.edu.twweb.thu.edu.tw/lu.bk/www/news/99-1.pdf · 主題:公共管理的基礎理論 焦點一:試說明Weber對傳統官僚制度的反省。

服務之總體概念。

(二)電子化政府網路服務運作方式

4. 資訊公布:

隨著政府發展,民眾的「民主意識」日漸覺醒,對「知的權利」

也愈有主張,向國家請求公開資訊,乃屬人民所擁有的一種基本

權利。

5. 線上申辦:

民眾可以透過政府的網站下載洽辦各項業務的表格或直接進入政

府申辦業務。例如,國稅局開放民眾可於網路上報稅,節省民眾

往返的時間,並透過業務電腦化縮短作業時間。

6. 電子民主:

資訊科技的發達,穿透了組織與組織的界限,當然更穿透公民與

政府部門間的界限,使得公民只要一個按鈕,就輕易地透過網際

網路,直接進入政府所提供的資系統,並且與該部門進行對話,

因此,有人認為這是末來的民主政治將是按鈕式的民主政治,而

電子民主下的公民則稱為網路公民。

(三)電子化政府推行之困境

1.數位落差導致公平性的問題:

偏遠地區或家境困難之民眾,恐無法具有足夠經驗及技術能力使

用電子化政府,如何使這些邊陲民眾亦能具有足夠能夠接近利用

電子化政府,可為電子化政府推行是否成功的指標之一。

3. 推動成果的迷思:

電子化政府在推行之初,著重硬體績效無可厚非,但於硬體績效

完成之後,應實際落實電子化政府為民服務之目的。

4. 政府資訊公開完整性及兼具「質與量」之問題:

電子化政府自資訊提供而言,係希望民眾可以透過網際網路,打

破地域的限制,獲取政府機關之訊息,但目前政府機關之資訊,

除了在「量」方面有所增加外,資訊的「品質」及完整性,較少

被論及。

5. 個人資料保護的問題:

政府機關在設計資訊服務系統時,未能充分考量人民的技術能

力,以及人民對制度與系統的信任傾向,以致於受到惡意第三者

Page 115: 主題:公共管理的基礎理論 - web.thu.edu.twweb.thu.edu.tw/lu.bk/www/news/99-1.pdf · 主題:公共管理的基礎理論 焦點一:試說明Weber對傳統官僚制度的反省。

的侵害(例如駭客入侵或詐騙集團的誘導式欺騙).或推行無必要的

計畫以蒐集公民的個人資料,在在都侵害公民隱私並形成資料保

護的漏洞。

6. 官僚權力集中的問題:

部分學者憂心資訊科技一但為有權者所控制,則資訊發達可能產

生令人悲觀的後果,諸如權力更為集中,擴大提供資訊者與接收

資訊者的不公平狀態. ,惡化個人與官僚組織間的裂痕等。